CCJM delivers practical clinical articles relevant to internists, cardiologists, endocrinologists, and other specialists, all written by known experts.
Malignant bowel obstruction occurs in 5% to 51% of women with ovarian cancer and in 10% to 28% of patients with gastrointestinal cancer, predominantly in the advanced stages.1 Median survival after its onset ranges from 30 to 90 days.2–5
Its symptoms are challenging to manage, since nausea, vomiting, colic, and abdominal pain, which are common, cause significant physical distress and demoralization. The decision whether to correct it with surgery requires an individualized approach and a clear understanding of the goals of care and expected survival in the individual patient.
This review focuses on the management of inoperable malignant bowel obstruction and includes discussion of hydration, nutrition, and endoscopic palliative options.
WHAT ARE THE DIFFERENT TYPES OF OBSTRUCTION?
Bowel obstruction may be mechanical or functional, partial or complete, and may occur at one or at many sites. Tumors can impair bowel function in several ways6–8:
Intraluminal tumors can occlude the lumen or act as a point of intussusception.
Intramural tumors can extend to the mucosa and obstruct the lumen or impair peristalsis.
Mesenteric and omental masses or malignant adhesions can kink or angulate the bowel, creating an extramural obstruction.
Tumors that infiltrate into the mesentery, bowel muscle, or the enteric or celiac plexus can cause dysmotility.
Cholangiocarcinoma, pancreatic carcinoma, and gallbladder carcinoma are the most common tumors causing duodenal obstruction.9 Distal obstruction is caused mainly by colon and ovarian cancer.
Obstruction can be due to treatment
In a minority of patients, obstruction is unrelated to the cancer and is instead due to adhesions arising from surgery, radiation therapy (causing enteritis and strictures), desmoplastic reactions to intraperitoneal chemotherapy, torsion, or internal hernias.10–12
In rare cases, a patient has intestinal pseudo-obstruction from paraneoplastic destruction of enteric neurons, or severe ileus from anticholinergic or sympathomimetic drugs, as seen with acute colonic pseudo-obstruction (Ogilvie syndrome).13
Physiologic reactions to obstruction
Malignant bowel obstruction stimulates gastric, biliary, pancreatic, and intestinal secretions, decreases intraluminal sodium and water reabsorption, and increases mucosal sodium and water secretion.6,14 In response to the obstruction, peristalsis increases, and prostaglandin, vasoactive intestinal peptide, and nociceptive mediators are released. Vasoactive intestinal polypeptide perpetuates a cycle of secretion, distention, and contraction that leads to intestinal hyperemia, bowel edema, and accumulation of fluid in the lumen.8,10,11,15
Signs and symptoms depend on the site
The site of obstruction determines the signs and symptoms patients experience.7,14 Obstructions high in the gastrointestinal tract are associated with greater symptoms but fewer signs than colonic obstructions.1 Patients with proximal small-bowel obstruction have more severe nausea and a greater number of episodes of emesis, but they have relatively normal plain radiographs of the abdomen, which do not have the characteristic air-fluid levels commonly seen with distal small-bowel obstruction.
Most malignant obstructions remain partial, but increasing abdominal distention, worsening nausea, vomiting, abdominal pain, and obstipation over 1 to 2 weeks1 suggest progression to complete obstruction.
IMAGING TESTS FOR MALIGNANT BOWEL OBSTRUCTION
What is the value of plain radiography?
Figure 1. Top, plain radiography shows dilated loops of small bowel (arrows) in the mid-abdominal region of a woman with a small-bowel obstruction from advanced ovarian cancer. Bottom, computed tomography in the same patient shows multiple, dilated loops of small bowel and air-fluid levels (arrows) in the anterior midline of the abdomen.Plain radiography of the abdomen (kidney, ureter, bladder views) has only modest specificity and sensitivity in detecting bowel obstruction (Figure 1). In a patient who has symptoms of obstruction, overreliance on plain radiography can lead to false reassurances that there is no obstruction. The absence of air-fluid levels, dilated loops of bowel, or thickened bowel loops does not exclude malignant bowel obstruction. The overall accuracy of kidney, ureter, bladder radiography is reported to be as low as 50%, with more than 75% of plain films classified as nondiagnostic or not helpful.16,17
Despite these limitations, plain radiography is useful in assessing constipation and its severity as a potential cause of symptoms, and thus it remains an important initial imaging study in almost all patients with suspected malignant bowel obstruction.17,18 It is also used to assess response to treatment.
When do you need contrast radiographs?
Contrast radiography (barium swallow or barium or Gastrograffin enema) is helpful in patients with symptoms of dysmotility from suspected bowel obstruction. It defines the site or sites of obstruction and the extent of the obstruction with a fair degree of accuracy.7,19 Single-contrast studies, if positive, exclude opioid-induced bowel dysfunction or pseudo-obstruction in 83% of patients, with a sensitivity and specificity of 96% and 98%, respectively.8,20 Small-bowel follow-through with barium is more appropriate for low-grade obstructions or for symptomatic patients with a normal kidney, ureter, bladder radiograph.19
However, contrast radiography is limited by the patient’s ability to swallow barium or water-soluble contrast agents, and it can worsen nausea or vomiting.17,18 Also, barium is not absorbed systemically and may interfere with subsequent radiologic studies. Large volumes of contrast agents increase the risk of aspiration pneumonia in patients with poorly controlled nausea and can lead to severe impaction proximal to the obstructed site.8
Is enteroclysis better than barium swallow?
Enteroclysis, ie, injecting radiographic contrast into the bowel via a nasoduodenal tube, has some advantages over the barium swallow technique for detecting partial small-bowel obstruction, since it bypasses the stomach and allows for therapeutic decompression as well as direct visualization of the area of concern.17,18 Enteroclysis radiography objectively gauges severity of intestinal obstruction and bowel wall distensibility, which is an advantage over other imaging studies. Its sensitivity is 100% and specificity 88% in experienced hands.17 Enteroclysis studies also detect nonobstructing intraluminal tumors when computed tomography (CT) is not diagnostic.17,18,21
The drawbacks to enteroclysis are that it is technically difficult to perform and that few radiologists are trained in it.
When is CT useful?
CT is the primary imaging study for patients with obstructive symptoms and a history of abdominal malignancy or a palpable abdominal mass17,20,22,23 (Figure 1). It has a specificity of 100% and a sensitivity of 94%. It plays a major role in decision-making regarding surgery, endoscopy, or palliative interventions,7,19 as it locates the obstruction and differentiates benign from malignant causes with a fair degree of precision.22
CT findings in malignant bowel obstruction may include:
A mass at the site of obstruction or within the original surgical field
Lymphadenopathy
Abrupt transitions in luminal diameter or irregular thickening of the bowel wall at the site or sites of obstruction.7
SURGERY: A DIFFICULT DECISION
Is the patient fit for surgery?
Surgery for malignant bowel obstruction should not be done in patients who have advanced malignancies with bulky intra-abdominal metastases or cancer that has spread outside the abdominal cavity without taking into account treatment options for the cancer, the patient’s nutritional status, and the goals of care.
The role of abdominal surgery (debulking, resection, or bypass) in advanced cancer remains unclear and controversial.24 From 42% to 80% of patients report that symptoms improve after surgery, but recurrent obstruction occurs in 10% to 50%.10 Even in patients with low tumor bulk and good nutritional status, 30-day mortality rates range from 5% to 40%, and complication rates range from 9% to 90%.3,4,6,7,10,14
Outcomes after surgery depend on patient selection criteria perhaps as much as on the surgeon’s experience and skill. Patients with more advanced cancer who have had multiple surgical procedures and those with cancer that does not respond to chemotherapy and radiation present the greatest challenge to surgeons.23
What is the benefit of surgery?
Reports of palliative surgery have included information about 60-day survival rates after the operation, but a number of factors may be more meaningful in this context, such as postoperative symptoms, the patient’s overall wellbeing, how the original symptoms respond to the surgery, complications, and length of hospitalization.14 The paucity of published, validated, patient-related outcome data on which to gauge the value of surgery and the lack of a standard definition of “benefit” further confuse the objective determination of whether these patients benefit from surgery.
In a cohort with advanced ovarian cancer and bowel obstruction, surgery was detrimental to survival and quality of life for all subgroups, and most patients died in the hospital.6
The risk of surgery for malignant bowel obstruction is presumably higher than for abdominal surgery for other indications, since many of the patients are debilitated from their cancer and chemotherapy, and many are malnourished.23 Even when taking into account a potential selection bias in favor of surgery, several studies have reported no significant difference in 30-day mortality rates or median survival between operative and nonoperative groups.2,12 Neither the type of obstruction nor the extent of the surgery influenced outcomes. Surgical outcomes are best in patients with a benign cause of obstruction; little benefit is seen in operating on those with abdominal carcinomatosis.12
Nevertheless, surgery is beneficial in a select few. For patients with a good performance status, slowly progressive cancer, and an expected survival of more than 6 months, surgical bypass or resection is preferred.7,12,25 The challenge is to identify these surgical candidates, taking into account prognostic factors such as nutritional status, tumor burden, performance status, presence of ascites, advanced age, extensive prior chemotherapy or radiotherapy, and diffuse carcinomatosis.3,10,12,20,23
Is surgery consistent with the goals of care?
Crucial to decision-making are the goals of care. Since palliative surgery carries a low level of evidence for benefit in terms of quality of life and survival, time should be set aside to thoroughly review the patient’s medical condition, to explore options, and to clarify expectations and goals of care.3,10 Family members should be invited to be present during these discussions and to be involved in the decision-making process.
WHAT IS THE BENEFIT OF GASTRIC OR COLONIC STENTING?
Endoscopic procedures are alternatives to surgery and offer a palliative option in malignant bowel obstruction. Endoscopic procedures are associated with a shorter hospital stay and quicker recovery than after laparotomy.9,26–30 In certain situations, stenting serves as a bridge to surgery, allowing time to mitigate comorbid conditions, to enhance nutrition, and to complete staging, while relieving symptoms.27–29,31,32 Definitive surgery can be done as a single-stage procedure without a diverting enterostomy.
Self-expanding metal stents for gastric outlet, small-bowel, and colonic obstructions are an option in patients who have incurable metastatic disease who are unfit for surgery, in patients with a single point of obstruction or locally extensive disease, or in patients who do not want to undergo laparotomy.28–30
Technical and clinical success rates for colorectal stenting are high (88% to 93%).26,27 Stenting is more successful for left-sided colonic obstructions than for proximal colonic obstructions. Even for patients with extracolonic malignancies such as ovarian cancer, the technical success rate of colorectal stenting is 87%.26 However, patients with unrecognized peritoneal carcinomatosis or multifocal bowel obstruction are less likely to have symptomatic relief even after successful stenting.6,9
Contraindications to stenting
Absolute contraindications to stenting are colonic or tumor perforation with peritonitis. A relative contraindication is a rectal tumor within 2 cm of the anal margin. Stenting in this circumstance leads to tenesmus and incontinence.33
Complications of stenting
Death rates during colorectal stent insertion are less than 1%. The hospital stay and incidence of complications are significantly less than with surgery.26,30
Stent migration occurs in 10% of cases and is asymptomatic, but half of patients with this complication require a repeat intervention. The risk of migration is greater if chemotherapy or radiation therapy succeeds in shrinking the tumor.
Bleeding occurs in 5% of cases, usually from the underlying tumor.
Perforation occurs in 4%, but the rate increases to 10% with the use of dilatation before stent placement.
The rate of recurrent obstruction from tumor ingrowth, overgrowth, or fecal impaction is 10%.9,26,29 Recurrent obstruction may be treated with additional stents inserted within the original stent.9
GASTRIC OUTLET OBSTRUCTION: SURGERY VS STENTING
Gastrojejunostomy has in the past been the treatment of choice for gastric outlet obstruction. Certainly, patients with slow-growing tumors and an expected survival of greater than 60 days may be considered for this bypass procedure; those with a short tumor length, a single site of obstruction (preferably in the pylorus or proximal duodenum), a good performance status, and a life expectancy greater than 30 days are good candidates.7 Nevertheless, for patients with advanced cancer and poor performance status, gastroenterostomy carries a significant risk of morbidity and death.28
Endoscopic stenting of gastric outlet obstruction has a greater success rate, a shorter time to oral intake, a lower morbidity rate, a lower incidence of delayed gastric emptying, and a shorter hospital stay compared with gastroenterostomy.28,29 Technical success rates of stenting are 90%, and 75% of patients have resolution of nausea and vomiting.7 Stenting is generally not possible if the obstruction occurs beyond the ligament of Treitz.
Patients who are expected to survive less than 1 month or who have rapidly progressive disease, overt ascites, carcinomatosis, or multiple sites of obstruction should be managed with percutaneous, endoscopically placed gastrostomy tubes.7
Late complications of stenting for gastric outlet obstruction are occlusion with food or ingrowth of tumor through or around the wire mesh.7 This may require laser therapy or placement of a second stent, or both.
DRUG THERAPY
Medical therapy can palliate symptoms of malignant bowel obstruction for most patients.34 Recommendations have been published by the Working Group of the European Association for Palliative Care.24 Symptom management is focused on pain, nausea, and vomiting.
Which drugs can I use for abdominal pain?
Patients experience two types of abdominal pain: continuous and colic. Each type of pain requires different treatment approaches and classes of drugs.
Potent opioids such as morphine, hydromorphone (Dilaudid), and fentanyl (Fentora) are used to relieve continuous abdominal pain.7 The dose is titrated for adequate relief. Subcutaneous, intravenous, sublingual, and transdermal routes can be used if nausea and vomiting prevent oral administration.
However, opioids can aggravate colic by stimulating circular smooth muscle, leading to segmental contractions. Opioid-sparing adjuvant drugs such as ketorolac (Toradol) may improve colic and continuous pain and prevent a partial obstruction from becoming a complete obstruction by sparing opioid doses.35
Colic may persist or worsen with the use of opioids. Drugs that reduce colic include the scopolamine drugs hyoscine butylbromide and hyoscine hydrobromide, glycopyrrolate (Robinul), and octreotide (Sandostatin).7,34–37
Which drugs are appropriate for reducing nausea and vomiting?
Phenothiazines reduce nausea and control vomiting. Chlorpromazine (Thorazine), prochlorperazine (Compro, Compazine), and promethazine (Phenergan) have all been reported to treat nausea successfully.35,37
Haloperidol (Haldol), a butyrophenone selective dopamine D2-receptor antagonist, has negligible anticholinergic activity. At low doses it produces less sedation than phenothiazines and is an ideal agent for patients with nausea and delirium.35 Doses range from 5 to 15 mg/day, given in divided doses or as intermittent or continuous intravenous infusions.
Anticholinergics, with or without somatostatin analogues, reduce gastrointestinal secretions, fluid accumulation, and vomiting. Anticholinergics bind to muscarinic receptors on enteric neurons in the myenteric and the submucosal plexus. Dosages:
Hyoscine butylbromide 40 to 120 mg/day.
Hyoscine hydrobromide 0.2 to 0.9 mg/day.7,34
Glycopyrrolate, a quaternary ammonium anticholinergic, has minimal central nervous system penetration and is less likely to cause delirium or cardiac side effects compared with tertiary amine anticholinergics such as atropine and scopolamine.38 The recommended dose is 0.1 to 0.2 mg subcutaneously or intravenously three to four times daily.
Octreotide, an analogue of somatostatin, blocks the release of vasoactive intestinal polypeptide, which is increased in malignant bowel obstruction.14,15 It reduces the excretion of water, sodium, and chloride into the bowel lumen and increases the absorption of electrolytes and water. It also inhibits pancreatic enzyme secretion and splanchnic blood flow. The result of all these effects is reduced luminal content, reduced motility, reduced vascular congestion of the bowel wall, and, in certain circumstances, reduced ascites.39
In small randomized trials, octreotide was more successful than anticholinergics at improving nausea, vomiting, and colic in patients requiring a nasogastric tube and in those whose symptoms were refractory to standard medical treatment.5,34,40–43 A recent case report found octreotide helpful in resolving symptoms of partial bowel obstruction that were unresponsive to standard measures.44
Octreotide is well tolerated and reduces the time patients require a nasogastric tube without significantly worsening xerostomia. High cost limits its use in American hospice care due to the Medicare capitated system of reimbursement for drugs and services, and as a result it is a second-tier drug despite evidence of its efficacy.
Octreotide doses are 100 to 200 mg every 8 hours.
Metoclopramide (Reglan), a dopaminergic antagonist, a 5HT4 receptor agonist, and a 5HT3 receptor blocker at doses greater than 120 mg/day, combines the action of a phenothiazine, which blocks D2 receptors in the central chemoreceptor trigger zone, with promotility actions through serotonin receptors (5HT4).35,37
Metoclopramide should not be used with anticholinergics or in patients with colic or complete obstruction.35,45 In some centers it is the first-line drug for functional or partial bowel obstruction.7 Dosages range from 40 to 240 mg/day.
Olanzapine (Zyprexa), an atypical antipsychotic, blocks multiple neurotransmitter receptors (D2, H1, Ach, 5HT3) responsible for initiating emesis. It is an option in patients whose nausea and vomiting fail to respond to standard antiemetics.46 Dosages range from 2.5 to 20 mg/day.
Dissolvable tablets are given sublingually, which makes olanzapine a versatile antiemetic in cases of intractable nausea. Our unpublished experience is that the sublingual route reduces nausea associated with malignant bowel obstruction and obviates the need for subcutaneous injections or intravenous antiemetic infusions.
Corticosteroids. Although how corticosteroids relieve malignant bowel obstruction is unknown, they are presumed to act centrally.37,45 In addition, they reduce peritumoral edema and luminal salt and water, and they also have antiemetic and analgesic properties.
Evidence from a meta-analysis found that 6 to 16 mg of parenteral dexamethasone per day reduced symptoms and improved bowel function in 60% of patients but did not change the prognosis.11
A trial of 4 or 5 days is adequate to determine response. If there is no response, the corticosteroid should be rapidly tapered. Side effects are minimal when corticosteroids are used short-term.
Combination therapy. Only rarely does a single drug resolve symptoms of malignant bowel obstruction. Antiemetics, analgesics, corticosteroids, antisecretory anticholinergics, and octreotide are often required in combination to achieve acceptable symptom relief.3,5,7,47
In a small prospective case series, the combination of metoclopramide 60 mg/day, octreotide 0.3 mg/day, and dexamethasone 12 mg/day with a single bolus of amidotrizoic acid (a contrast agent) improved intestinal transit within 1 to 5 days and resolved vomiting within 24 hours.45
Compatibility and the route of administration of medications are key considerations when choosing drug combinations.
WHEN TO CONSIDER A VENTING GASTROSTOMY
Patients with a poor performance status, rapidly progressive disease, peritoneal carcinomatosis, a life expectancy of less than 30 days, or multiple levels of obstruction benefit from placement of a percutaneous endoscopic gastrostomy tube (ie, a venting gastrostomy) rather than surgery if symptoms do not respond to drug therapy.7,48 There is compelling evidence that this procedure relieves nausea and vomiting in 80% to 90% of patients and restores some level of oral intake in many.5,6,48,49 A venting gastrostomy tube can be placed during surgical exploration, percutaneously with fluoroscopy, or endoscopically.9
There are no absolute contraindications to gastrostomy tube placement. It is feasible even in patients with tumors encasing the stomach, diffuse carcinomatosis, and ascites.48 However, massive ascites, previous upper abdominal surgery, or a large mass attached to the abdominal wall make tube placement difficult.
Complications are often local. Patients experience transient abdominal wall pain after the procedure. Dislodgement, bleeding, catheter migration, peritonitis, and necrotizing fasciitis are early complications. Others include skin excoriation from leakage of gastric contents, leakage of ascitic fluid from the site, and obstruction or dislodgement of the tube.48,49
Patients can be discharged from the hospital soon after the tube is placed, usually with fewer medications than for patients who undergo surgery.48 This is particularly important for patients with a short expected survival. Some patients at home benefit from hydration (less than 2 L/day) via an existing central venous port or peripherally inserted central catheter, or by hypodermoclysis.
WHEN IS A NASOGASTRIC TUBE APPROPRIATE?
Some patients with malignant bowel obstruction require a nasogastric tube early in their hospital course.12 Unfortunately, nasogastric tubes, if left in place, cause nose and throat pain, sinusitis, abscess formation, erosion of nasal cartilage, aspiration, esophageal erosion, pharyngitis, and social isolation.5,6
Nasogastric tubes should be a temporizing measure to vent gastrointestinal secretions, reduce abdominal distension, and improve nausea and vomiting while a decision about surgery is being made.13,24 If surgery is not feasible, one can avoid the long-term complications and discomfort of a nasogastric tube via medical management and earlier evaluation for venting gastrostomy in those with symptoms that respond poorly to optimal medical management.49
WHICH PATIENTS BENEFIT FROM TOTAL PARENTERAL NUTRITION?
The use of total parenteral nutrition in patients with incurable malignancies is controversial. Enteral and parenteral feeding can increase muscle mass and improve functional status and quality of life in a subset of patients who are not suffering from cancer-related cachexia.2,50,51 However, for those whose weight loss and malnutrition are consequences of tumor-mediated cachexia, as demonstrated by anorexia and an elevated C-reactive protein level, parenteral nutrition is unlikely to improve the outcome.51 For most terminally ill patients, retrospective studies have failed to show that parenteral nutrition improves overall survival, performance status, or quality of life.2,48,50–54
Total parenteral nutrition poses risks: it is invasive and requires central venous access, which predisposes to infection; it requires frequent monitoring of hydration and electrolytes; and it predisposes to thrombosis, diarrhea, hyperglycemia, and liver failure.50–56
Total parenteral nutrition may be justified in patients with minimal tumor burden who are candidates for definitive surgery, or in those with a good performance status early in the disease course who have not had chemotherapy or whose cancer responds to chemotherapy.2,50–56
The American College of Physicians discourages the routine use of parenteral nutrition in those with advanced cancer who are undergoing palliative chemotherapy, since few patients benefit and many experience side effects.53
Total parenteral nutrition is much like a medical intervention in that it should be offered or continued only if it provides benefit. Conversations at the time that it is begun must include adverse effects that will lead to its discontinuation, and criteria for response. In certain situations, a limited trial of parenteral nutrition may be considered for patients with an uncertain prognosis or for those who have potentially reversible conditions that limit oral intake.51 In such cases, there should be a clear understanding between patient and physician that parenteral nutrition will be discontinued if it fails to show benefit.53
ADDITIONAL CONCERNS OF PATIENTS AND FAMILIES
‘Will I starve to death?’
Starvation is a fear echoed by patients and families. Ethical discourse on the continuation of nutrition and hydration for the terminally ill has been polarizing.57–60 Withdrawal of nutrition can be perceived as euthanasia.
Advanced cancer patients in general do not experience hunger, and those who do require only small amounts of food for satiation.61 In one report, most patients died of their advanced cancer and not from starvation.52 Artificial hydration and nutrition will thus not influence survival and can even be a burden without benefit in the imminently dying.60 These patients should be encouraged to take food orally for pleasure, as long as it is tolerated, without consideration of end points such as weight gain, body mass index, or albumin levels.
Complaints of thirst and dryness of the mouth are relieved with mouth care, ice chips, lubrication to the lips, and sips of fluid, rather than by parenteral nutrition.59 Patients with a terminal illness experience relief from thirst with minimal intake. The symptom of thirst may be relieved without hydration.34,61 Adequate hydration requires smaller fluid volumes due to decreased body weight, decreased renal clearance of free water, and decreased insensible water losses from reduced physical activity.58
‘Can we continue intravenous hydration so he won’t die of thirst?’
Overzealous intravenous hydration may worsen the symptoms of malignant bowel obstruction. Overhydration can increase secretions in the gut lumen and worsen the secretion-distention-contraction cycle, leading to greater abdominal pain and to nausea and vomiting.7 There is a greater risk of fluid overload in these patients, since they have edema and excessive interstitial fluid. Most have a low serum albumin level, which results in movement of fluid from intravascular to interstitial spaces due to reduced colloid osmotic pressure. In these instances, overzealous hydration can lead to respiratory insufficiency and worsening edema.
In spite of numerous discussions in the medical literature of the benefits and burdens of continual hydration, there is no consensus or guideline. When a patient has limited oral intake, the decision to hydrate should be individualized, with careful assessment of the risks and benefits and in accordance with the patient’s or family’s wishes.57,58
Is treatment at home feasible?
Discharging patients with inoperable malignant bowel obstruction requires careful planning. Patients and family members need to be educated on the use of around-the-clock medications and symptom-targeted, as-needed drugs. Days before discharge, questions about diet need to be clarified. Education about total parenteral nutrition and gastrostomy tube care should be completed before discharge from the hospital.
Drug management should be simplified, or compatible medications should be combined into a single infusion. For example, morphine, glycopyrrolate, and haloperidol or metoclopramide are chemically compatible in standard intravenous solutions and can be combined.
Families feel less anxious about the foreseen and the possible unforeseen course of the illness if they can talk with hospice workers early on. This early involvement also facilitates the transition to home hospice care.
SUMMARY OF IMPORTANT POINTS
Patients with malignant bowel obstruction need a highly individualized approach, tailored to their medical condition, the prognosis, and the goals of care.
Surgery should not be routinely undertaken; less-invasive approaches such as gastric or colonic stenting should be considered first.
Combinations of analgesics, antisecretory drugs, and antiemetics can provide acceptable symptom relief in the inoperable patient.
A venting gastrostomy should be considered if drug therapy fails to reduce nausea and vomiting to an acceptable level.
A nasogastric tube should be used only as a temporizing measure, until symptoms are controlled medically or a venting gastrostomy is placed.
Total parenteral nutrition is of benefit only in patients with intermediate life expectancy who may otherwise die of starvation rather than from the cancer itself.
References
Mercadante S. Intestinal dysfunction and obstruction. In:Walsh D, editor. Palliative Medicine. Philadelphia, PA: Saunders/Elsevier, 2009:1267–1275.
Pasanisi F, Orban A, Scalfi L, et al. Predictors of survival in terminal-cancer patients with irreversible bowel obstruction receiving home parenteral nutrition. Nutrition2001; 17:581–584.
Pameijer CR, Mahvi DM, Stewart JA, Weber SM. Bowel obstruction in patients with metastatic cancer: does intervention influence outcome?Int J Gastrointest Cancer2005; 35:127–133.
Bais JMJ, Schilthuis MS, Ansink AC. Palliative management of intestinal obstruction in patients with advanced gynaecological cancer. J Gynecol Oncol2002; 7:299–305.
Laval G, Arvieux C, Stefani L, Villard ML, Mestrallet JP, Cardin N. Protocol for the treatment of malignant inoperable bowel obstruction: a prospective study of 80 cases at Grenoble University Hospital Center. J Pain Symptom Manage2006; 31:502–512.
Jatoi A, Podratz KC, Gill P, Hartmann LC. Pathophysiology and palliation of inoperable bowel obstruction in patients with ovarian cancer. J Support Oncol2004; 2:323–334.
Ripamonti CI, Easson AM, Gerdes H. Management of malignant bowel obstruction. Eur J Cancer2008; 44:1105–1115.
Roeland E, von Gunten CF. Current concepts in malignant bowel obstruction management. Curr Oncol Rep2009; 11:298–303.
Feuer DJ, Broadley KE, Shepherd JH, Barton DP. Surgery for the resolution of symptoms in malignant bowel obstruction in advanced gynaecological and gastrointestinal cancer. Cochrane Database Syst Rev2000;CD002764.
Feuer DJ, Broadley KE. Corticosteroids for the resolution of malignant bowel obstruction in advanced gynaecological and gastrointestinal cancer. Cochrane Database Syst Rev2000;CD001219.
Woolfson RG, Jennings K, Whalen GF. Management of bowel obstruction in patients with abdominal cancer. Arch Surg1997; 132:1093–1097.
Ripamonti C, Bruera E. Palliative management of malignant bowel obstruction. Int J Gynecol Cancer2002; 12:135–143.
Nellgård P, Bojö L, Cassuto J. Importance of vasoactive intestinal peptide and somatostatin for fluid losses in small-bowel obstruction. Scand J Gastroenterol1995; 30:464–469.
Böhner H, Yang Q, Franke C, Verreet PR, Ohmann C. Simple data from history and physical examination help to exclude bowel obstruction and to avoid radiographic studies in patients with acute abdominal pain. Eur J Surg1998; 164:777–784.
Maglinte DD, Kelvin FM, Sandrasegaran K, et al. Radiology of small bowel obstruction: contemporary approach and controversies. Abdom Imaging2005; 30:160–178.
Maglinte DD, Howard TJ, Lillemoe KD, Sandrasegaran K, Rex DK. Small-bowel obstruction: state-of-the-art imaging and its role in clinical management. Clin Gastroenterol Hepatol2008; 6:130–139.
Silva AC, Pimenta M, Guimarães LS. Small bowel obstruction: what to look for. Radiographics2009; 29:423–439.
Finan PJ, Campbell S, Verma R, et al. The management of malignant large bowel obstruction: ACPGBI position statement. Colorectal Dis2007; 9(suppl 4):1–17.
Kohli MD, Maglinte DD. CT enteroclysis in incomplete small bowel obstruction. Abdom Imaging2009; 34:321–327.
Ha HK, Shin BS, Lee SI, et al. Usefulness of CT in patients with intestinal obstruction who have undergone abdominal surgery for malignancy. AJR Am J Roentgenol1998; 171:1587–1593.
DeBernardo R. Surgical management of malignant bowel obstruction: strategies toward palliation of patients with advanced cancer. Curr Oncol Rep2009; 11:287–292.
Ripamonti C, Twycross R, Baines M, et al; Working Group of the European Association for Palliative Care. Clinical-practice recommendations for the management of bowel obstruction in patients with end-stage cancer. Support Care Cancer2001; 9:223–233.
Mangili G, Aletti G, Frigerio L, et al. Palliative care for intestinal obstruction in recurrent ovarian cancer: a multivariate analysis. Int J Gynecol Cancer2005; 15:830–835.
Turner J, Cummin T, Bennett A, Swift G, Green J. Stents and stentability: treatment for malignant bowel obstruction. Br J Hosp Med (Lond)2008; 69:676–680.
Khot UP, Lang AW, Murali K, Parker MC. Systematic review of the efficacy and safety of colorectal stents. Br J Surg2002; 89:1096–1102.
Hosono S, Ohtani H, Arimoto Y, Kanamiya Y. Endoscopic stenting versus surgical gastroenterostomy for palliation of malignant gastroduodenal obstruction: a meta-analysis. J Gastroenterol2007; 42:283–290.
Del Piano M, Ballarè M, Montino F, et al. Endoscopy or surgery for malignant GI outlet obstruction?Gastrointest Endosc2005; 61:421–426.
Tilney HS, Lovegrove RE, Purkayastha S, et al. Comparison of colonic stenting and open surgery for malignant large bowel obstruction. Surg Endosc2007; 21:225–233.
Holt AP, Patel M, Ahmed MM. Palliation of patients with malignant gastroduodenal obstruction with self-expanding metallic stents: the treatment of choice?Gastrointest Endosc2004; 60:1010–1017.
Dastur JK, Forshaw MJ, Modarai B, Solkar MM, Raymond T, Parker MC. Comparison of short-and long-term outcomes following either insertion of self-expanding metallic stents or emergency surgery in malignant large bowel obstruction. Tech Coloproctol2008; 12:51–55.
Turner J, Cummin T, Bennett A, Swift G, Green J. Stents and stentability: treatment for malignant bowel obstruction. Br J Hosp Med (Lond)2008; 69:676–680.
Ripamonti C, Mercadante S, Groff L, Zecca E, De Conno F, Casuccio A. Role of octreotide, scopolamine butylbromide, and hydration in symptom control of patients with inoperable bowel obstruction and nasogastric tubes: a prospective randomized trial. J Pain Symptom Manage2000; 19:23–34.
Davis MP, Walsh D. Treatment of nausea and vomiting in advanced cancer. Support Care Cancer2000; 8:444–452.
Bicanovsky LK, Lagman RL, Davis MP, Walsh D. Managing nonmalignant chronic abdominal pain and malignant bowel obstruction. Gastroenterol Clin North Am2006; 35:131–142.
Glare P, Pereira G, Kristjanson LJ, Stockler M, Tattersall M. Systematic review of the efficacy of antiemetics in the treatment of nausea in patients with far-advanced cancer. Support Care Cancer2004; 12:432–440.
Davis MP, Furste A. Glycopyrrolate: a useful drug in the palliation of mechanical bowel obstruction. J Pain Symptom Manage1999; 18:153–154.
Ripamonti C, Mercadante S. How to use octreotide for malignant bowel obstruction. J Support Oncol2004; 2:357–364.
Shima Y, Ohtsu A, Shirao K, Sasaki Y. Clinical efficacy and safety of octreotide (SMS201-995) in terminally ill Japanese cancer patients with malignant bowel obstruction. Jpn J Clin Oncol2008; 38:354–359.
Mercadante S, Casuccio A, Mangione S. Medical treatment for inoperable malignant bowel obstruction: a qualitative systematic review. J Pain Symptom Manage2007; 33:217–223.
Mystakidou K, Tsilika E, Kalaidopoulou O, Chondros K, Georgaki S, Papadimitriou L. Comparison of octreotide administration vs conservative treatment in the management of inoperable bowel obstruction in patients with far advanced cancer: a randomized, double-blind, controlled clinical trial. Anticancer Res2002; 22:1187–1192.
Mercadante S, Ripamonti C, Casuccio A, Zecca E, Groff L. Comparison of octreotide and hyoscine butylbromide in controlling gastrointestinal symptoms due to malignant inoperable bowel obstruction. Support Care Cancer2000; 8:188–191.
Myers J, Tamber A, Farhadian M. Management of treatment-related intermittent partial small bowel obstruction: the use of octreotide. J Pain Symptom Manage2010; 39:e1–e3.
Srivastava M, Brito-Dellan N, Davis MP, Leach M, Lagman R. Olanzapine as an antiemetic in refractory nausea and vomiting in advanced cancer. J Pain Symptom Manage2003; 25:578–582.
Bentley A, Boyd K. Use of clinical pictures in the management of nausea and vomiting: a prospective audit. Palliat Med2001; 15:247–253.
Pothuri B, Montemarano M, Gerardi M, et al. Percutaneous endoscopic gastrostomy tube placement in patients with malignant bowel obstruction due to ovarian carcinoma. Gynecol Oncol2005; 96:330–334.
Brooksbank MA, Game PA, Ashby MA. Palliative venting gastrostomy in malignant intestinal obstruction. Palliat Med2002; 16:520–526.
Wang MY, Wu MH, Hsieh DY, et al. Home parenteral nutrition support in adults: experience of a medical center in Asia. JPEN J Parenter Enteral Nutr2007; 31:306–310.
Dy SM. Enteral and parenteral nutrition in terminally ill cancer patients: a review of the literature. Am J Hosp Palliat Care2006; 23:369–377.
Whitworth MK, Whitfield A, Holm S, Shaffer J, Makin W, Jayson GC. Doctor, does this mean I’m going to starve to death?J Clin Oncol2004; 22:199–201.
Hoda D, Jatoi A, Burnes J, Loprinzi C, Kelly D. Should patients with advanced, incurable cancers ever be sent home with total parenteral nutrition? A single institution’s 20-year experience. Cancer2005; 103:863–868.
Philip J, Depczynski B. The role of total parenteral nutrition for patients with irreversible bowel obstruction secondary to gynecological malignancy. J Pain Symptom Manage1997; 13:104–111.
August DA, Thorn D, Fisher RL, Welchek CM. Home parenteral nutrition for patients with inoperable malignant bowel obstruction. JPEN J Parenter Enteral Nutr1991; 15:323–327.
Abu-Rustum NR, Barakat RR, Venkatraman E, Spriggs D. Chemotherapy and total parenteral nutrition for advanced ovarian cancer with bowel obstruction. Gynecol Oncol1997; 64:493–495.
Fainsinger RL, Bruera E. When to treat dehydration in a terminally ill patient?Support Care Cancer1997; 5:205–211.
Steiner N, Bruera E. Methods of hydration in palliative care patients. J Palliat Care1998; 14:6–13.
Slomka J. Withholding nutrition at the end of life: clinical and ethical issues. Cleve Clin J Med2003; 70:548–552.
Chiu TY, Hu WY, Chuang RB, Chen CY. Nutrition and hydration for terminal cancer patients in Taiwan. Support Care Cancer2002; 10:630–636.
McCann RM, Hall WJ, Groth-Juncker A. Comfort care for terminally ill patients. The appropriate use of nutrition and hydration. JAMA1994; 272:1263–1266.
Aileen Soriano, MD Harry R. Horvitz Center for Palliative Medicine, Section of Palliative Medicine and Supportive Oncology, Cleveland Clinic
Mellar P. Davis, MD, FCCP, FAAHPM Professor of Medicine, Cleveland Clinic Lerner College of Medicine of Case Western Reserve University, Cleveland, OH; Harry R. Horvitz Center for Palliative Medicine, Section of Palliative Medicine and Supportive Oncology, Department of Solid Tumor Oncology, Taussig Cancer Institute, Cleveland Clinic
Address: Mellar P. Davis, MD, Taussig Cancer Institute, Department of Solid Tumor Oncology, R35, Cleveland Clinic, 9500 Euclid Avenue, Cleveland OH 44195; e-mail davism6@ccf.org
Aileen Soriano, MD Harry R. Horvitz Center for Palliative Medicine, Section of Palliative Medicine and Supportive Oncology, Cleveland Clinic
Mellar P. Davis, MD, FCCP, FAAHPM Professor of Medicine, Cleveland Clinic Lerner College of Medicine of Case Western Reserve University, Cleveland, OH; Harry R. Horvitz Center for Palliative Medicine, Section of Palliative Medicine and Supportive Oncology, Department of Solid Tumor Oncology, Taussig Cancer Institute, Cleveland Clinic
Address: Mellar P. Davis, MD, Taussig Cancer Institute, Department of Solid Tumor Oncology, R35, Cleveland Clinic, 9500 Euclid Avenue, Cleveland OH 44195; e-mail davism6@ccf.org
Author and Disclosure Information
Aileen Soriano, MD Harry R. Horvitz Center for Palliative Medicine, Section of Palliative Medicine and Supportive Oncology, Cleveland Clinic
Mellar P. Davis, MD, FCCP, FAAHPM Professor of Medicine, Cleveland Clinic Lerner College of Medicine of Case Western Reserve University, Cleveland, OH; Harry R. Horvitz Center for Palliative Medicine, Section of Palliative Medicine and Supportive Oncology, Department of Solid Tumor Oncology, Taussig Cancer Institute, Cleveland Clinic
Address: Mellar P. Davis, MD, Taussig Cancer Institute, Department of Solid Tumor Oncology, R35, Cleveland Clinic, 9500 Euclid Avenue, Cleveland OH 44195; e-mail davism6@ccf.org
Malignant bowel obstruction occurs in 5% to 51% of women with ovarian cancer and in 10% to 28% of patients with gastrointestinal cancer, predominantly in the advanced stages.1 Median survival after its onset ranges from 30 to 90 days.2–5
Its symptoms are challenging to manage, since nausea, vomiting, colic, and abdominal pain, which are common, cause significant physical distress and demoralization. The decision whether to correct it with surgery requires an individualized approach and a clear understanding of the goals of care and expected survival in the individual patient.
This review focuses on the management of inoperable malignant bowel obstruction and includes discussion of hydration, nutrition, and endoscopic palliative options.
WHAT ARE THE DIFFERENT TYPES OF OBSTRUCTION?
Bowel obstruction may be mechanical or functional, partial or complete, and may occur at one or at many sites. Tumors can impair bowel function in several ways6–8:
Intraluminal tumors can occlude the lumen or act as a point of intussusception.
Intramural tumors can extend to the mucosa and obstruct the lumen or impair peristalsis.
Mesenteric and omental masses or malignant adhesions can kink or angulate the bowel, creating an extramural obstruction.
Tumors that infiltrate into the mesentery, bowel muscle, or the enteric or celiac plexus can cause dysmotility.
Cholangiocarcinoma, pancreatic carcinoma, and gallbladder carcinoma are the most common tumors causing duodenal obstruction.9 Distal obstruction is caused mainly by colon and ovarian cancer.
Obstruction can be due to treatment
In a minority of patients, obstruction is unrelated to the cancer and is instead due to adhesions arising from surgery, radiation therapy (causing enteritis and strictures), desmoplastic reactions to intraperitoneal chemotherapy, torsion, or internal hernias.10–12
In rare cases, a patient has intestinal pseudo-obstruction from paraneoplastic destruction of enteric neurons, or severe ileus from anticholinergic or sympathomimetic drugs, as seen with acute colonic pseudo-obstruction (Ogilvie syndrome).13
Physiologic reactions to obstruction
Malignant bowel obstruction stimulates gastric, biliary, pancreatic, and intestinal secretions, decreases intraluminal sodium and water reabsorption, and increases mucosal sodium and water secretion.6,14 In response to the obstruction, peristalsis increases, and prostaglandin, vasoactive intestinal peptide, and nociceptive mediators are released. Vasoactive intestinal polypeptide perpetuates a cycle of secretion, distention, and contraction that leads to intestinal hyperemia, bowel edema, and accumulation of fluid in the lumen.8,10,11,15
Signs and symptoms depend on the site
The site of obstruction determines the signs and symptoms patients experience.7,14 Obstructions high in the gastrointestinal tract are associated with greater symptoms but fewer signs than colonic obstructions.1 Patients with proximal small-bowel obstruction have more severe nausea and a greater number of episodes of emesis, but they have relatively normal plain radiographs of the abdomen, which do not have the characteristic air-fluid levels commonly seen with distal small-bowel obstruction.
Most malignant obstructions remain partial, but increasing abdominal distention, worsening nausea, vomiting, abdominal pain, and obstipation over 1 to 2 weeks1 suggest progression to complete obstruction.
IMAGING TESTS FOR MALIGNANT BOWEL OBSTRUCTION
What is the value of plain radiography?
Figure 1. Top, plain radiography shows dilated loops of small bowel (arrows) in the mid-abdominal region of a woman with a small-bowel obstruction from advanced ovarian cancer. Bottom, computed tomography in the same patient shows multiple, dilated loops of small bowel and air-fluid levels (arrows) in the anterior midline of the abdomen.Plain radiography of the abdomen (kidney, ureter, bladder views) has only modest specificity and sensitivity in detecting bowel obstruction (Figure 1). In a patient who has symptoms of obstruction, overreliance on plain radiography can lead to false reassurances that there is no obstruction. The absence of air-fluid levels, dilated loops of bowel, or thickened bowel loops does not exclude malignant bowel obstruction. The overall accuracy of kidney, ureter, bladder radiography is reported to be as low as 50%, with more than 75% of plain films classified as nondiagnostic or not helpful.16,17
Despite these limitations, plain radiography is useful in assessing constipation and its severity as a potential cause of symptoms, and thus it remains an important initial imaging study in almost all patients with suspected malignant bowel obstruction.17,18 It is also used to assess response to treatment.
When do you need contrast radiographs?
Contrast radiography (barium swallow or barium or Gastrograffin enema) is helpful in patients with symptoms of dysmotility from suspected bowel obstruction. It defines the site or sites of obstruction and the extent of the obstruction with a fair degree of accuracy.7,19 Single-contrast studies, if positive, exclude opioid-induced bowel dysfunction or pseudo-obstruction in 83% of patients, with a sensitivity and specificity of 96% and 98%, respectively.8,20 Small-bowel follow-through with barium is more appropriate for low-grade obstructions or for symptomatic patients with a normal kidney, ureter, bladder radiograph.19
However, contrast radiography is limited by the patient’s ability to swallow barium or water-soluble contrast agents, and it can worsen nausea or vomiting.17,18 Also, barium is not absorbed systemically and may interfere with subsequent radiologic studies. Large volumes of contrast agents increase the risk of aspiration pneumonia in patients with poorly controlled nausea and can lead to severe impaction proximal to the obstructed site.8
Is enteroclysis better than barium swallow?
Enteroclysis, ie, injecting radiographic contrast into the bowel via a nasoduodenal tube, has some advantages over the barium swallow technique for detecting partial small-bowel obstruction, since it bypasses the stomach and allows for therapeutic decompression as well as direct visualization of the area of concern.17,18 Enteroclysis radiography objectively gauges severity of intestinal obstruction and bowel wall distensibility, which is an advantage over other imaging studies. Its sensitivity is 100% and specificity 88% in experienced hands.17 Enteroclysis studies also detect nonobstructing intraluminal tumors when computed tomography (CT) is not diagnostic.17,18,21
The drawbacks to enteroclysis are that it is technically difficult to perform and that few radiologists are trained in it.
When is CT useful?
CT is the primary imaging study for patients with obstructive symptoms and a history of abdominal malignancy or a palpable abdominal mass17,20,22,23 (Figure 1). It has a specificity of 100% and a sensitivity of 94%. It plays a major role in decision-making regarding surgery, endoscopy, or palliative interventions,7,19 as it locates the obstruction and differentiates benign from malignant causes with a fair degree of precision.22
CT findings in malignant bowel obstruction may include:
A mass at the site of obstruction or within the original surgical field
Lymphadenopathy
Abrupt transitions in luminal diameter or irregular thickening of the bowel wall at the site or sites of obstruction.7
SURGERY: A DIFFICULT DECISION
Is the patient fit for surgery?
Surgery for malignant bowel obstruction should not be done in patients who have advanced malignancies with bulky intra-abdominal metastases or cancer that has spread outside the abdominal cavity without taking into account treatment options for the cancer, the patient’s nutritional status, and the goals of care.
The role of abdominal surgery (debulking, resection, or bypass) in advanced cancer remains unclear and controversial.24 From 42% to 80% of patients report that symptoms improve after surgery, but recurrent obstruction occurs in 10% to 50%.10 Even in patients with low tumor bulk and good nutritional status, 30-day mortality rates range from 5% to 40%, and complication rates range from 9% to 90%.3,4,6,7,10,14
Outcomes after surgery depend on patient selection criteria perhaps as much as on the surgeon’s experience and skill. Patients with more advanced cancer who have had multiple surgical procedures and those with cancer that does not respond to chemotherapy and radiation present the greatest challenge to surgeons.23
What is the benefit of surgery?
Reports of palliative surgery have included information about 60-day survival rates after the operation, but a number of factors may be more meaningful in this context, such as postoperative symptoms, the patient’s overall wellbeing, how the original symptoms respond to the surgery, complications, and length of hospitalization.14 The paucity of published, validated, patient-related outcome data on which to gauge the value of surgery and the lack of a standard definition of “benefit” further confuse the objective determination of whether these patients benefit from surgery.
In a cohort with advanced ovarian cancer and bowel obstruction, surgery was detrimental to survival and quality of life for all subgroups, and most patients died in the hospital.6
The risk of surgery for malignant bowel obstruction is presumably higher than for abdominal surgery for other indications, since many of the patients are debilitated from their cancer and chemotherapy, and many are malnourished.23 Even when taking into account a potential selection bias in favor of surgery, several studies have reported no significant difference in 30-day mortality rates or median survival between operative and nonoperative groups.2,12 Neither the type of obstruction nor the extent of the surgery influenced outcomes. Surgical outcomes are best in patients with a benign cause of obstruction; little benefit is seen in operating on those with abdominal carcinomatosis.12
Nevertheless, surgery is beneficial in a select few. For patients with a good performance status, slowly progressive cancer, and an expected survival of more than 6 months, surgical bypass or resection is preferred.7,12,25 The challenge is to identify these surgical candidates, taking into account prognostic factors such as nutritional status, tumor burden, performance status, presence of ascites, advanced age, extensive prior chemotherapy or radiotherapy, and diffuse carcinomatosis.3,10,12,20,23
Is surgery consistent with the goals of care?
Crucial to decision-making are the goals of care. Since palliative surgery carries a low level of evidence for benefit in terms of quality of life and survival, time should be set aside to thoroughly review the patient’s medical condition, to explore options, and to clarify expectations and goals of care.3,10 Family members should be invited to be present during these discussions and to be involved in the decision-making process.
WHAT IS THE BENEFIT OF GASTRIC OR COLONIC STENTING?
Endoscopic procedures are alternatives to surgery and offer a palliative option in malignant bowel obstruction. Endoscopic procedures are associated with a shorter hospital stay and quicker recovery than after laparotomy.9,26–30 In certain situations, stenting serves as a bridge to surgery, allowing time to mitigate comorbid conditions, to enhance nutrition, and to complete staging, while relieving symptoms.27–29,31,32 Definitive surgery can be done as a single-stage procedure without a diverting enterostomy.
Self-expanding metal stents for gastric outlet, small-bowel, and colonic obstructions are an option in patients who have incurable metastatic disease who are unfit for surgery, in patients with a single point of obstruction or locally extensive disease, or in patients who do not want to undergo laparotomy.28–30
Technical and clinical success rates for colorectal stenting are high (88% to 93%).26,27 Stenting is more successful for left-sided colonic obstructions than for proximal colonic obstructions. Even for patients with extracolonic malignancies such as ovarian cancer, the technical success rate of colorectal stenting is 87%.26 However, patients with unrecognized peritoneal carcinomatosis or multifocal bowel obstruction are less likely to have symptomatic relief even after successful stenting.6,9
Contraindications to stenting
Absolute contraindications to stenting are colonic or tumor perforation with peritonitis. A relative contraindication is a rectal tumor within 2 cm of the anal margin. Stenting in this circumstance leads to tenesmus and incontinence.33
Complications of stenting
Death rates during colorectal stent insertion are less than 1%. The hospital stay and incidence of complications are significantly less than with surgery.26,30
Stent migration occurs in 10% of cases and is asymptomatic, but half of patients with this complication require a repeat intervention. The risk of migration is greater if chemotherapy or radiation therapy succeeds in shrinking the tumor.
Bleeding occurs in 5% of cases, usually from the underlying tumor.
Perforation occurs in 4%, but the rate increases to 10% with the use of dilatation before stent placement.
The rate of recurrent obstruction from tumor ingrowth, overgrowth, or fecal impaction is 10%.9,26,29 Recurrent obstruction may be treated with additional stents inserted within the original stent.9
GASTRIC OUTLET OBSTRUCTION: SURGERY VS STENTING
Gastrojejunostomy has in the past been the treatment of choice for gastric outlet obstruction. Certainly, patients with slow-growing tumors and an expected survival of greater than 60 days may be considered for this bypass procedure; those with a short tumor length, a single site of obstruction (preferably in the pylorus or proximal duodenum), a good performance status, and a life expectancy greater than 30 days are good candidates.7 Nevertheless, for patients with advanced cancer and poor performance status, gastroenterostomy carries a significant risk of morbidity and death.28
Endoscopic stenting of gastric outlet obstruction has a greater success rate, a shorter time to oral intake, a lower morbidity rate, a lower incidence of delayed gastric emptying, and a shorter hospital stay compared with gastroenterostomy.28,29 Technical success rates of stenting are 90%, and 75% of patients have resolution of nausea and vomiting.7 Stenting is generally not possible if the obstruction occurs beyond the ligament of Treitz.
Patients who are expected to survive less than 1 month or who have rapidly progressive disease, overt ascites, carcinomatosis, or multiple sites of obstruction should be managed with percutaneous, endoscopically placed gastrostomy tubes.7
Late complications of stenting for gastric outlet obstruction are occlusion with food or ingrowth of tumor through or around the wire mesh.7 This may require laser therapy or placement of a second stent, or both.
DRUG THERAPY
Medical therapy can palliate symptoms of malignant bowel obstruction for most patients.34 Recommendations have been published by the Working Group of the European Association for Palliative Care.24 Symptom management is focused on pain, nausea, and vomiting.
Which drugs can I use for abdominal pain?
Patients experience two types of abdominal pain: continuous and colic. Each type of pain requires different treatment approaches and classes of drugs.
Potent opioids such as morphine, hydromorphone (Dilaudid), and fentanyl (Fentora) are used to relieve continuous abdominal pain.7 The dose is titrated for adequate relief. Subcutaneous, intravenous, sublingual, and transdermal routes can be used if nausea and vomiting prevent oral administration.
However, opioids can aggravate colic by stimulating circular smooth muscle, leading to segmental contractions. Opioid-sparing adjuvant drugs such as ketorolac (Toradol) may improve colic and continuous pain and prevent a partial obstruction from becoming a complete obstruction by sparing opioid doses.35
Colic may persist or worsen with the use of opioids. Drugs that reduce colic include the scopolamine drugs hyoscine butylbromide and hyoscine hydrobromide, glycopyrrolate (Robinul), and octreotide (Sandostatin).7,34–37
Which drugs are appropriate for reducing nausea and vomiting?
Phenothiazines reduce nausea and control vomiting. Chlorpromazine (Thorazine), prochlorperazine (Compro, Compazine), and promethazine (Phenergan) have all been reported to treat nausea successfully.35,37
Haloperidol (Haldol), a butyrophenone selective dopamine D2-receptor antagonist, has negligible anticholinergic activity. At low doses it produces less sedation than phenothiazines and is an ideal agent for patients with nausea and delirium.35 Doses range from 5 to 15 mg/day, given in divided doses or as intermittent or continuous intravenous infusions.
Anticholinergics, with or without somatostatin analogues, reduce gastrointestinal secretions, fluid accumulation, and vomiting. Anticholinergics bind to muscarinic receptors on enteric neurons in the myenteric and the submucosal plexus. Dosages:
Hyoscine butylbromide 40 to 120 mg/day.
Hyoscine hydrobromide 0.2 to 0.9 mg/day.7,34
Glycopyrrolate, a quaternary ammonium anticholinergic, has minimal central nervous system penetration and is less likely to cause delirium or cardiac side effects compared with tertiary amine anticholinergics such as atropine and scopolamine.38 The recommended dose is 0.1 to 0.2 mg subcutaneously or intravenously three to four times daily.
Octreotide, an analogue of somatostatin, blocks the release of vasoactive intestinal polypeptide, which is increased in malignant bowel obstruction.14,15 It reduces the excretion of water, sodium, and chloride into the bowel lumen and increases the absorption of electrolytes and water. It also inhibits pancreatic enzyme secretion and splanchnic blood flow. The result of all these effects is reduced luminal content, reduced motility, reduced vascular congestion of the bowel wall, and, in certain circumstances, reduced ascites.39
In small randomized trials, octreotide was more successful than anticholinergics at improving nausea, vomiting, and colic in patients requiring a nasogastric tube and in those whose symptoms were refractory to standard medical treatment.5,34,40–43 A recent case report found octreotide helpful in resolving symptoms of partial bowel obstruction that were unresponsive to standard measures.44
Octreotide is well tolerated and reduces the time patients require a nasogastric tube without significantly worsening xerostomia. High cost limits its use in American hospice care due to the Medicare capitated system of reimbursement for drugs and services, and as a result it is a second-tier drug despite evidence of its efficacy.
Octreotide doses are 100 to 200 mg every 8 hours.
Metoclopramide (Reglan), a dopaminergic antagonist, a 5HT4 receptor agonist, and a 5HT3 receptor blocker at doses greater than 120 mg/day, combines the action of a phenothiazine, which blocks D2 receptors in the central chemoreceptor trigger zone, with promotility actions through serotonin receptors (5HT4).35,37
Metoclopramide should not be used with anticholinergics or in patients with colic or complete obstruction.35,45 In some centers it is the first-line drug for functional or partial bowel obstruction.7 Dosages range from 40 to 240 mg/day.
Olanzapine (Zyprexa), an atypical antipsychotic, blocks multiple neurotransmitter receptors (D2, H1, Ach, 5HT3) responsible for initiating emesis. It is an option in patients whose nausea and vomiting fail to respond to standard antiemetics.46 Dosages range from 2.5 to 20 mg/day.
Dissolvable tablets are given sublingually, which makes olanzapine a versatile antiemetic in cases of intractable nausea. Our unpublished experience is that the sublingual route reduces nausea associated with malignant bowel obstruction and obviates the need for subcutaneous injections or intravenous antiemetic infusions.
Corticosteroids. Although how corticosteroids relieve malignant bowel obstruction is unknown, they are presumed to act centrally.37,45 In addition, they reduce peritumoral edema and luminal salt and water, and they also have antiemetic and analgesic properties.
Evidence from a meta-analysis found that 6 to 16 mg of parenteral dexamethasone per day reduced symptoms and improved bowel function in 60% of patients but did not change the prognosis.11
A trial of 4 or 5 days is adequate to determine response. If there is no response, the corticosteroid should be rapidly tapered. Side effects are minimal when corticosteroids are used short-term.
Combination therapy. Only rarely does a single drug resolve symptoms of malignant bowel obstruction. Antiemetics, analgesics, corticosteroids, antisecretory anticholinergics, and octreotide are often required in combination to achieve acceptable symptom relief.3,5,7,47
In a small prospective case series, the combination of metoclopramide 60 mg/day, octreotide 0.3 mg/day, and dexamethasone 12 mg/day with a single bolus of amidotrizoic acid (a contrast agent) improved intestinal transit within 1 to 5 days and resolved vomiting within 24 hours.45
Compatibility and the route of administration of medications are key considerations when choosing drug combinations.
WHEN TO CONSIDER A VENTING GASTROSTOMY
Patients with a poor performance status, rapidly progressive disease, peritoneal carcinomatosis, a life expectancy of less than 30 days, or multiple levels of obstruction benefit from placement of a percutaneous endoscopic gastrostomy tube (ie, a venting gastrostomy) rather than surgery if symptoms do not respond to drug therapy.7,48 There is compelling evidence that this procedure relieves nausea and vomiting in 80% to 90% of patients and restores some level of oral intake in many.5,6,48,49 A venting gastrostomy tube can be placed during surgical exploration, percutaneously with fluoroscopy, or endoscopically.9
There are no absolute contraindications to gastrostomy tube placement. It is feasible even in patients with tumors encasing the stomach, diffuse carcinomatosis, and ascites.48 However, massive ascites, previous upper abdominal surgery, or a large mass attached to the abdominal wall make tube placement difficult.
Complications are often local. Patients experience transient abdominal wall pain after the procedure. Dislodgement, bleeding, catheter migration, peritonitis, and necrotizing fasciitis are early complications. Others include skin excoriation from leakage of gastric contents, leakage of ascitic fluid from the site, and obstruction or dislodgement of the tube.48,49
Patients can be discharged from the hospital soon after the tube is placed, usually with fewer medications than for patients who undergo surgery.48 This is particularly important for patients with a short expected survival. Some patients at home benefit from hydration (less than 2 L/day) via an existing central venous port or peripherally inserted central catheter, or by hypodermoclysis.
WHEN IS A NASOGASTRIC TUBE APPROPRIATE?
Some patients with malignant bowel obstruction require a nasogastric tube early in their hospital course.12 Unfortunately, nasogastric tubes, if left in place, cause nose and throat pain, sinusitis, abscess formation, erosion of nasal cartilage, aspiration, esophageal erosion, pharyngitis, and social isolation.5,6
Nasogastric tubes should be a temporizing measure to vent gastrointestinal secretions, reduce abdominal distension, and improve nausea and vomiting while a decision about surgery is being made.13,24 If surgery is not feasible, one can avoid the long-term complications and discomfort of a nasogastric tube via medical management and earlier evaluation for venting gastrostomy in those with symptoms that respond poorly to optimal medical management.49
WHICH PATIENTS BENEFIT FROM TOTAL PARENTERAL NUTRITION?
The use of total parenteral nutrition in patients with incurable malignancies is controversial. Enteral and parenteral feeding can increase muscle mass and improve functional status and quality of life in a subset of patients who are not suffering from cancer-related cachexia.2,50,51 However, for those whose weight loss and malnutrition are consequences of tumor-mediated cachexia, as demonstrated by anorexia and an elevated C-reactive protein level, parenteral nutrition is unlikely to improve the outcome.51 For most terminally ill patients, retrospective studies have failed to show that parenteral nutrition improves overall survival, performance status, or quality of life.2,48,50–54
Total parenteral nutrition poses risks: it is invasive and requires central venous access, which predisposes to infection; it requires frequent monitoring of hydration and electrolytes; and it predisposes to thrombosis, diarrhea, hyperglycemia, and liver failure.50–56
Total parenteral nutrition may be justified in patients with minimal tumor burden who are candidates for definitive surgery, or in those with a good performance status early in the disease course who have not had chemotherapy or whose cancer responds to chemotherapy.2,50–56
The American College of Physicians discourages the routine use of parenteral nutrition in those with advanced cancer who are undergoing palliative chemotherapy, since few patients benefit and many experience side effects.53
Total parenteral nutrition is much like a medical intervention in that it should be offered or continued only if it provides benefit. Conversations at the time that it is begun must include adverse effects that will lead to its discontinuation, and criteria for response. In certain situations, a limited trial of parenteral nutrition may be considered for patients with an uncertain prognosis or for those who have potentially reversible conditions that limit oral intake.51 In such cases, there should be a clear understanding between patient and physician that parenteral nutrition will be discontinued if it fails to show benefit.53
ADDITIONAL CONCERNS OF PATIENTS AND FAMILIES
‘Will I starve to death?’
Starvation is a fear echoed by patients and families. Ethical discourse on the continuation of nutrition and hydration for the terminally ill has been polarizing.57–60 Withdrawal of nutrition can be perceived as euthanasia.
Advanced cancer patients in general do not experience hunger, and those who do require only small amounts of food for satiation.61 In one report, most patients died of their advanced cancer and not from starvation.52 Artificial hydration and nutrition will thus not influence survival and can even be a burden without benefit in the imminently dying.60 These patients should be encouraged to take food orally for pleasure, as long as it is tolerated, without consideration of end points such as weight gain, body mass index, or albumin levels.
Complaints of thirst and dryness of the mouth are relieved with mouth care, ice chips, lubrication to the lips, and sips of fluid, rather than by parenteral nutrition.59 Patients with a terminal illness experience relief from thirst with minimal intake. The symptom of thirst may be relieved without hydration.34,61 Adequate hydration requires smaller fluid volumes due to decreased body weight, decreased renal clearance of free water, and decreased insensible water losses from reduced physical activity.58
‘Can we continue intravenous hydration so he won’t die of thirst?’
Overzealous intravenous hydration may worsen the symptoms of malignant bowel obstruction. Overhydration can increase secretions in the gut lumen and worsen the secretion-distention-contraction cycle, leading to greater abdominal pain and to nausea and vomiting.7 There is a greater risk of fluid overload in these patients, since they have edema and excessive interstitial fluid. Most have a low serum albumin level, which results in movement of fluid from intravascular to interstitial spaces due to reduced colloid osmotic pressure. In these instances, overzealous hydration can lead to respiratory insufficiency and worsening edema.
In spite of numerous discussions in the medical literature of the benefits and burdens of continual hydration, there is no consensus or guideline. When a patient has limited oral intake, the decision to hydrate should be individualized, with careful assessment of the risks and benefits and in accordance with the patient’s or family’s wishes.57,58
Is treatment at home feasible?
Discharging patients with inoperable malignant bowel obstruction requires careful planning. Patients and family members need to be educated on the use of around-the-clock medications and symptom-targeted, as-needed drugs. Days before discharge, questions about diet need to be clarified. Education about total parenteral nutrition and gastrostomy tube care should be completed before discharge from the hospital.
Drug management should be simplified, or compatible medications should be combined into a single infusion. For example, morphine, glycopyrrolate, and haloperidol or metoclopramide are chemically compatible in standard intravenous solutions and can be combined.
Families feel less anxious about the foreseen and the possible unforeseen course of the illness if they can talk with hospice workers early on. This early involvement also facilitates the transition to home hospice care.
SUMMARY OF IMPORTANT POINTS
Patients with malignant bowel obstruction need a highly individualized approach, tailored to their medical condition, the prognosis, and the goals of care.
Surgery should not be routinely undertaken; less-invasive approaches such as gastric or colonic stenting should be considered first.
Combinations of analgesics, antisecretory drugs, and antiemetics can provide acceptable symptom relief in the inoperable patient.
A venting gastrostomy should be considered if drug therapy fails to reduce nausea and vomiting to an acceptable level.
A nasogastric tube should be used only as a temporizing measure, until symptoms are controlled medically or a venting gastrostomy is placed.
Total parenteral nutrition is of benefit only in patients with intermediate life expectancy who may otherwise die of starvation rather than from the cancer itself.
Malignant bowel obstruction occurs in 5% to 51% of women with ovarian cancer and in 10% to 28% of patients with gastrointestinal cancer, predominantly in the advanced stages.1 Median survival after its onset ranges from 30 to 90 days.2–5
Its symptoms are challenging to manage, since nausea, vomiting, colic, and abdominal pain, which are common, cause significant physical distress and demoralization. The decision whether to correct it with surgery requires an individualized approach and a clear understanding of the goals of care and expected survival in the individual patient.
This review focuses on the management of inoperable malignant bowel obstruction and includes discussion of hydration, nutrition, and endoscopic palliative options.
WHAT ARE THE DIFFERENT TYPES OF OBSTRUCTION?
Bowel obstruction may be mechanical or functional, partial or complete, and may occur at one or at many sites. Tumors can impair bowel function in several ways6–8:
Intraluminal tumors can occlude the lumen or act as a point of intussusception.
Intramural tumors can extend to the mucosa and obstruct the lumen or impair peristalsis.
Mesenteric and omental masses or malignant adhesions can kink or angulate the bowel, creating an extramural obstruction.
Tumors that infiltrate into the mesentery, bowel muscle, or the enteric or celiac plexus can cause dysmotility.
Cholangiocarcinoma, pancreatic carcinoma, and gallbladder carcinoma are the most common tumors causing duodenal obstruction.9 Distal obstruction is caused mainly by colon and ovarian cancer.
Obstruction can be due to treatment
In a minority of patients, obstruction is unrelated to the cancer and is instead due to adhesions arising from surgery, radiation therapy (causing enteritis and strictures), desmoplastic reactions to intraperitoneal chemotherapy, torsion, or internal hernias.10–12
In rare cases, a patient has intestinal pseudo-obstruction from paraneoplastic destruction of enteric neurons, or severe ileus from anticholinergic or sympathomimetic drugs, as seen with acute colonic pseudo-obstruction (Ogilvie syndrome).13
Physiologic reactions to obstruction
Malignant bowel obstruction stimulates gastric, biliary, pancreatic, and intestinal secretions, decreases intraluminal sodium and water reabsorption, and increases mucosal sodium and water secretion.6,14 In response to the obstruction, peristalsis increases, and prostaglandin, vasoactive intestinal peptide, and nociceptive mediators are released. Vasoactive intestinal polypeptide perpetuates a cycle of secretion, distention, and contraction that leads to intestinal hyperemia, bowel edema, and accumulation of fluid in the lumen.8,10,11,15
Signs and symptoms depend on the site
The site of obstruction determines the signs and symptoms patients experience.7,14 Obstructions high in the gastrointestinal tract are associated with greater symptoms but fewer signs than colonic obstructions.1 Patients with proximal small-bowel obstruction have more severe nausea and a greater number of episodes of emesis, but they have relatively normal plain radiographs of the abdomen, which do not have the characteristic air-fluid levels commonly seen with distal small-bowel obstruction.
Most malignant obstructions remain partial, but increasing abdominal distention, worsening nausea, vomiting, abdominal pain, and obstipation over 1 to 2 weeks1 suggest progression to complete obstruction.
IMAGING TESTS FOR MALIGNANT BOWEL OBSTRUCTION
What is the value of plain radiography?
Figure 1. Top, plain radiography shows dilated loops of small bowel (arrows) in the mid-abdominal region of a woman with a small-bowel obstruction from advanced ovarian cancer. Bottom, computed tomography in the same patient shows multiple, dilated loops of small bowel and air-fluid levels (arrows) in the anterior midline of the abdomen.Plain radiography of the abdomen (kidney, ureter, bladder views) has only modest specificity and sensitivity in detecting bowel obstruction (Figure 1). In a patient who has symptoms of obstruction, overreliance on plain radiography can lead to false reassurances that there is no obstruction. The absence of air-fluid levels, dilated loops of bowel, or thickened bowel loops does not exclude malignant bowel obstruction. The overall accuracy of kidney, ureter, bladder radiography is reported to be as low as 50%, with more than 75% of plain films classified as nondiagnostic or not helpful.16,17
Despite these limitations, plain radiography is useful in assessing constipation and its severity as a potential cause of symptoms, and thus it remains an important initial imaging study in almost all patients with suspected malignant bowel obstruction.17,18 It is also used to assess response to treatment.
When do you need contrast radiographs?
Contrast radiography (barium swallow or barium or Gastrograffin enema) is helpful in patients with symptoms of dysmotility from suspected bowel obstruction. It defines the site or sites of obstruction and the extent of the obstruction with a fair degree of accuracy.7,19 Single-contrast studies, if positive, exclude opioid-induced bowel dysfunction or pseudo-obstruction in 83% of patients, with a sensitivity and specificity of 96% and 98%, respectively.8,20 Small-bowel follow-through with barium is more appropriate for low-grade obstructions or for symptomatic patients with a normal kidney, ureter, bladder radiograph.19
However, contrast radiography is limited by the patient’s ability to swallow barium or water-soluble contrast agents, and it can worsen nausea or vomiting.17,18 Also, barium is not absorbed systemically and may interfere with subsequent radiologic studies. Large volumes of contrast agents increase the risk of aspiration pneumonia in patients with poorly controlled nausea and can lead to severe impaction proximal to the obstructed site.8
Is enteroclysis better than barium swallow?
Enteroclysis, ie, injecting radiographic contrast into the bowel via a nasoduodenal tube, has some advantages over the barium swallow technique for detecting partial small-bowel obstruction, since it bypasses the stomach and allows for therapeutic decompression as well as direct visualization of the area of concern.17,18 Enteroclysis radiography objectively gauges severity of intestinal obstruction and bowel wall distensibility, which is an advantage over other imaging studies. Its sensitivity is 100% and specificity 88% in experienced hands.17 Enteroclysis studies also detect nonobstructing intraluminal tumors when computed tomography (CT) is not diagnostic.17,18,21
The drawbacks to enteroclysis are that it is technically difficult to perform and that few radiologists are trained in it.
When is CT useful?
CT is the primary imaging study for patients with obstructive symptoms and a history of abdominal malignancy or a palpable abdominal mass17,20,22,23 (Figure 1). It has a specificity of 100% and a sensitivity of 94%. It plays a major role in decision-making regarding surgery, endoscopy, or palliative interventions,7,19 as it locates the obstruction and differentiates benign from malignant causes with a fair degree of precision.22
CT findings in malignant bowel obstruction may include:
A mass at the site of obstruction or within the original surgical field
Lymphadenopathy
Abrupt transitions in luminal diameter or irregular thickening of the bowel wall at the site or sites of obstruction.7
SURGERY: A DIFFICULT DECISION
Is the patient fit for surgery?
Surgery for malignant bowel obstruction should not be done in patients who have advanced malignancies with bulky intra-abdominal metastases or cancer that has spread outside the abdominal cavity without taking into account treatment options for the cancer, the patient’s nutritional status, and the goals of care.
The role of abdominal surgery (debulking, resection, or bypass) in advanced cancer remains unclear and controversial.24 From 42% to 80% of patients report that symptoms improve after surgery, but recurrent obstruction occurs in 10% to 50%.10 Even in patients with low tumor bulk and good nutritional status, 30-day mortality rates range from 5% to 40%, and complication rates range from 9% to 90%.3,4,6,7,10,14
Outcomes after surgery depend on patient selection criteria perhaps as much as on the surgeon’s experience and skill. Patients with more advanced cancer who have had multiple surgical procedures and those with cancer that does not respond to chemotherapy and radiation present the greatest challenge to surgeons.23
What is the benefit of surgery?
Reports of palliative surgery have included information about 60-day survival rates after the operation, but a number of factors may be more meaningful in this context, such as postoperative symptoms, the patient’s overall wellbeing, how the original symptoms respond to the surgery, complications, and length of hospitalization.14 The paucity of published, validated, patient-related outcome data on which to gauge the value of surgery and the lack of a standard definition of “benefit” further confuse the objective determination of whether these patients benefit from surgery.
In a cohort with advanced ovarian cancer and bowel obstruction, surgery was detrimental to survival and quality of life for all subgroups, and most patients died in the hospital.6
The risk of surgery for malignant bowel obstruction is presumably higher than for abdominal surgery for other indications, since many of the patients are debilitated from their cancer and chemotherapy, and many are malnourished.23 Even when taking into account a potential selection bias in favor of surgery, several studies have reported no significant difference in 30-day mortality rates or median survival between operative and nonoperative groups.2,12 Neither the type of obstruction nor the extent of the surgery influenced outcomes. Surgical outcomes are best in patients with a benign cause of obstruction; little benefit is seen in operating on those with abdominal carcinomatosis.12
Nevertheless, surgery is beneficial in a select few. For patients with a good performance status, slowly progressive cancer, and an expected survival of more than 6 months, surgical bypass or resection is preferred.7,12,25 The challenge is to identify these surgical candidates, taking into account prognostic factors such as nutritional status, tumor burden, performance status, presence of ascites, advanced age, extensive prior chemotherapy or radiotherapy, and diffuse carcinomatosis.3,10,12,20,23
Is surgery consistent with the goals of care?
Crucial to decision-making are the goals of care. Since palliative surgery carries a low level of evidence for benefit in terms of quality of life and survival, time should be set aside to thoroughly review the patient’s medical condition, to explore options, and to clarify expectations and goals of care.3,10 Family members should be invited to be present during these discussions and to be involved in the decision-making process.
WHAT IS THE BENEFIT OF GASTRIC OR COLONIC STENTING?
Endoscopic procedures are alternatives to surgery and offer a palliative option in malignant bowel obstruction. Endoscopic procedures are associated with a shorter hospital stay and quicker recovery than after laparotomy.9,26–30 In certain situations, stenting serves as a bridge to surgery, allowing time to mitigate comorbid conditions, to enhance nutrition, and to complete staging, while relieving symptoms.27–29,31,32 Definitive surgery can be done as a single-stage procedure without a diverting enterostomy.
Self-expanding metal stents for gastric outlet, small-bowel, and colonic obstructions are an option in patients who have incurable metastatic disease who are unfit for surgery, in patients with a single point of obstruction or locally extensive disease, or in patients who do not want to undergo laparotomy.28–30
Technical and clinical success rates for colorectal stenting are high (88% to 93%).26,27 Stenting is more successful for left-sided colonic obstructions than for proximal colonic obstructions. Even for patients with extracolonic malignancies such as ovarian cancer, the technical success rate of colorectal stenting is 87%.26 However, patients with unrecognized peritoneal carcinomatosis or multifocal bowel obstruction are less likely to have symptomatic relief even after successful stenting.6,9
Contraindications to stenting
Absolute contraindications to stenting are colonic or tumor perforation with peritonitis. A relative contraindication is a rectal tumor within 2 cm of the anal margin. Stenting in this circumstance leads to tenesmus and incontinence.33
Complications of stenting
Death rates during colorectal stent insertion are less than 1%. The hospital stay and incidence of complications are significantly less than with surgery.26,30
Stent migration occurs in 10% of cases and is asymptomatic, but half of patients with this complication require a repeat intervention. The risk of migration is greater if chemotherapy or radiation therapy succeeds in shrinking the tumor.
Bleeding occurs in 5% of cases, usually from the underlying tumor.
Perforation occurs in 4%, but the rate increases to 10% with the use of dilatation before stent placement.
The rate of recurrent obstruction from tumor ingrowth, overgrowth, or fecal impaction is 10%.9,26,29 Recurrent obstruction may be treated with additional stents inserted within the original stent.9
GASTRIC OUTLET OBSTRUCTION: SURGERY VS STENTING
Gastrojejunostomy has in the past been the treatment of choice for gastric outlet obstruction. Certainly, patients with slow-growing tumors and an expected survival of greater than 60 days may be considered for this bypass procedure; those with a short tumor length, a single site of obstruction (preferably in the pylorus or proximal duodenum), a good performance status, and a life expectancy greater than 30 days are good candidates.7 Nevertheless, for patients with advanced cancer and poor performance status, gastroenterostomy carries a significant risk of morbidity and death.28
Endoscopic stenting of gastric outlet obstruction has a greater success rate, a shorter time to oral intake, a lower morbidity rate, a lower incidence of delayed gastric emptying, and a shorter hospital stay compared with gastroenterostomy.28,29 Technical success rates of stenting are 90%, and 75% of patients have resolution of nausea and vomiting.7 Stenting is generally not possible if the obstruction occurs beyond the ligament of Treitz.
Patients who are expected to survive less than 1 month or who have rapidly progressive disease, overt ascites, carcinomatosis, or multiple sites of obstruction should be managed with percutaneous, endoscopically placed gastrostomy tubes.7
Late complications of stenting for gastric outlet obstruction are occlusion with food or ingrowth of tumor through or around the wire mesh.7 This may require laser therapy or placement of a second stent, or both.
DRUG THERAPY
Medical therapy can palliate symptoms of malignant bowel obstruction for most patients.34 Recommendations have been published by the Working Group of the European Association for Palliative Care.24 Symptom management is focused on pain, nausea, and vomiting.
Which drugs can I use for abdominal pain?
Patients experience two types of abdominal pain: continuous and colic. Each type of pain requires different treatment approaches and classes of drugs.
Potent opioids such as morphine, hydromorphone (Dilaudid), and fentanyl (Fentora) are used to relieve continuous abdominal pain.7 The dose is titrated for adequate relief. Subcutaneous, intravenous, sublingual, and transdermal routes can be used if nausea and vomiting prevent oral administration.
However, opioids can aggravate colic by stimulating circular smooth muscle, leading to segmental contractions. Opioid-sparing adjuvant drugs such as ketorolac (Toradol) may improve colic and continuous pain and prevent a partial obstruction from becoming a complete obstruction by sparing opioid doses.35
Colic may persist or worsen with the use of opioids. Drugs that reduce colic include the scopolamine drugs hyoscine butylbromide and hyoscine hydrobromide, glycopyrrolate (Robinul), and octreotide (Sandostatin).7,34–37
Which drugs are appropriate for reducing nausea and vomiting?
Phenothiazines reduce nausea and control vomiting. Chlorpromazine (Thorazine), prochlorperazine (Compro, Compazine), and promethazine (Phenergan) have all been reported to treat nausea successfully.35,37
Haloperidol (Haldol), a butyrophenone selective dopamine D2-receptor antagonist, has negligible anticholinergic activity. At low doses it produces less sedation than phenothiazines and is an ideal agent for patients with nausea and delirium.35 Doses range from 5 to 15 mg/day, given in divided doses or as intermittent or continuous intravenous infusions.
Anticholinergics, with or without somatostatin analogues, reduce gastrointestinal secretions, fluid accumulation, and vomiting. Anticholinergics bind to muscarinic receptors on enteric neurons in the myenteric and the submucosal plexus. Dosages:
Hyoscine butylbromide 40 to 120 mg/day.
Hyoscine hydrobromide 0.2 to 0.9 mg/day.7,34
Glycopyrrolate, a quaternary ammonium anticholinergic, has minimal central nervous system penetration and is less likely to cause delirium or cardiac side effects compared with tertiary amine anticholinergics such as atropine and scopolamine.38 The recommended dose is 0.1 to 0.2 mg subcutaneously or intravenously three to four times daily.
Octreotide, an analogue of somatostatin, blocks the release of vasoactive intestinal polypeptide, which is increased in malignant bowel obstruction.14,15 It reduces the excretion of water, sodium, and chloride into the bowel lumen and increases the absorption of electrolytes and water. It also inhibits pancreatic enzyme secretion and splanchnic blood flow. The result of all these effects is reduced luminal content, reduced motility, reduced vascular congestion of the bowel wall, and, in certain circumstances, reduced ascites.39
In small randomized trials, octreotide was more successful than anticholinergics at improving nausea, vomiting, and colic in patients requiring a nasogastric tube and in those whose symptoms were refractory to standard medical treatment.5,34,40–43 A recent case report found octreotide helpful in resolving symptoms of partial bowel obstruction that were unresponsive to standard measures.44
Octreotide is well tolerated and reduces the time patients require a nasogastric tube without significantly worsening xerostomia. High cost limits its use in American hospice care due to the Medicare capitated system of reimbursement for drugs and services, and as a result it is a second-tier drug despite evidence of its efficacy.
Octreotide doses are 100 to 200 mg every 8 hours.
Metoclopramide (Reglan), a dopaminergic antagonist, a 5HT4 receptor agonist, and a 5HT3 receptor blocker at doses greater than 120 mg/day, combines the action of a phenothiazine, which blocks D2 receptors in the central chemoreceptor trigger zone, with promotility actions through serotonin receptors (5HT4).35,37
Metoclopramide should not be used with anticholinergics or in patients with colic or complete obstruction.35,45 In some centers it is the first-line drug for functional or partial bowel obstruction.7 Dosages range from 40 to 240 mg/day.
Olanzapine (Zyprexa), an atypical antipsychotic, blocks multiple neurotransmitter receptors (D2, H1, Ach, 5HT3) responsible for initiating emesis. It is an option in patients whose nausea and vomiting fail to respond to standard antiemetics.46 Dosages range from 2.5 to 20 mg/day.
Dissolvable tablets are given sublingually, which makes olanzapine a versatile antiemetic in cases of intractable nausea. Our unpublished experience is that the sublingual route reduces nausea associated with malignant bowel obstruction and obviates the need for subcutaneous injections or intravenous antiemetic infusions.
Corticosteroids. Although how corticosteroids relieve malignant bowel obstruction is unknown, they are presumed to act centrally.37,45 In addition, they reduce peritumoral edema and luminal salt and water, and they also have antiemetic and analgesic properties.
Evidence from a meta-analysis found that 6 to 16 mg of parenteral dexamethasone per day reduced symptoms and improved bowel function in 60% of patients but did not change the prognosis.11
A trial of 4 or 5 days is adequate to determine response. If there is no response, the corticosteroid should be rapidly tapered. Side effects are minimal when corticosteroids are used short-term.
Combination therapy. Only rarely does a single drug resolve symptoms of malignant bowel obstruction. Antiemetics, analgesics, corticosteroids, antisecretory anticholinergics, and octreotide are often required in combination to achieve acceptable symptom relief.3,5,7,47
In a small prospective case series, the combination of metoclopramide 60 mg/day, octreotide 0.3 mg/day, and dexamethasone 12 mg/day with a single bolus of amidotrizoic acid (a contrast agent) improved intestinal transit within 1 to 5 days and resolved vomiting within 24 hours.45
Compatibility and the route of administration of medications are key considerations when choosing drug combinations.
WHEN TO CONSIDER A VENTING GASTROSTOMY
Patients with a poor performance status, rapidly progressive disease, peritoneal carcinomatosis, a life expectancy of less than 30 days, or multiple levels of obstruction benefit from placement of a percutaneous endoscopic gastrostomy tube (ie, a venting gastrostomy) rather than surgery if symptoms do not respond to drug therapy.7,48 There is compelling evidence that this procedure relieves nausea and vomiting in 80% to 90% of patients and restores some level of oral intake in many.5,6,48,49 A venting gastrostomy tube can be placed during surgical exploration, percutaneously with fluoroscopy, or endoscopically.9
There are no absolute contraindications to gastrostomy tube placement. It is feasible even in patients with tumors encasing the stomach, diffuse carcinomatosis, and ascites.48 However, massive ascites, previous upper abdominal surgery, or a large mass attached to the abdominal wall make tube placement difficult.
Complications are often local. Patients experience transient abdominal wall pain after the procedure. Dislodgement, bleeding, catheter migration, peritonitis, and necrotizing fasciitis are early complications. Others include skin excoriation from leakage of gastric contents, leakage of ascitic fluid from the site, and obstruction or dislodgement of the tube.48,49
Patients can be discharged from the hospital soon after the tube is placed, usually with fewer medications than for patients who undergo surgery.48 This is particularly important for patients with a short expected survival. Some patients at home benefit from hydration (less than 2 L/day) via an existing central venous port or peripherally inserted central catheter, or by hypodermoclysis.
WHEN IS A NASOGASTRIC TUBE APPROPRIATE?
Some patients with malignant bowel obstruction require a nasogastric tube early in their hospital course.12 Unfortunately, nasogastric tubes, if left in place, cause nose and throat pain, sinusitis, abscess formation, erosion of nasal cartilage, aspiration, esophageal erosion, pharyngitis, and social isolation.5,6
Nasogastric tubes should be a temporizing measure to vent gastrointestinal secretions, reduce abdominal distension, and improve nausea and vomiting while a decision about surgery is being made.13,24 If surgery is not feasible, one can avoid the long-term complications and discomfort of a nasogastric tube via medical management and earlier evaluation for venting gastrostomy in those with symptoms that respond poorly to optimal medical management.49
WHICH PATIENTS BENEFIT FROM TOTAL PARENTERAL NUTRITION?
The use of total parenteral nutrition in patients with incurable malignancies is controversial. Enteral and parenteral feeding can increase muscle mass and improve functional status and quality of life in a subset of patients who are not suffering from cancer-related cachexia.2,50,51 However, for those whose weight loss and malnutrition are consequences of tumor-mediated cachexia, as demonstrated by anorexia and an elevated C-reactive protein level, parenteral nutrition is unlikely to improve the outcome.51 For most terminally ill patients, retrospective studies have failed to show that parenteral nutrition improves overall survival, performance status, or quality of life.2,48,50–54
Total parenteral nutrition poses risks: it is invasive and requires central venous access, which predisposes to infection; it requires frequent monitoring of hydration and electrolytes; and it predisposes to thrombosis, diarrhea, hyperglycemia, and liver failure.50–56
Total parenteral nutrition may be justified in patients with minimal tumor burden who are candidates for definitive surgery, or in those with a good performance status early in the disease course who have not had chemotherapy or whose cancer responds to chemotherapy.2,50–56
The American College of Physicians discourages the routine use of parenteral nutrition in those with advanced cancer who are undergoing palliative chemotherapy, since few patients benefit and many experience side effects.53
Total parenteral nutrition is much like a medical intervention in that it should be offered or continued only if it provides benefit. Conversations at the time that it is begun must include adverse effects that will lead to its discontinuation, and criteria for response. In certain situations, a limited trial of parenteral nutrition may be considered for patients with an uncertain prognosis or for those who have potentially reversible conditions that limit oral intake.51 In such cases, there should be a clear understanding between patient and physician that parenteral nutrition will be discontinued if it fails to show benefit.53
ADDITIONAL CONCERNS OF PATIENTS AND FAMILIES
‘Will I starve to death?’
Starvation is a fear echoed by patients and families. Ethical discourse on the continuation of nutrition and hydration for the terminally ill has been polarizing.57–60 Withdrawal of nutrition can be perceived as euthanasia.
Advanced cancer patients in general do not experience hunger, and those who do require only small amounts of food for satiation.61 In one report, most patients died of their advanced cancer and not from starvation.52 Artificial hydration and nutrition will thus not influence survival and can even be a burden without benefit in the imminently dying.60 These patients should be encouraged to take food orally for pleasure, as long as it is tolerated, without consideration of end points such as weight gain, body mass index, or albumin levels.
Complaints of thirst and dryness of the mouth are relieved with mouth care, ice chips, lubrication to the lips, and sips of fluid, rather than by parenteral nutrition.59 Patients with a terminal illness experience relief from thirst with minimal intake. The symptom of thirst may be relieved without hydration.34,61 Adequate hydration requires smaller fluid volumes due to decreased body weight, decreased renal clearance of free water, and decreased insensible water losses from reduced physical activity.58
‘Can we continue intravenous hydration so he won’t die of thirst?’
Overzealous intravenous hydration may worsen the symptoms of malignant bowel obstruction. Overhydration can increase secretions in the gut lumen and worsen the secretion-distention-contraction cycle, leading to greater abdominal pain and to nausea and vomiting.7 There is a greater risk of fluid overload in these patients, since they have edema and excessive interstitial fluid. Most have a low serum albumin level, which results in movement of fluid from intravascular to interstitial spaces due to reduced colloid osmotic pressure. In these instances, overzealous hydration can lead to respiratory insufficiency and worsening edema.
In spite of numerous discussions in the medical literature of the benefits and burdens of continual hydration, there is no consensus or guideline. When a patient has limited oral intake, the decision to hydrate should be individualized, with careful assessment of the risks and benefits and in accordance with the patient’s or family’s wishes.57,58
Is treatment at home feasible?
Discharging patients with inoperable malignant bowel obstruction requires careful planning. Patients and family members need to be educated on the use of around-the-clock medications and symptom-targeted, as-needed drugs. Days before discharge, questions about diet need to be clarified. Education about total parenteral nutrition and gastrostomy tube care should be completed before discharge from the hospital.
Drug management should be simplified, or compatible medications should be combined into a single infusion. For example, morphine, glycopyrrolate, and haloperidol or metoclopramide are chemically compatible in standard intravenous solutions and can be combined.
Families feel less anxious about the foreseen and the possible unforeseen course of the illness if they can talk with hospice workers early on. This early involvement also facilitates the transition to home hospice care.
SUMMARY OF IMPORTANT POINTS
Patients with malignant bowel obstruction need a highly individualized approach, tailored to their medical condition, the prognosis, and the goals of care.
Surgery should not be routinely undertaken; less-invasive approaches such as gastric or colonic stenting should be considered first.
Combinations of analgesics, antisecretory drugs, and antiemetics can provide acceptable symptom relief in the inoperable patient.
A venting gastrostomy should be considered if drug therapy fails to reduce nausea and vomiting to an acceptable level.
A nasogastric tube should be used only as a temporizing measure, until symptoms are controlled medically or a venting gastrostomy is placed.
Total parenteral nutrition is of benefit only in patients with intermediate life expectancy who may otherwise die of starvation rather than from the cancer itself.
References
Mercadante S. Intestinal dysfunction and obstruction. In:Walsh D, editor. Palliative Medicine. Philadelphia, PA: Saunders/Elsevier, 2009:1267–1275.
Pasanisi F, Orban A, Scalfi L, et al. Predictors of survival in terminal-cancer patients with irreversible bowel obstruction receiving home parenteral nutrition. Nutrition2001; 17:581–584.
Pameijer CR, Mahvi DM, Stewart JA, Weber SM. Bowel obstruction in patients with metastatic cancer: does intervention influence outcome?Int J Gastrointest Cancer2005; 35:127–133.
Bais JMJ, Schilthuis MS, Ansink AC. Palliative management of intestinal obstruction in patients with advanced gynaecological cancer. J Gynecol Oncol2002; 7:299–305.
Laval G, Arvieux C, Stefani L, Villard ML, Mestrallet JP, Cardin N. Protocol for the treatment of malignant inoperable bowel obstruction: a prospective study of 80 cases at Grenoble University Hospital Center. J Pain Symptom Manage2006; 31:502–512.
Jatoi A, Podratz KC, Gill P, Hartmann LC. Pathophysiology and palliation of inoperable bowel obstruction in patients with ovarian cancer. J Support Oncol2004; 2:323–334.
Ripamonti CI, Easson AM, Gerdes H. Management of malignant bowel obstruction. Eur J Cancer2008; 44:1105–1115.
Roeland E, von Gunten CF. Current concepts in malignant bowel obstruction management. Curr Oncol Rep2009; 11:298–303.
Feuer DJ, Broadley KE, Shepherd JH, Barton DP. Surgery for the resolution of symptoms in malignant bowel obstruction in advanced gynaecological and gastrointestinal cancer. Cochrane Database Syst Rev2000;CD002764.
Feuer DJ, Broadley KE. Corticosteroids for the resolution of malignant bowel obstruction in advanced gynaecological and gastrointestinal cancer. Cochrane Database Syst Rev2000;CD001219.
Woolfson RG, Jennings K, Whalen GF. Management of bowel obstruction in patients with abdominal cancer. Arch Surg1997; 132:1093–1097.
Ripamonti C, Bruera E. Palliative management of malignant bowel obstruction. Int J Gynecol Cancer2002; 12:135–143.
Nellgård P, Bojö L, Cassuto J. Importance of vasoactive intestinal peptide and somatostatin for fluid losses in small-bowel obstruction. Scand J Gastroenterol1995; 30:464–469.
Böhner H, Yang Q, Franke C, Verreet PR, Ohmann C. Simple data from history and physical examination help to exclude bowel obstruction and to avoid radiographic studies in patients with acute abdominal pain. Eur J Surg1998; 164:777–784.
Maglinte DD, Kelvin FM, Sandrasegaran K, et al. Radiology of small bowel obstruction: contemporary approach and controversies. Abdom Imaging2005; 30:160–178.
Maglinte DD, Howard TJ, Lillemoe KD, Sandrasegaran K, Rex DK. Small-bowel obstruction: state-of-the-art imaging and its role in clinical management. Clin Gastroenterol Hepatol2008; 6:130–139.
Silva AC, Pimenta M, Guimarães LS. Small bowel obstruction: what to look for. Radiographics2009; 29:423–439.
Finan PJ, Campbell S, Verma R, et al. The management of malignant large bowel obstruction: ACPGBI position statement. Colorectal Dis2007; 9(suppl 4):1–17.
Kohli MD, Maglinte DD. CT enteroclysis in incomplete small bowel obstruction. Abdom Imaging2009; 34:321–327.
Ha HK, Shin BS, Lee SI, et al. Usefulness of CT in patients with intestinal obstruction who have undergone abdominal surgery for malignancy. AJR Am J Roentgenol1998; 171:1587–1593.
DeBernardo R. Surgical management of malignant bowel obstruction: strategies toward palliation of patients with advanced cancer. Curr Oncol Rep2009; 11:287–292.
Ripamonti C, Twycross R, Baines M, et al; Working Group of the European Association for Palliative Care. Clinical-practice recommendations for the management of bowel obstruction in patients with end-stage cancer. Support Care Cancer2001; 9:223–233.
Mangili G, Aletti G, Frigerio L, et al. Palliative care for intestinal obstruction in recurrent ovarian cancer: a multivariate analysis. Int J Gynecol Cancer2005; 15:830–835.
Turner J, Cummin T, Bennett A, Swift G, Green J. Stents and stentability: treatment for malignant bowel obstruction. Br J Hosp Med (Lond)2008; 69:676–680.
Khot UP, Lang AW, Murali K, Parker MC. Systematic review of the efficacy and safety of colorectal stents. Br J Surg2002; 89:1096–1102.
Hosono S, Ohtani H, Arimoto Y, Kanamiya Y. Endoscopic stenting versus surgical gastroenterostomy for palliation of malignant gastroduodenal obstruction: a meta-analysis. J Gastroenterol2007; 42:283–290.
Del Piano M, Ballarè M, Montino F, et al. Endoscopy or surgery for malignant GI outlet obstruction?Gastrointest Endosc2005; 61:421–426.
Tilney HS, Lovegrove RE, Purkayastha S, et al. Comparison of colonic stenting and open surgery for malignant large bowel obstruction. Surg Endosc2007; 21:225–233.
Holt AP, Patel M, Ahmed MM. Palliation of patients with malignant gastroduodenal obstruction with self-expanding metallic stents: the treatment of choice?Gastrointest Endosc2004; 60:1010–1017.
Dastur JK, Forshaw MJ, Modarai B, Solkar MM, Raymond T, Parker MC. Comparison of short-and long-term outcomes following either insertion of self-expanding metallic stents or emergency surgery in malignant large bowel obstruction. Tech Coloproctol2008; 12:51–55.
Turner J, Cummin T, Bennett A, Swift G, Green J. Stents and stentability: treatment for malignant bowel obstruction. Br J Hosp Med (Lond)2008; 69:676–680.
Ripamonti C, Mercadante S, Groff L, Zecca E, De Conno F, Casuccio A. Role of octreotide, scopolamine butylbromide, and hydration in symptom control of patients with inoperable bowel obstruction and nasogastric tubes: a prospective randomized trial. J Pain Symptom Manage2000; 19:23–34.
Davis MP, Walsh D. Treatment of nausea and vomiting in advanced cancer. Support Care Cancer2000; 8:444–452.
Bicanovsky LK, Lagman RL, Davis MP, Walsh D. Managing nonmalignant chronic abdominal pain and malignant bowel obstruction. Gastroenterol Clin North Am2006; 35:131–142.
Glare P, Pereira G, Kristjanson LJ, Stockler M, Tattersall M. Systematic review of the efficacy of antiemetics in the treatment of nausea in patients with far-advanced cancer. Support Care Cancer2004; 12:432–440.
Davis MP, Furste A. Glycopyrrolate: a useful drug in the palliation of mechanical bowel obstruction. J Pain Symptom Manage1999; 18:153–154.
Ripamonti C, Mercadante S. How to use octreotide for malignant bowel obstruction. J Support Oncol2004; 2:357–364.
Shima Y, Ohtsu A, Shirao K, Sasaki Y. Clinical efficacy and safety of octreotide (SMS201-995) in terminally ill Japanese cancer patients with malignant bowel obstruction. Jpn J Clin Oncol2008; 38:354–359.
Mercadante S, Casuccio A, Mangione S. Medical treatment for inoperable malignant bowel obstruction: a qualitative systematic review. J Pain Symptom Manage2007; 33:217–223.
Mystakidou K, Tsilika E, Kalaidopoulou O, Chondros K, Georgaki S, Papadimitriou L. Comparison of octreotide administration vs conservative treatment in the management of inoperable bowel obstruction in patients with far advanced cancer: a randomized, double-blind, controlled clinical trial. Anticancer Res2002; 22:1187–1192.
Mercadante S, Ripamonti C, Casuccio A, Zecca E, Groff L. Comparison of octreotide and hyoscine butylbromide in controlling gastrointestinal symptoms due to malignant inoperable bowel obstruction. Support Care Cancer2000; 8:188–191.
Myers J, Tamber A, Farhadian M. Management of treatment-related intermittent partial small bowel obstruction: the use of octreotide. J Pain Symptom Manage2010; 39:e1–e3.
Srivastava M, Brito-Dellan N, Davis MP, Leach M, Lagman R. Olanzapine as an antiemetic in refractory nausea and vomiting in advanced cancer. J Pain Symptom Manage2003; 25:578–582.
Bentley A, Boyd K. Use of clinical pictures in the management of nausea and vomiting: a prospective audit. Palliat Med2001; 15:247–253.
Pothuri B, Montemarano M, Gerardi M, et al. Percutaneous endoscopic gastrostomy tube placement in patients with malignant bowel obstruction due to ovarian carcinoma. Gynecol Oncol2005; 96:330–334.
Brooksbank MA, Game PA, Ashby MA. Palliative venting gastrostomy in malignant intestinal obstruction. Palliat Med2002; 16:520–526.
Wang MY, Wu MH, Hsieh DY, et al. Home parenteral nutrition support in adults: experience of a medical center in Asia. JPEN J Parenter Enteral Nutr2007; 31:306–310.
Dy SM. Enteral and parenteral nutrition in terminally ill cancer patients: a review of the literature. Am J Hosp Palliat Care2006; 23:369–377.
Whitworth MK, Whitfield A, Holm S, Shaffer J, Makin W, Jayson GC. Doctor, does this mean I’m going to starve to death?J Clin Oncol2004; 22:199–201.
Hoda D, Jatoi A, Burnes J, Loprinzi C, Kelly D. Should patients with advanced, incurable cancers ever be sent home with total parenteral nutrition? A single institution’s 20-year experience. Cancer2005; 103:863–868.
Philip J, Depczynski B. The role of total parenteral nutrition for patients with irreversible bowel obstruction secondary to gynecological malignancy. J Pain Symptom Manage1997; 13:104–111.
August DA, Thorn D, Fisher RL, Welchek CM. Home parenteral nutrition for patients with inoperable malignant bowel obstruction. JPEN J Parenter Enteral Nutr1991; 15:323–327.
Abu-Rustum NR, Barakat RR, Venkatraman E, Spriggs D. Chemotherapy and total parenteral nutrition for advanced ovarian cancer with bowel obstruction. Gynecol Oncol1997; 64:493–495.
Fainsinger RL, Bruera E. When to treat dehydration in a terminally ill patient?Support Care Cancer1997; 5:205–211.
Steiner N, Bruera E. Methods of hydration in palliative care patients. J Palliat Care1998; 14:6–13.
Slomka J. Withholding nutrition at the end of life: clinical and ethical issues. Cleve Clin J Med2003; 70:548–552.
Chiu TY, Hu WY, Chuang RB, Chen CY. Nutrition and hydration for terminal cancer patients in Taiwan. Support Care Cancer2002; 10:630–636.
McCann RM, Hall WJ, Groth-Juncker A. Comfort care for terminally ill patients. The appropriate use of nutrition and hydration. JAMA1994; 272:1263–1266.
References
Mercadante S. Intestinal dysfunction and obstruction. In:Walsh D, editor. Palliative Medicine. Philadelphia, PA: Saunders/Elsevier, 2009:1267–1275.
Pasanisi F, Orban A, Scalfi L, et al. Predictors of survival in terminal-cancer patients with irreversible bowel obstruction receiving home parenteral nutrition. Nutrition2001; 17:581–584.
Pameijer CR, Mahvi DM, Stewart JA, Weber SM. Bowel obstruction in patients with metastatic cancer: does intervention influence outcome?Int J Gastrointest Cancer2005; 35:127–133.
Bais JMJ, Schilthuis MS, Ansink AC. Palliative management of intestinal obstruction in patients with advanced gynaecological cancer. J Gynecol Oncol2002; 7:299–305.
Laval G, Arvieux C, Stefani L, Villard ML, Mestrallet JP, Cardin N. Protocol for the treatment of malignant inoperable bowel obstruction: a prospective study of 80 cases at Grenoble University Hospital Center. J Pain Symptom Manage2006; 31:502–512.
Jatoi A, Podratz KC, Gill P, Hartmann LC. Pathophysiology and palliation of inoperable bowel obstruction in patients with ovarian cancer. J Support Oncol2004; 2:323–334.
Ripamonti CI, Easson AM, Gerdes H. Management of malignant bowel obstruction. Eur J Cancer2008; 44:1105–1115.
Roeland E, von Gunten CF. Current concepts in malignant bowel obstruction management. Curr Oncol Rep2009; 11:298–303.
Feuer DJ, Broadley KE, Shepherd JH, Barton DP. Surgery for the resolution of symptoms in malignant bowel obstruction in advanced gynaecological and gastrointestinal cancer. Cochrane Database Syst Rev2000;CD002764.
Feuer DJ, Broadley KE. Corticosteroids for the resolution of malignant bowel obstruction in advanced gynaecological and gastrointestinal cancer. Cochrane Database Syst Rev2000;CD001219.
Woolfson RG, Jennings K, Whalen GF. Management of bowel obstruction in patients with abdominal cancer. Arch Surg1997; 132:1093–1097.
Ripamonti C, Bruera E. Palliative management of malignant bowel obstruction. Int J Gynecol Cancer2002; 12:135–143.
Nellgård P, Bojö L, Cassuto J. Importance of vasoactive intestinal peptide and somatostatin for fluid losses in small-bowel obstruction. Scand J Gastroenterol1995; 30:464–469.
Böhner H, Yang Q, Franke C, Verreet PR, Ohmann C. Simple data from history and physical examination help to exclude bowel obstruction and to avoid radiographic studies in patients with acute abdominal pain. Eur J Surg1998; 164:777–784.
Maglinte DD, Kelvin FM, Sandrasegaran K, et al. Radiology of small bowel obstruction: contemporary approach and controversies. Abdom Imaging2005; 30:160–178.
Maglinte DD, Howard TJ, Lillemoe KD, Sandrasegaran K, Rex DK. Small-bowel obstruction: state-of-the-art imaging and its role in clinical management. Clin Gastroenterol Hepatol2008; 6:130–139.
Silva AC, Pimenta M, Guimarães LS. Small bowel obstruction: what to look for. Radiographics2009; 29:423–439.
Finan PJ, Campbell S, Verma R, et al. The management of malignant large bowel obstruction: ACPGBI position statement. Colorectal Dis2007; 9(suppl 4):1–17.
Kohli MD, Maglinte DD. CT enteroclysis in incomplete small bowel obstruction. Abdom Imaging2009; 34:321–327.
Ha HK, Shin BS, Lee SI, et al. Usefulness of CT in patients with intestinal obstruction who have undergone abdominal surgery for malignancy. AJR Am J Roentgenol1998; 171:1587–1593.
DeBernardo R. Surgical management of malignant bowel obstruction: strategies toward palliation of patients with advanced cancer. Curr Oncol Rep2009; 11:287–292.
Ripamonti C, Twycross R, Baines M, et al; Working Group of the European Association for Palliative Care. Clinical-practice recommendations for the management of bowel obstruction in patients with end-stage cancer. Support Care Cancer2001; 9:223–233.
Mangili G, Aletti G, Frigerio L, et al. Palliative care for intestinal obstruction in recurrent ovarian cancer: a multivariate analysis. Int J Gynecol Cancer2005; 15:830–835.
Turner J, Cummin T, Bennett A, Swift G, Green J. Stents and stentability: treatment for malignant bowel obstruction. Br J Hosp Med (Lond)2008; 69:676–680.
Khot UP, Lang AW, Murali K, Parker MC. Systematic review of the efficacy and safety of colorectal stents. Br J Surg2002; 89:1096–1102.
Hosono S, Ohtani H, Arimoto Y, Kanamiya Y. Endoscopic stenting versus surgical gastroenterostomy for palliation of malignant gastroduodenal obstruction: a meta-analysis. J Gastroenterol2007; 42:283–290.
Del Piano M, Ballarè M, Montino F, et al. Endoscopy or surgery for malignant GI outlet obstruction?Gastrointest Endosc2005; 61:421–426.
Tilney HS, Lovegrove RE, Purkayastha S, et al. Comparison of colonic stenting and open surgery for malignant large bowel obstruction. Surg Endosc2007; 21:225–233.
Holt AP, Patel M, Ahmed MM. Palliation of patients with malignant gastroduodenal obstruction with self-expanding metallic stents: the treatment of choice?Gastrointest Endosc2004; 60:1010–1017.
Dastur JK, Forshaw MJ, Modarai B, Solkar MM, Raymond T, Parker MC. Comparison of short-and long-term outcomes following either insertion of self-expanding metallic stents or emergency surgery in malignant large bowel obstruction. Tech Coloproctol2008; 12:51–55.
Turner J, Cummin T, Bennett A, Swift G, Green J. Stents and stentability: treatment for malignant bowel obstruction. Br J Hosp Med (Lond)2008; 69:676–680.
Ripamonti C, Mercadante S, Groff L, Zecca E, De Conno F, Casuccio A. Role of octreotide, scopolamine butylbromide, and hydration in symptom control of patients with inoperable bowel obstruction and nasogastric tubes: a prospective randomized trial. J Pain Symptom Manage2000; 19:23–34.
Davis MP, Walsh D. Treatment of nausea and vomiting in advanced cancer. Support Care Cancer2000; 8:444–452.
Bicanovsky LK, Lagman RL, Davis MP, Walsh D. Managing nonmalignant chronic abdominal pain and malignant bowel obstruction. Gastroenterol Clin North Am2006; 35:131–142.
Glare P, Pereira G, Kristjanson LJ, Stockler M, Tattersall M. Systematic review of the efficacy of antiemetics in the treatment of nausea in patients with far-advanced cancer. Support Care Cancer2004; 12:432–440.
Davis MP, Furste A. Glycopyrrolate: a useful drug in the palliation of mechanical bowel obstruction. J Pain Symptom Manage1999; 18:153–154.
Ripamonti C, Mercadante S. How to use octreotide for malignant bowel obstruction. J Support Oncol2004; 2:357–364.
Shima Y, Ohtsu A, Shirao K, Sasaki Y. Clinical efficacy and safety of octreotide (SMS201-995) in terminally ill Japanese cancer patients with malignant bowel obstruction. Jpn J Clin Oncol2008; 38:354–359.
Mercadante S, Casuccio A, Mangione S. Medical treatment for inoperable malignant bowel obstruction: a qualitative systematic review. J Pain Symptom Manage2007; 33:217–223.
Mystakidou K, Tsilika E, Kalaidopoulou O, Chondros K, Georgaki S, Papadimitriou L. Comparison of octreotide administration vs conservative treatment in the management of inoperable bowel obstruction in patients with far advanced cancer: a randomized, double-blind, controlled clinical trial. Anticancer Res2002; 22:1187–1192.
Mercadante S, Ripamonti C, Casuccio A, Zecca E, Groff L. Comparison of octreotide and hyoscine butylbromide in controlling gastrointestinal symptoms due to malignant inoperable bowel obstruction. Support Care Cancer2000; 8:188–191.
Myers J, Tamber A, Farhadian M. Management of treatment-related intermittent partial small bowel obstruction: the use of octreotide. J Pain Symptom Manage2010; 39:e1–e3.
Srivastava M, Brito-Dellan N, Davis MP, Leach M, Lagman R. Olanzapine as an antiemetic in refractory nausea and vomiting in advanced cancer. J Pain Symptom Manage2003; 25:578–582.
Bentley A, Boyd K. Use of clinical pictures in the management of nausea and vomiting: a prospective audit. Palliat Med2001; 15:247–253.
Pothuri B, Montemarano M, Gerardi M, et al. Percutaneous endoscopic gastrostomy tube placement in patients with malignant bowel obstruction due to ovarian carcinoma. Gynecol Oncol2005; 96:330–334.
Brooksbank MA, Game PA, Ashby MA. Palliative venting gastrostomy in malignant intestinal obstruction. Palliat Med2002; 16:520–526.
Wang MY, Wu MH, Hsieh DY, et al. Home parenteral nutrition support in adults: experience of a medical center in Asia. JPEN J Parenter Enteral Nutr2007; 31:306–310.
Dy SM. Enteral and parenteral nutrition in terminally ill cancer patients: a review of the literature. Am J Hosp Palliat Care2006; 23:369–377.
Whitworth MK, Whitfield A, Holm S, Shaffer J, Makin W, Jayson GC. Doctor, does this mean I’m going to starve to death?J Clin Oncol2004; 22:199–201.
Hoda D, Jatoi A, Burnes J, Loprinzi C, Kelly D. Should patients with advanced, incurable cancers ever be sent home with total parenteral nutrition? A single institution’s 20-year experience. Cancer2005; 103:863–868.
Philip J, Depczynski B. The role of total parenteral nutrition for patients with irreversible bowel obstruction secondary to gynecological malignancy. J Pain Symptom Manage1997; 13:104–111.
August DA, Thorn D, Fisher RL, Welchek CM. Home parenteral nutrition for patients with inoperable malignant bowel obstruction. JPEN J Parenter Enteral Nutr1991; 15:323–327.
Abu-Rustum NR, Barakat RR, Venkatraman E, Spriggs D. Chemotherapy and total parenteral nutrition for advanced ovarian cancer with bowel obstruction. Gynecol Oncol1997; 64:493–495.
Fainsinger RL, Bruera E. When to treat dehydration in a terminally ill patient?Support Care Cancer1997; 5:205–211.
Steiner N, Bruera E. Methods of hydration in palliative care patients. J Palliat Care1998; 14:6–13.
Slomka J. Withholding nutrition at the end of life: clinical and ethical issues. Cleve Clin J Med2003; 70:548–552.
Chiu TY, Hu WY, Chuang RB, Chen CY. Nutrition and hydration for terminal cancer patients in Taiwan. Support Care Cancer2002; 10:630–636.
McCann RM, Hall WJ, Groth-Juncker A. Comfort care for terminally ill patients. The appropriate use of nutrition and hydration. JAMA1994; 272:1263–1266.
Combinations of analgesics, antisecretory drugs, and antiemetics can provide acceptable symptom relief.
A venting gastrostomy should be considered if drug therapy fails to reduce nausea and vomiting to an acceptable level.
A nasogastric tube should be used only as a temporizing measure, until symptoms are controlled medically or a venting gastrostomy is placed.
Total parenteral nutrition is beneficial only in patients with intermediate life expectancy who may otherwise die of starvation rather than the cancer itself.
At the American Society of Nephrology Renal Week 2010, one of the authors (A.D.R.) presented the following question at an In-Depth Nephrology Course on Geriatric Nephrology:
A 65-year-old woman donated a kidney to her son. Before donation, her serum creatinine level was 1.0 mg/dL, her estimated glomerular filtration rate (GFR) was 56 mL/min/1.73 m2, and her measured GFR was 82 mL/min/1.73 m2, which was below the 2.5th percentile for 20-year-old potential kidney donors. The patient had no albuminuria or hypertension and was otherwise healthy. The kidney was biopsied during the transplant surgery. The biopsy revealed 2 of 20 glomeruli as globally sclerosed, a focus of tubular atrophy, and mild arteriosclerosis (findings present in less than 2.5% of 20-year-old donors).
Choose one. Prior to donation, this woman had:
Chronic kidney disease (CKD), and she should not have donated her kidney
CKD, but kidney donation was reasonable
Age-related (senescent) changes in her kidneys, and should not have donated her kidney
Age-related (senescent) changes in her kidneys, but kidney donation was reasonable
Using an electronic response system, 36 (82%) of 44 physicians in the audience chose the last option, even though this patient meets the current definition of CKD (an estimated GFR less than 60 mL/min/1.73 m2) and has chronic parenchymal damage documented by a kidney biopsy.
PROBLEMS WITH THE GFR AND CKD CLASSIFICATION
This question highlights several key problems with the GFR and CKD classification.
First, in low-risk populations such as potential kidney donors, serum-creatinine-based equations such as the Modification of Diet in Renal Disease (MDRD) equation and the Chronic Kidney Disease Epidemiology Study (CKD-EPI) equation substantially underestimate the GFR.1
Second, many healthy older adults with normal serum creatinine levels have an estimated GFR and a measured GFR below the normal range for young adults.2
Third, many healthy older adults have evidence of chronic parenchymal damage on renal biopsy, unlike healthy young adults.3
Finally, many health care providers did not previously recognize that people with a normal serum creatinine level could have a reduced GFR, and widespread use of the estimated GFR has addressed this problem. However, many physicians remain skeptical about efforts this past decade to classify age-related changes in kidney function as a “disease” in the absence of a clear benefit to older patients.4
TWO POINTS ABOUT THE ESTIMATED GFR
In this issue of the Cleveland Clinic Journal of Medicine, Simon and colleagues5 provide a balanced assessment of the benefits and pitfalls of using the estimated GFR in clinical practice. Two points they make deserve further discussion:
Bigger people make more creatinine. GFR can be reported in units of milliliters per minute, or in units normalized to body surface area (mL/min/1.73 m2). Contemporary equations for identifying and classifying CKD use the latter, because the GFR is considered inappropriately low when metabolic waste is not being adequately cleared. It is intuitive that smaller people require less absolute GFR than larger people, who generate more metabolic waste. Indexing GFR to 1.73 m2 assumes that body surface area is a good surrogate for metabolic waste generation. However, whether body surface area is the best surrogate for the rate of metabolic waste generation has long been a subject of debate.6
The relationship between GFR and serum creatinine is not linear. Due to the inverse relationship between serum creatinine and GFR, a small change in serum creatinine from 0.9 to 1.2 mg/dL will represent a relatively large change in GFR (eg, 85 to 65 mL/min/1.73 m2), whereas a large change in serum creatinine from 5 to 9 mg/dL will represent a smaller change in GFR (eg, 10 to 5 mL/min/1.73 m2). The latter may be of great concern since it represents a fall in GFR to levels at which dialysis is likely needed. With the former, subtle changes in serum creatinine represent large changes in GFR, but there is also much more day-to-day variability in GFR in the normal or near-normal range than in the advanced range of kidney disease. This is one of the reasons the MDRD and CKD-EPI equations were developed, using logarithmic models that emphasize percentage instead of absolute differences in GFR.
BEYOND CREATININE?
As Simon and colleagues point out,5 although serum creatinine is a flawed surrogate for GFR, there are many problems with determining GFR by other means.
Direct GFR measurement relies on the use of an exogenous marker such as inulin or iothalamate that is infused or injected, followed by timed urine and plasma measurements to calculate GFR by the urinary clearance method (UV/P, where U is the concentration of the marker in the urine, V is the urine volume, and P is the concentration of the marker in the plasma). Alternatively, timed plasma measurements of the marker alone can be used to determine GFR by the plasma clearance method. The problem is that direct GFR measurement is costly, invasive, imprecise, time-consuming, and impractical in most clinical settings.
Exogenous markers for determining GFR are chosen because they are metabolically inert, are cleared by glomerular filtration without tubular secretion or reabsorption, and have no extrarenal clearance via the liver or intestines. Endogenous markers such as serum creatinine do not fulfill all of these ideal criteria.
Simon and colleagues highlight the problem of using the estimated GFR to screen for CKD in populations of ostensibly healthy persons.5 The MDRD and CKD-EPI equations contain demographic variables to approximate the creatinine generation rate. The primary source of creatinine generation is muscle, and the coefficients in these equations reflect the higher muscle mass of younger individuals, males, and African Americans. However, any creatinine-based equation is fundamentally flawed because overall health also affects muscle mass: healthy people have greater muscle mass than people with chronic illness, including those with CKD. Therefore, at the same serum creatinine level, a healthy person has a higher GFR than a patient with CKD.1,7 This problem leads to circular reasoning, since you need to know whether the patient has CKD or is healthy in order to accurately estimate GFR, but estimated GFR is being used to determine whether the patient is healthy or has CKD.
Therefore, other endogenous markers that are also eliminated via glomerular filtration, such as cystatin C, have been used to construct equations that estimate GFR. Unfortunately, factors other than GFR, such as inflammation, can also influence blood cystatin C levels. This in turn impairs the accuracy of equations that use cystatin C to estimate GFR in the general population.8 No known endogenous marker of GFR can be used in all patients without any confounding factors.
To rectify this problem, recent studies have investigated the use of a confirmatory test to determine which patients with a creatinine-based estimated GFR less than 60 mL/min/1.73 m2 actually have kidney disease or have a false-positive result due to higher-than-average creatinine generation. Both albuminuria and elevated serum cystatin C are examples of useful confirmatory tests that substantially decrease the misdiagnosis of CKD in healthy adults with an estimated GFR less than 60 mL/min/1.73 m2.9,10
Imagine if we identified and staged systemic lupus erythematosus on the basis of antinuclear antibody levels alone: this would parallel the current approach that largely uses serum creatinine alone to classify CKD. Confirmatory tests and considering patient-specific risk factors could avoid potential harm to healthy individuals and yet retain gains that have been made to improve the interpretation of serum creatinine levels in CKD patients.
References
Tent H, Rook M, Stevens LA, et a.l Renal function equations before and after living kidney donation: a within-individual comparison of performance at different levels of renal function. Clin J Am Soc Nephrol2010; 5:1960–1968.
Poggio ED, Rule AD, Tanchanco R, et al. Demographic and clinical characteristics associated with glomerular filtration rates in living kidney donors. Kidney Int2009; 75:1079–1087.
Rule AD, Amer H, Cornell LD, et al. The association between age and nephrosclerosis on renal biopsy among healthy adults. Ann Intern Med2010; 152:561–567.
Spence D. Bad medicine: chronic kidney disease. BMJ2010; 340:c3188.
Simon J, Amde M, Poggio E. Interpreting the estimated glomerular filtration rate in the primary care setting: benefits and pitfalls. Cleve Clin J Med2011; 78:189–195.
Daugirdas JT, Meyer K, Greene T, Butler RS, Poggio ED. Scaling of measured glomerular filtration rate in kidney donor candidates by anthropometric estimates of body surface area, body water, metabolic rate, or liver size. Clin J Am Soc Nephrol2009; 4:1575–1583.
Rule AD, Larson TS, Bergstralh EJ, Slezak JM, Jacobsen SJ, Cosio FG. Using serum creatinine to estimate glomerular filtration rate: accuracy in good health and in chronic kidney disease. Ann Intern Med2004; 141:929–937.
Eriksen BO, Mathisen UD, Melsom T, et al. Cystatin C is not a better estimator of GFR than plasma creatinine in the general population. Kidney Int2010; 78:1305–1311.
Tonelli M, Muntner P, Lloyd A, et al. Using proteinuria and estimated glomerular filtration rate to classify risk in patients with chronic kidney disease: a cohort study. Ann Intern Med2011; 154:12–21.
Peralta CA, Katz R, Sarnak MJ, et al. Cystatin C identifies chronic kidney disease patients at higher risk for complications. J Am Soc Nephrol2011; 22:147–155.
At the American Society of Nephrology Renal Week 2010, one of the authors (A.D.R.) presented the following question at an In-Depth Nephrology Course on Geriatric Nephrology:
A 65-year-old woman donated a kidney to her son. Before donation, her serum creatinine level was 1.0 mg/dL, her estimated glomerular filtration rate (GFR) was 56 mL/min/1.73 m2, and her measured GFR was 82 mL/min/1.73 m2, which was below the 2.5th percentile for 20-year-old potential kidney donors. The patient had no albuminuria or hypertension and was otherwise healthy. The kidney was biopsied during the transplant surgery. The biopsy revealed 2 of 20 glomeruli as globally sclerosed, a focus of tubular atrophy, and mild arteriosclerosis (findings present in less than 2.5% of 20-year-old donors).
Choose one. Prior to donation, this woman had:
Chronic kidney disease (CKD), and she should not have donated her kidney
CKD, but kidney donation was reasonable
Age-related (senescent) changes in her kidneys, and should not have donated her kidney
Age-related (senescent) changes in her kidneys, but kidney donation was reasonable
Using an electronic response system, 36 (82%) of 44 physicians in the audience chose the last option, even though this patient meets the current definition of CKD (an estimated GFR less than 60 mL/min/1.73 m2) and has chronic parenchymal damage documented by a kidney biopsy.
PROBLEMS WITH THE GFR AND CKD CLASSIFICATION
This question highlights several key problems with the GFR and CKD classification.
First, in low-risk populations such as potential kidney donors, serum-creatinine-based equations such as the Modification of Diet in Renal Disease (MDRD) equation and the Chronic Kidney Disease Epidemiology Study (CKD-EPI) equation substantially underestimate the GFR.1
Second, many healthy older adults with normal serum creatinine levels have an estimated GFR and a measured GFR below the normal range for young adults.2
Third, many healthy older adults have evidence of chronic parenchymal damage on renal biopsy, unlike healthy young adults.3
Finally, many health care providers did not previously recognize that people with a normal serum creatinine level could have a reduced GFR, and widespread use of the estimated GFR has addressed this problem. However, many physicians remain skeptical about efforts this past decade to classify age-related changes in kidney function as a “disease” in the absence of a clear benefit to older patients.4
TWO POINTS ABOUT THE ESTIMATED GFR
In this issue of the Cleveland Clinic Journal of Medicine, Simon and colleagues5 provide a balanced assessment of the benefits and pitfalls of using the estimated GFR in clinical practice. Two points they make deserve further discussion:
Bigger people make more creatinine. GFR can be reported in units of milliliters per minute, or in units normalized to body surface area (mL/min/1.73 m2). Contemporary equations for identifying and classifying CKD use the latter, because the GFR is considered inappropriately low when metabolic waste is not being adequately cleared. It is intuitive that smaller people require less absolute GFR than larger people, who generate more metabolic waste. Indexing GFR to 1.73 m2 assumes that body surface area is a good surrogate for metabolic waste generation. However, whether body surface area is the best surrogate for the rate of metabolic waste generation has long been a subject of debate.6
The relationship between GFR and serum creatinine is not linear. Due to the inverse relationship between serum creatinine and GFR, a small change in serum creatinine from 0.9 to 1.2 mg/dL will represent a relatively large change in GFR (eg, 85 to 65 mL/min/1.73 m2), whereas a large change in serum creatinine from 5 to 9 mg/dL will represent a smaller change in GFR (eg, 10 to 5 mL/min/1.73 m2). The latter may be of great concern since it represents a fall in GFR to levels at which dialysis is likely needed. With the former, subtle changes in serum creatinine represent large changes in GFR, but there is also much more day-to-day variability in GFR in the normal or near-normal range than in the advanced range of kidney disease. This is one of the reasons the MDRD and CKD-EPI equations were developed, using logarithmic models that emphasize percentage instead of absolute differences in GFR.
BEYOND CREATININE?
As Simon and colleagues point out,5 although serum creatinine is a flawed surrogate for GFR, there are many problems with determining GFR by other means.
Direct GFR measurement relies on the use of an exogenous marker such as inulin or iothalamate that is infused or injected, followed by timed urine and plasma measurements to calculate GFR by the urinary clearance method (UV/P, where U is the concentration of the marker in the urine, V is the urine volume, and P is the concentration of the marker in the plasma). Alternatively, timed plasma measurements of the marker alone can be used to determine GFR by the plasma clearance method. The problem is that direct GFR measurement is costly, invasive, imprecise, time-consuming, and impractical in most clinical settings.
Exogenous markers for determining GFR are chosen because they are metabolically inert, are cleared by glomerular filtration without tubular secretion or reabsorption, and have no extrarenal clearance via the liver or intestines. Endogenous markers such as serum creatinine do not fulfill all of these ideal criteria.
Simon and colleagues highlight the problem of using the estimated GFR to screen for CKD in populations of ostensibly healthy persons.5 The MDRD and CKD-EPI equations contain demographic variables to approximate the creatinine generation rate. The primary source of creatinine generation is muscle, and the coefficients in these equations reflect the higher muscle mass of younger individuals, males, and African Americans. However, any creatinine-based equation is fundamentally flawed because overall health also affects muscle mass: healthy people have greater muscle mass than people with chronic illness, including those with CKD. Therefore, at the same serum creatinine level, a healthy person has a higher GFR than a patient with CKD.1,7 This problem leads to circular reasoning, since you need to know whether the patient has CKD or is healthy in order to accurately estimate GFR, but estimated GFR is being used to determine whether the patient is healthy or has CKD.
Therefore, other endogenous markers that are also eliminated via glomerular filtration, such as cystatin C, have been used to construct equations that estimate GFR. Unfortunately, factors other than GFR, such as inflammation, can also influence blood cystatin C levels. This in turn impairs the accuracy of equations that use cystatin C to estimate GFR in the general population.8 No known endogenous marker of GFR can be used in all patients without any confounding factors.
To rectify this problem, recent studies have investigated the use of a confirmatory test to determine which patients with a creatinine-based estimated GFR less than 60 mL/min/1.73 m2 actually have kidney disease or have a false-positive result due to higher-than-average creatinine generation. Both albuminuria and elevated serum cystatin C are examples of useful confirmatory tests that substantially decrease the misdiagnosis of CKD in healthy adults with an estimated GFR less than 60 mL/min/1.73 m2.9,10
Imagine if we identified and staged systemic lupus erythematosus on the basis of antinuclear antibody levels alone: this would parallel the current approach that largely uses serum creatinine alone to classify CKD. Confirmatory tests and considering patient-specific risk factors could avoid potential harm to healthy individuals and yet retain gains that have been made to improve the interpretation of serum creatinine levels in CKD patients.
At the American Society of Nephrology Renal Week 2010, one of the authors (A.D.R.) presented the following question at an In-Depth Nephrology Course on Geriatric Nephrology:
A 65-year-old woman donated a kidney to her son. Before donation, her serum creatinine level was 1.0 mg/dL, her estimated glomerular filtration rate (GFR) was 56 mL/min/1.73 m2, and her measured GFR was 82 mL/min/1.73 m2, which was below the 2.5th percentile for 20-year-old potential kidney donors. The patient had no albuminuria or hypertension and was otherwise healthy. The kidney was biopsied during the transplant surgery. The biopsy revealed 2 of 20 glomeruli as globally sclerosed, a focus of tubular atrophy, and mild arteriosclerosis (findings present in less than 2.5% of 20-year-old donors).
Choose one. Prior to donation, this woman had:
Chronic kidney disease (CKD), and she should not have donated her kidney
CKD, but kidney donation was reasonable
Age-related (senescent) changes in her kidneys, and should not have donated her kidney
Age-related (senescent) changes in her kidneys, but kidney donation was reasonable
Using an electronic response system, 36 (82%) of 44 physicians in the audience chose the last option, even though this patient meets the current definition of CKD (an estimated GFR less than 60 mL/min/1.73 m2) and has chronic parenchymal damage documented by a kidney biopsy.
PROBLEMS WITH THE GFR AND CKD CLASSIFICATION
This question highlights several key problems with the GFR and CKD classification.
First, in low-risk populations such as potential kidney donors, serum-creatinine-based equations such as the Modification of Diet in Renal Disease (MDRD) equation and the Chronic Kidney Disease Epidemiology Study (CKD-EPI) equation substantially underestimate the GFR.1
Second, many healthy older adults with normal serum creatinine levels have an estimated GFR and a measured GFR below the normal range for young adults.2
Third, many healthy older adults have evidence of chronic parenchymal damage on renal biopsy, unlike healthy young adults.3
Finally, many health care providers did not previously recognize that people with a normal serum creatinine level could have a reduced GFR, and widespread use of the estimated GFR has addressed this problem. However, many physicians remain skeptical about efforts this past decade to classify age-related changes in kidney function as a “disease” in the absence of a clear benefit to older patients.4
TWO POINTS ABOUT THE ESTIMATED GFR
In this issue of the Cleveland Clinic Journal of Medicine, Simon and colleagues5 provide a balanced assessment of the benefits and pitfalls of using the estimated GFR in clinical practice. Two points they make deserve further discussion:
Bigger people make more creatinine. GFR can be reported in units of milliliters per minute, or in units normalized to body surface area (mL/min/1.73 m2). Contemporary equations for identifying and classifying CKD use the latter, because the GFR is considered inappropriately low when metabolic waste is not being adequately cleared. It is intuitive that smaller people require less absolute GFR than larger people, who generate more metabolic waste. Indexing GFR to 1.73 m2 assumes that body surface area is a good surrogate for metabolic waste generation. However, whether body surface area is the best surrogate for the rate of metabolic waste generation has long been a subject of debate.6
The relationship between GFR and serum creatinine is not linear. Due to the inverse relationship between serum creatinine and GFR, a small change in serum creatinine from 0.9 to 1.2 mg/dL will represent a relatively large change in GFR (eg, 85 to 65 mL/min/1.73 m2), whereas a large change in serum creatinine from 5 to 9 mg/dL will represent a smaller change in GFR (eg, 10 to 5 mL/min/1.73 m2). The latter may be of great concern since it represents a fall in GFR to levels at which dialysis is likely needed. With the former, subtle changes in serum creatinine represent large changes in GFR, but there is also much more day-to-day variability in GFR in the normal or near-normal range than in the advanced range of kidney disease. This is one of the reasons the MDRD and CKD-EPI equations were developed, using logarithmic models that emphasize percentage instead of absolute differences in GFR.
BEYOND CREATININE?
As Simon and colleagues point out,5 although serum creatinine is a flawed surrogate for GFR, there are many problems with determining GFR by other means.
Direct GFR measurement relies on the use of an exogenous marker such as inulin or iothalamate that is infused or injected, followed by timed urine and plasma measurements to calculate GFR by the urinary clearance method (UV/P, where U is the concentration of the marker in the urine, V is the urine volume, and P is the concentration of the marker in the plasma). Alternatively, timed plasma measurements of the marker alone can be used to determine GFR by the plasma clearance method. The problem is that direct GFR measurement is costly, invasive, imprecise, time-consuming, and impractical in most clinical settings.
Exogenous markers for determining GFR are chosen because they are metabolically inert, are cleared by glomerular filtration without tubular secretion or reabsorption, and have no extrarenal clearance via the liver or intestines. Endogenous markers such as serum creatinine do not fulfill all of these ideal criteria.
Simon and colleagues highlight the problem of using the estimated GFR to screen for CKD in populations of ostensibly healthy persons.5 The MDRD and CKD-EPI equations contain demographic variables to approximate the creatinine generation rate. The primary source of creatinine generation is muscle, and the coefficients in these equations reflect the higher muscle mass of younger individuals, males, and African Americans. However, any creatinine-based equation is fundamentally flawed because overall health also affects muscle mass: healthy people have greater muscle mass than people with chronic illness, including those with CKD. Therefore, at the same serum creatinine level, a healthy person has a higher GFR than a patient with CKD.1,7 This problem leads to circular reasoning, since you need to know whether the patient has CKD or is healthy in order to accurately estimate GFR, but estimated GFR is being used to determine whether the patient is healthy or has CKD.
Therefore, other endogenous markers that are also eliminated via glomerular filtration, such as cystatin C, have been used to construct equations that estimate GFR. Unfortunately, factors other than GFR, such as inflammation, can also influence blood cystatin C levels. This in turn impairs the accuracy of equations that use cystatin C to estimate GFR in the general population.8 No known endogenous marker of GFR can be used in all patients without any confounding factors.
To rectify this problem, recent studies have investigated the use of a confirmatory test to determine which patients with a creatinine-based estimated GFR less than 60 mL/min/1.73 m2 actually have kidney disease or have a false-positive result due to higher-than-average creatinine generation. Both albuminuria and elevated serum cystatin C are examples of useful confirmatory tests that substantially decrease the misdiagnosis of CKD in healthy adults with an estimated GFR less than 60 mL/min/1.73 m2.9,10
Imagine if we identified and staged systemic lupus erythematosus on the basis of antinuclear antibody levels alone: this would parallel the current approach that largely uses serum creatinine alone to classify CKD. Confirmatory tests and considering patient-specific risk factors could avoid potential harm to healthy individuals and yet retain gains that have been made to improve the interpretation of serum creatinine levels in CKD patients.
References
Tent H, Rook M, Stevens LA, et a.l Renal function equations before and after living kidney donation: a within-individual comparison of performance at different levels of renal function. Clin J Am Soc Nephrol2010; 5:1960–1968.
Poggio ED, Rule AD, Tanchanco R, et al. Demographic and clinical characteristics associated with glomerular filtration rates in living kidney donors. Kidney Int2009; 75:1079–1087.
Rule AD, Amer H, Cornell LD, et al. The association between age and nephrosclerosis on renal biopsy among healthy adults. Ann Intern Med2010; 152:561–567.
Spence D. Bad medicine: chronic kidney disease. BMJ2010; 340:c3188.
Simon J, Amde M, Poggio E. Interpreting the estimated glomerular filtration rate in the primary care setting: benefits and pitfalls. Cleve Clin J Med2011; 78:189–195.
Daugirdas JT, Meyer K, Greene T, Butler RS, Poggio ED. Scaling of measured glomerular filtration rate in kidney donor candidates by anthropometric estimates of body surface area, body water, metabolic rate, or liver size. Clin J Am Soc Nephrol2009; 4:1575–1583.
Rule AD, Larson TS, Bergstralh EJ, Slezak JM, Jacobsen SJ, Cosio FG. Using serum creatinine to estimate glomerular filtration rate: accuracy in good health and in chronic kidney disease. Ann Intern Med2004; 141:929–937.
Eriksen BO, Mathisen UD, Melsom T, et al. Cystatin C is not a better estimator of GFR than plasma creatinine in the general population. Kidney Int2010; 78:1305–1311.
Tonelli M, Muntner P, Lloyd A, et al. Using proteinuria and estimated glomerular filtration rate to classify risk in patients with chronic kidney disease: a cohort study. Ann Intern Med2011; 154:12–21.
Peralta CA, Katz R, Sarnak MJ, et al. Cystatin C identifies chronic kidney disease patients at higher risk for complications. J Am Soc Nephrol2011; 22:147–155.
References
Tent H, Rook M, Stevens LA, et a.l Renal function equations before and after living kidney donation: a within-individual comparison of performance at different levels of renal function. Clin J Am Soc Nephrol2010; 5:1960–1968.
Poggio ED, Rule AD, Tanchanco R, et al. Demographic and clinical characteristics associated with glomerular filtration rates in living kidney donors. Kidney Int2009; 75:1079–1087.
Rule AD, Amer H, Cornell LD, et al. The association between age and nephrosclerosis on renal biopsy among healthy adults. Ann Intern Med2010; 152:561–567.
Spence D. Bad medicine: chronic kidney disease. BMJ2010; 340:c3188.
Simon J, Amde M, Poggio E. Interpreting the estimated glomerular filtration rate in the primary care setting: benefits and pitfalls. Cleve Clin J Med2011; 78:189–195.
Daugirdas JT, Meyer K, Greene T, Butler RS, Poggio ED. Scaling of measured glomerular filtration rate in kidney donor candidates by anthropometric estimates of body surface area, body water, metabolic rate, or liver size. Clin J Am Soc Nephrol2009; 4:1575–1583.
Rule AD, Larson TS, Bergstralh EJ, Slezak JM, Jacobsen SJ, Cosio FG. Using serum creatinine to estimate glomerular filtration rate: accuracy in good health and in chronic kidney disease. Ann Intern Med2004; 141:929–937.
Eriksen BO, Mathisen UD, Melsom T, et al. Cystatin C is not a better estimator of GFR than plasma creatinine in the general population. Kidney Int2010; 78:1305–1311.
Tonelli M, Muntner P, Lloyd A, et al. Using proteinuria and estimated glomerular filtration rate to classify risk in patients with chronic kidney disease: a cohort study. Ann Intern Med2011; 154:12–21.
Peralta CA, Katz R, Sarnak MJ, et al. Cystatin C identifies chronic kidney disease patients at higher risk for complications. J Am Soc Nephrol2011; 22:147–155.
Chronic kidney disease is most often discovered and diagnosed by primary care providers. The equations for estimating the glomerular filtration rate (GFR) facilitate earlier detection of this disease. However, the estimated GFR must be interpreted in the context of the individual patient. The diagnostic criteria and staging of chronic kidney disease must be understood so that it can be recognized and managed at the earliest possible stage. In this way, primary care physicians and nephrologists can better coordinate the care of these patients.
THE STAGES OF RENAL DISEASE AND THE GFR
Before 2002, an organized approach to the clinical management of patients with renal dysfunction was hampered by a lack of a standardized way to define this condition. This changed when the National Kidney Foundation, through the Kidney Disease Outcomes Quality Initiative (K/DOQI),1 defined the stages of chronic kidney disease based on the GFR as estimated by the Modification of Diet in Renal Disease (MDRD) equation.2,3
This system has increased the recognition of chronic kidney disease by the health care community and the general public. But the entire system hinges on the utility, accuracy, and reliability of the equations used to estimate the GFR.
In this article, we review the concepts of renal clearance and how to interpret the GFR in healthy patients and in those with chronic kidney disease. The following cases illustrate the interpretation of GFR in the context of patient care.
CASE 1: A 60-YEAR-OLD WOMAN WITH A ‘NORMAL’ CREATININE LEVEL
A 60-year-old white woman with no significant medical history has routine laboratory tests done as part of her annual physical examination. She weighs 135 pounds (61.2 kg) and is 64 inches (163 cm) tall. Her serum creatinine level is 1.1 mg/dL; her estimated GFR is 53 mL/min/1.73 m2. A urine dipstick test for protein and blood is normal.
CASE 2: PROTEINURIA WITH A PRESERVED GFR
A 20-year-old African American man with no medical history is undergoing routine blood testing. His serum creatinine level is 1.1 mg/dL; his estimated GFR is reported as “> 60 mL/min/1.73 m2” (calculated at 109 mL/min/1.73 m2). He is 72 inches (183 cm) tall and weighs 180 pounds (83.0 kg); he lifts weights four times a week. Urine dipstick testing reveals 3+ proteinuria.
SERUM CREATININE: AN IMPERFECT MARKER OF KIDNEY FUNCTION
Of the various functions of the kidney, the ability of the glomeruli to filter the blood, as assessed by the GFR, is considered the best index of overall kidney function.4,5 The GFR can be thought of as the clearance of a substance from the plasma by the kidney in a period of time. This is useful because no method is available to routinely and directly measure filtration across the glomerular basement membrane.
Substances that are cleared by the kidney are used to estimate the GFR. The ideal substance for this estimate is one that is cleared only by filtration and not through metabolism or excretion by other means.
The urinary clearance of the exogenous substance inulin is considered the gold standard method, but radioisotopes such as iothalamate and other markers have replaced inulin in clinical laboratories. Because these methods are expensive, time-consuming, and not widely available, alternative methods that use endogenous markers such as creatinine have been developed for clinical practice.
The serum creatinine concentration possesses many of the qualities of an ideal marker for estimating kidney function. Creatinine is produced by the body at a relatively constant rate under normal conditions and is easy and inexpensive to measure. However, it has several limitations:
Data presented in Rolin HA III, et al. Evaluation of glomerular filtration rate and renal plasma flow. In: Jacobson HR, et al, eds. The Principles and Practice of Nephrology. St. Louis: Mosby-Year Book 1995:8-13.
Figure 1. The relationship between serum creatinine concentration, creatinine clearance, and glomerular filtration rate (GFR), shown with a 95% confidence interval (blue band). Points A and B illustrate the large change in GFR that results from a small change in serum creatinine at higher levels of kidney function. Points C and D illustrate the small change in GFR that results from a large change in serum creatinine at lower levels of kidney function. Creatinine clearance tends to overestimate the GFR.Its clearance does not solely reflect glomerular filtration because the renal tubules also excrete it into the urine.6 As a result, creatinine clearance (see below) will tend to overestimate the GFR (Figure 1).
The serum creatinine concentration is directly dependent on muscle mass, which varies with sex (women tend to have less muscle mass as a percent of body weight than men), age (muscle mass decreases with age), and race (African Americans have a higher serum creatinine level for the same GFR than other Americans).6 Thus, there is no “normal” value for serum creatinine that applies to all patients.
Other factors can alter the creatinine level without changing the GFR, such as changes in dietary protein intake, exercise, and drugs such as cimetidine7 and fibrates8 (Table 1).
Another important point is that the relationship between the serum creatinine concentration and the GFR is parabolic.9 At high kidney function, large changes in the GFR are reflected by very small changes in serum creatinine—the GFR must fall quite a bit before the serum creatinine level rises very much (points A to B in Figure 1). At lower kidney function, small changes in GFR are reflected by large changes in serum creatinine (points C to D in Figure 1). This phenomenon can cause physicians to view small changes in creatinine as unimportant in patients with creatinine levels in the normal or near-normal range. Conversely, small changes may be due to random error inherent in the methods of measuring creatinine rather than to changes in kidney function.
Because the serum creatinine concentration by itself may be misleading when estimating GFR, the National Kidney Foundation and the National Kidney Disease Education Program recommend that it not be used on its own to estimate kidney function.
ESTIMATING THE GFR
Measuring 24-hour creatinine clearance
Measuring 24-hour creatinine clearance involves measuring the concentrations of creatinine in the serum and the urine and the volume of urine excreted in 24 hours.
The 24-hour creatinine clearance was long considered the best alternative to the serum creatinine concentration for assessing kidney function, as it adjusts for changes in the creatinine concentration by taking into account creatinine’s excretion in the urine. However, 24-hour urine collection is burdensome for the patient, and the results are not always reliable because of variations in collection technique. Also, using the creatinine clearance does not resolve problems with using the serum creatinine concentration, such as tubular secretion and overestimation of GFR.
In an effort to more easily estimate GFR from blood tests alone, efforts to develop mathematical equations that more closely estimate GFR began over 40 years ago. These equations take into account factors such as age, sex, and ethnicity. The best known of these are the Cockcroft and Gault10 and the MDRD equations.5
The Cockcroft-Gault equation
The Cockcroft-Gault equation is fairly simple, using serum creatinine, ideal body weight, and an adjustment factor for sex. Its main drawbacks are that it was developed to model creatinine clearance, itself an imperfect estimation of GFR, and it depends heavily on the accuracy of the value for “lean” body weight used in the equation.
The MDRD equation
The MDRD equation has now largely replaced the Cockcroft-Gault equation. Developed using iothalamate GFR measurements, it therefore estimates GFR rather than the less-accurate creatinine clearance. Also, it is normalized to a standard body surface area (1.73 m2), obviating the need to determine ideal body weight.
Since the estimated GFR can often be calculated using data available in most electronic medical record systems, it can be reported directly with any laboratory report that includes a serum creatinine value.
The main drawback of the MDRD equation is that it tends to underestimate GFR at higher ranges of kidney function, ie, higher than 60 mL/min/1.73 m2).3,11
The CKD-EPI equation
The Chronic Kidney Disease Epidemiology Collaboration study (CKD-EPI) equation,12 published in 2009, is expected to eventually replace the currently used MDRD equation, as it performs better at higher ranges of GFR.
Although the CKD-EPI equation still lacks precision and accuracy, it underestimates GFR to a lesser degree than the MDRD equation in patients with preserved renal function. Also, it was developed with the objective of reporting a specific value even when the estimated GFR is greater than 60 mL/min/1.73 m2. (In contrast, when laboratories use the MDRD equation, the recommendation is to report any value above this level as “greater than 60 mL/min/1.73 m2”).
A limitation of all equations that use the serum creatinine concentration to assess kidney function is the assumption that creatinine production is both stable over time and similar among patients. As a result, these equations should not be used in situations in which renal function is changing rapidly, such as in acute kidney injury. Also, they should be used with caution in patients at the extremes of body mass, since they underestimate GFR in very muscular patients (eg, as in case 2) and overestimate GFR in very small patients (eg, as in case 1).
As most patients with established medical problems have blood drawn periodically for routine chemistry panels, the diagnosis of chronic kidney disease often occurs through routine testing. For patients who do not yet carry this diagnosis, it is important to recognize the risk factors for chronic kidney disease (Table 2) and to determine who should be screened.
In general, anyone at higher risk of chronic kidney disease should be screened for it. This group includes US minorities and patients with hypertension, cardiovascular disease, and diabetes mellitus, among others.13 Screening includes an assessment of estimated GFR and urinalysis for proteinuria or hematuria.
CHRONIC KIDNEY DISEASE DEFINED: DAMAGE AND DURATION
The definition of chronic kidney disease contains two components—kidney damage and duration (Table 3).1
The kidney damage can be either parenchymal renal damage independent of GFR (for example, cystic disease, glomerular hematuria, or proteinuria) or depressed GFR independent of evidence of parenchymal renal disease (an estimated GFR of less than 60 mL/min/1.73 m2).
The duration component requires that the abnormality be present for at least 3 months (ie, chronic).
Concerns about the definition
This definition has not been without controversy.
An unintended consequence of not reporting estimated GFR values above 60 mL/min/1.73 m2 in absolute numbers is that providers may ignore changes in serum creatinine at estimated GFRs in this range, as they assume that the kidney function is “normal.” This may change in the future if the CKD-EPI equation is used, which produces less bias at slightly higher GFRs.
Providers may also tend to focus solely on the estimated GFR criterion and ignore other evidence of chronic kidney disease, such as abnormalities in urinalysis or imaging studies. For example, proteinuria has been shown to be more important than absolute GFR values in predicting progression of renal dysfunction and cardiovascular risk.14 Proteinuria, especially in the setting of an estimated GFR above 60 mL/min/1.73 m2, can be missed if not screened for and underappreciated once found.
Moreover, in elderly patients, the current GFR equations underperform at borderline GFR values and can yield depressed values even at impressively “normal” serum creatinine levels. As a result, there is concern that chronic kidney disease is being overdiagnosed under the current system. This is especially worrisome in elderly white women without risk factors for chronic kidney disease (eg, as in case 1).
In addition, the question arises whether the arbitrary cutoff for chronic kidney disease— 60 mL/min/1.73 m2—applies to all populations.15,16 The utility of classifying someone as having chronic kidney disease who has an estimated GFR of 55 mL/min/1.73 m2 and no risk factors for chronic kidney disease (Table 2) should be questioned if the risk of progressing to end-stage renal disease or suffering a cardiovascular event is only minimally higher than in patients with a higher estimated GFR.6,17 If the true purpose of developing the chronic kidney disease classification system is to improve patient care and outcomes, then it is of no benefit to overclassify such patients. Indeed, the stress induced by the diagnosis and the negative implications on insurance coverage and health care costs may outweigh any benefits.18
Nevertheless, these concerns do not invalidate the entire chronic kidney disease definition system, but have stimulated current efforts to improve it based on outcomes research.19
CASES REVISITED
Case 1: Problems with estimating GFR in a small woman
Case 1 has several points to note.
The patient’s small body size reflects low-level creatinine production. It is not atypical to find serum creatinine levels of 0.5 mg/dL in such patients. Thus, her serum creatinine level of 1.1 mg/dL may be abnormal. The fact that the MDRD equation “normalizes” the result to 1.73 m2 of body surface area in patients with very low muscle mass will lead to an overestimation of GFR. However, she has no risk factors for chronic kidney disease.
Additionally, in up to two-thirds of patients kidney function declines with age.20 Whether or not this is “normal aging” of the kidney, it is not clear that this decline in GFR reflects an underlying pathologic process.
Finally, since the patient is an older white woman, the estimated GFR tends to underestimate the true GFR. So while her body size may predispose to an overestimation of GFR, her age, race, and sex predispose to an underestimation of GFR. Many nephrologists would simply order urinalysis and ultrasonography to rule out other evidence of renal dysfunction, then recommend routine monitoring of kidney function in this case.
Case 2: Proteinuria is not normal
In case 2, because the patient is African American, young, and male, his creatinine level yields a higher estimated GFR than in case 1, despite having the same value. However, his estimated GFR still underestimates his true GFR because of his greater creatinine production due to his muscular physique.
This patient subsequently underwent iothalamate GFR testing, which yielded a GFR of 115 mL/min/1.73 m2. However, he has dipstick-positive proteinuria, which, if confirmed on further testing, would meet the criteria for chronic kidney disease and put him at a higher risk of cardiovascular events and progression to lower kidney function than the patient in case 1. He also needs to be screened for undiagnosed hypertension and underlying glomerular disease.
REFERRAL TO (AND COLLABORATION WITH) A NEPHROLOGIST
Effective co-management with a nephrologist is essential for the overall health of the patient with chronic kidney disease, as well as slowing the progression to end-stage renal disease. Exactly when and to what extent the care of a patient with chronic kidney disease should be transferred to a nephrologist depends largely on the individual nephrologist and the comfort level of the primary care provider. When the referral does occur, effective communication between providers and a mutual understanding of the goals of care (eg, the blood pressure target) are essential to optimize patient care.
References
National Kidney Foundation. K/DOQI clinical practice guidelines for chronic kidney disease: evaluation, classification, and stratification. Am J Kidney Dis2002; 39(2 suppl 1):S1–S266.
Levey AS, Bosch JP, Lewis JB, Greene T, Rogers N, Roth D. A more accurate method to estimate glomerular filtration rate from serum creatinine: a new prediction equation. Modification of Diet in Renal Disease Study Group. Ann Intern Med1999; 130:461–470.
Levey AS, Coresh J, Greene T, et al; Chronic Kidney Disease Epidemiology Collaboration. Using standardized serum creatinine values in the Modification of Diet in Renal Disease study equation for estimating glomerular filtration rate. Ann Intern Med2006; 145:247–254.
Levey AS. Measurement of renal function in chronic renal disease. Kidney Int1990; 38:167–184.
Toto RD. Conventional measurement of renal function utilizing serum creatinine, creatinine clearance, inulin and para-aminohippuric acid clearance. Curr Opin Nephrol Hypertens1995; 4:505–509.
Hilbrands LB, Artz MA, Wetzels JF, Koene RA. Cimetidine improves the reliability of creatinine as a marker of glomerular filtration. Kidney Int1991; 40:1171–1176.
Hottelart C, El Esper N, Rose F, Achard JM, Fournier A. Fenofibrate increases creatininemia by increasing metabolic production of creatinine. Nephron2002; 92:536–541.
Rolin HA, Hall PM. Evaluation of glomerular filtration rate and renal plasma flow. In:Jacobson HR, Striker GE, Klahr S, editors. The Principles and Practice of Nephrology, 2nd ed. St. Louis: Mosby-Year Book, Inc., 1995:8–13.
Cockcroft DW, Gault MH. Prediction of creatinine clearance from serum creatinine. Nephron1976; 16:31–41.
Stevens LA, Coresh J, Feldman HI, et al. Evaluation of the Modification of Diet in Renal Disease study equation in a large diverse population. J Am Soc Nephrol2007; 18:2749–2757.
Levey AS, Stevens LA, Schmid CH, et al; CKD-EPI (Chronic Kidney Disease Epidemiology Collaboration). A new equation to estimate glomerular filtration rate. Ann Intern Med2009; 150:604–612.
Vassalotti JA, Stevens LA, Levey AS. Testing for chronic kidney disease: a position statement from the National Kidney Foundation. Am J Kidney Dis2007; 50:169–180.
Hemmelgarn BR, Manns BJ, Lloyd A, et al; Alberta Kidney Disease Network. Relation between kidney function, proteinuria, and adverse outcomes. JAMA2010; 303:423–429.
Glassock RJ, Winearls C. Screening for CKD with eGFR: doubts and dangers. Clin J Am Soc Nephrol2008; 3:1563–1568.
Poggio ED, Rule AD. A critical evaluation of chronic kidney disease—should isolated reduced estimated glomerular filtration rate be considered a ‘disease’?Nephrol Dial Transplant2009; 24:698–700.
Go AS, Chertow GM, Fan D, McCulloch CE, Hsu CY. Chronic kidney disease and the risks of death, cardiovascular events, and hospitalization. N Engl J Med2004; 351:1296–1305.
Glassock RJ. Referrals for chronic kidney disease: real problem or nuisance?JAMA2010; 303:1201–1203.
Tonelli M, Muntner P, Lloyd A, et al; for the Alberta Kidney Disease Network. Using proteinuria and estimated glomerular filtration rate to classify risk in patients with chronic kidney disease. A cohort study. Ann Intern Med2011; 154:12–21.
Lindeman RD, Tobin JD, Shock NW. Association between blood pressure and the rate of decline in renal function with age. Kidney Int1984; 26:861–868.
James Simon, MD Department of Nephrology and Hypertension, Glickman Urological and Kidney Institute, Department of General Internal Medicine, Cleveland Clinic
Milen Amde, MD Department of Nephrology and Hypertension, Glickman Urological and Kidney Institute, Department of General Internal Medicine, Cleveland Clinic
Emilio D. Poggio, MD Department of Nephrology and Hypertension, Glickman Urological and Kidney Institute, Department of General Internal Medicine, Cleveland Clinic
Address: James Simon, MD, Department of Nephrology and Hypertension, Glickman Urological and Kidney Institute, Q7, Cleveland Clinic, 9500 Euclid Avenue, Cleveland, OH 44195; e-mail simonj2@ccf.org
James Simon, MD Department of Nephrology and Hypertension, Glickman Urological and Kidney Institute, Department of General Internal Medicine, Cleveland Clinic
Milen Amde, MD Department of Nephrology and Hypertension, Glickman Urological and Kidney Institute, Department of General Internal Medicine, Cleveland Clinic
Emilio D. Poggio, MD Department of Nephrology and Hypertension, Glickman Urological and Kidney Institute, Department of General Internal Medicine, Cleveland Clinic
Address: James Simon, MD, Department of Nephrology and Hypertension, Glickman Urological and Kidney Institute, Q7, Cleveland Clinic, 9500 Euclid Avenue, Cleveland, OH 44195; e-mail simonj2@ccf.org
Author and Disclosure Information
James Simon, MD Department of Nephrology and Hypertension, Glickman Urological and Kidney Institute, Department of General Internal Medicine, Cleveland Clinic
Milen Amde, MD Department of Nephrology and Hypertension, Glickman Urological and Kidney Institute, Department of General Internal Medicine, Cleveland Clinic
Emilio D. Poggio, MD Department of Nephrology and Hypertension, Glickman Urological and Kidney Institute, Department of General Internal Medicine, Cleveland Clinic
Address: James Simon, MD, Department of Nephrology and Hypertension, Glickman Urological and Kidney Institute, Q7, Cleveland Clinic, 9500 Euclid Avenue, Cleveland, OH 44195; e-mail simonj2@ccf.org
Chronic kidney disease is most often discovered and diagnosed by primary care providers. The equations for estimating the glomerular filtration rate (GFR) facilitate earlier detection of this disease. However, the estimated GFR must be interpreted in the context of the individual patient. The diagnostic criteria and staging of chronic kidney disease must be understood so that it can be recognized and managed at the earliest possible stage. In this way, primary care physicians and nephrologists can better coordinate the care of these patients.
THE STAGES OF RENAL DISEASE AND THE GFR
Before 2002, an organized approach to the clinical management of patients with renal dysfunction was hampered by a lack of a standardized way to define this condition. This changed when the National Kidney Foundation, through the Kidney Disease Outcomes Quality Initiative (K/DOQI),1 defined the stages of chronic kidney disease based on the GFR as estimated by the Modification of Diet in Renal Disease (MDRD) equation.2,3
This system has increased the recognition of chronic kidney disease by the health care community and the general public. But the entire system hinges on the utility, accuracy, and reliability of the equations used to estimate the GFR.
In this article, we review the concepts of renal clearance and how to interpret the GFR in healthy patients and in those with chronic kidney disease. The following cases illustrate the interpretation of GFR in the context of patient care.
CASE 1: A 60-YEAR-OLD WOMAN WITH A ‘NORMAL’ CREATININE LEVEL
A 60-year-old white woman with no significant medical history has routine laboratory tests done as part of her annual physical examination. She weighs 135 pounds (61.2 kg) and is 64 inches (163 cm) tall. Her serum creatinine level is 1.1 mg/dL; her estimated GFR is 53 mL/min/1.73 m2. A urine dipstick test for protein and blood is normal.
CASE 2: PROTEINURIA WITH A PRESERVED GFR
A 20-year-old African American man with no medical history is undergoing routine blood testing. His serum creatinine level is 1.1 mg/dL; his estimated GFR is reported as “> 60 mL/min/1.73 m2” (calculated at 109 mL/min/1.73 m2). He is 72 inches (183 cm) tall and weighs 180 pounds (83.0 kg); he lifts weights four times a week. Urine dipstick testing reveals 3+ proteinuria.
SERUM CREATININE: AN IMPERFECT MARKER OF KIDNEY FUNCTION
Of the various functions of the kidney, the ability of the glomeruli to filter the blood, as assessed by the GFR, is considered the best index of overall kidney function.4,5 The GFR can be thought of as the clearance of a substance from the plasma by the kidney in a period of time. This is useful because no method is available to routinely and directly measure filtration across the glomerular basement membrane.
Substances that are cleared by the kidney are used to estimate the GFR. The ideal substance for this estimate is one that is cleared only by filtration and not through metabolism or excretion by other means.
The urinary clearance of the exogenous substance inulin is considered the gold standard method, but radioisotopes such as iothalamate and other markers have replaced inulin in clinical laboratories. Because these methods are expensive, time-consuming, and not widely available, alternative methods that use endogenous markers such as creatinine have been developed for clinical practice.
The serum creatinine concentration possesses many of the qualities of an ideal marker for estimating kidney function. Creatinine is produced by the body at a relatively constant rate under normal conditions and is easy and inexpensive to measure. However, it has several limitations:
Data presented in Rolin HA III, et al. Evaluation of glomerular filtration rate and renal plasma flow. In: Jacobson HR, et al, eds. The Principles and Practice of Nephrology. St. Louis: Mosby-Year Book 1995:8-13.
Figure 1. The relationship between serum creatinine concentration, creatinine clearance, and glomerular filtration rate (GFR), shown with a 95% confidence interval (blue band). Points A and B illustrate the large change in GFR that results from a small change in serum creatinine at higher levels of kidney function. Points C and D illustrate the small change in GFR that results from a large change in serum creatinine at lower levels of kidney function. Creatinine clearance tends to overestimate the GFR.Its clearance does not solely reflect glomerular filtration because the renal tubules also excrete it into the urine.6 As a result, creatinine clearance (see below) will tend to overestimate the GFR (Figure 1).
The serum creatinine concentration is directly dependent on muscle mass, which varies with sex (women tend to have less muscle mass as a percent of body weight than men), age (muscle mass decreases with age), and race (African Americans have a higher serum creatinine level for the same GFR than other Americans).6 Thus, there is no “normal” value for serum creatinine that applies to all patients.
Other factors can alter the creatinine level without changing the GFR, such as changes in dietary protein intake, exercise, and drugs such as cimetidine7 and fibrates8 (Table 1).
Another important point is that the relationship between the serum creatinine concentration and the GFR is parabolic.9 At high kidney function, large changes in the GFR are reflected by very small changes in serum creatinine—the GFR must fall quite a bit before the serum creatinine level rises very much (points A to B in Figure 1). At lower kidney function, small changes in GFR are reflected by large changes in serum creatinine (points C to D in Figure 1). This phenomenon can cause physicians to view small changes in creatinine as unimportant in patients with creatinine levels in the normal or near-normal range. Conversely, small changes may be due to random error inherent in the methods of measuring creatinine rather than to changes in kidney function.
Because the serum creatinine concentration by itself may be misleading when estimating GFR, the National Kidney Foundation and the National Kidney Disease Education Program recommend that it not be used on its own to estimate kidney function.
ESTIMATING THE GFR
Measuring 24-hour creatinine clearance
Measuring 24-hour creatinine clearance involves measuring the concentrations of creatinine in the serum and the urine and the volume of urine excreted in 24 hours.
The 24-hour creatinine clearance was long considered the best alternative to the serum creatinine concentration for assessing kidney function, as it adjusts for changes in the creatinine concentration by taking into account creatinine’s excretion in the urine. However, 24-hour urine collection is burdensome for the patient, and the results are not always reliable because of variations in collection technique. Also, using the creatinine clearance does not resolve problems with using the serum creatinine concentration, such as tubular secretion and overestimation of GFR.
In an effort to more easily estimate GFR from blood tests alone, efforts to develop mathematical equations that more closely estimate GFR began over 40 years ago. These equations take into account factors such as age, sex, and ethnicity. The best known of these are the Cockcroft and Gault10 and the MDRD equations.5
The Cockcroft-Gault equation
The Cockcroft-Gault equation is fairly simple, using serum creatinine, ideal body weight, and an adjustment factor for sex. Its main drawbacks are that it was developed to model creatinine clearance, itself an imperfect estimation of GFR, and it depends heavily on the accuracy of the value for “lean” body weight used in the equation.
The MDRD equation
The MDRD equation has now largely replaced the Cockcroft-Gault equation. Developed using iothalamate GFR measurements, it therefore estimates GFR rather than the less-accurate creatinine clearance. Also, it is normalized to a standard body surface area (1.73 m2), obviating the need to determine ideal body weight.
Since the estimated GFR can often be calculated using data available in most electronic medical record systems, it can be reported directly with any laboratory report that includes a serum creatinine value.
The main drawback of the MDRD equation is that it tends to underestimate GFR at higher ranges of kidney function, ie, higher than 60 mL/min/1.73 m2).3,11
The CKD-EPI equation
The Chronic Kidney Disease Epidemiology Collaboration study (CKD-EPI) equation,12 published in 2009, is expected to eventually replace the currently used MDRD equation, as it performs better at higher ranges of GFR.
Although the CKD-EPI equation still lacks precision and accuracy, it underestimates GFR to a lesser degree than the MDRD equation in patients with preserved renal function. Also, it was developed with the objective of reporting a specific value even when the estimated GFR is greater than 60 mL/min/1.73 m2. (In contrast, when laboratories use the MDRD equation, the recommendation is to report any value above this level as “greater than 60 mL/min/1.73 m2”).
A limitation of all equations that use the serum creatinine concentration to assess kidney function is the assumption that creatinine production is both stable over time and similar among patients. As a result, these equations should not be used in situations in which renal function is changing rapidly, such as in acute kidney injury. Also, they should be used with caution in patients at the extremes of body mass, since they underestimate GFR in very muscular patients (eg, as in case 2) and overestimate GFR in very small patients (eg, as in case 1).
As most patients with established medical problems have blood drawn periodically for routine chemistry panels, the diagnosis of chronic kidney disease often occurs through routine testing. For patients who do not yet carry this diagnosis, it is important to recognize the risk factors for chronic kidney disease (Table 2) and to determine who should be screened.
In general, anyone at higher risk of chronic kidney disease should be screened for it. This group includes US minorities and patients with hypertension, cardiovascular disease, and diabetes mellitus, among others.13 Screening includes an assessment of estimated GFR and urinalysis for proteinuria or hematuria.
CHRONIC KIDNEY DISEASE DEFINED: DAMAGE AND DURATION
The definition of chronic kidney disease contains two components—kidney damage and duration (Table 3).1
The kidney damage can be either parenchymal renal damage independent of GFR (for example, cystic disease, glomerular hematuria, or proteinuria) or depressed GFR independent of evidence of parenchymal renal disease (an estimated GFR of less than 60 mL/min/1.73 m2).
The duration component requires that the abnormality be present for at least 3 months (ie, chronic).
Concerns about the definition
This definition has not been without controversy.
An unintended consequence of not reporting estimated GFR values above 60 mL/min/1.73 m2 in absolute numbers is that providers may ignore changes in serum creatinine at estimated GFRs in this range, as they assume that the kidney function is “normal.” This may change in the future if the CKD-EPI equation is used, which produces less bias at slightly higher GFRs.
Providers may also tend to focus solely on the estimated GFR criterion and ignore other evidence of chronic kidney disease, such as abnormalities in urinalysis or imaging studies. For example, proteinuria has been shown to be more important than absolute GFR values in predicting progression of renal dysfunction and cardiovascular risk.14 Proteinuria, especially in the setting of an estimated GFR above 60 mL/min/1.73 m2, can be missed if not screened for and underappreciated once found.
Moreover, in elderly patients, the current GFR equations underperform at borderline GFR values and can yield depressed values even at impressively “normal” serum creatinine levels. As a result, there is concern that chronic kidney disease is being overdiagnosed under the current system. This is especially worrisome in elderly white women without risk factors for chronic kidney disease (eg, as in case 1).
In addition, the question arises whether the arbitrary cutoff for chronic kidney disease— 60 mL/min/1.73 m2—applies to all populations.15,16 The utility of classifying someone as having chronic kidney disease who has an estimated GFR of 55 mL/min/1.73 m2 and no risk factors for chronic kidney disease (Table 2) should be questioned if the risk of progressing to end-stage renal disease or suffering a cardiovascular event is only minimally higher than in patients with a higher estimated GFR.6,17 If the true purpose of developing the chronic kidney disease classification system is to improve patient care and outcomes, then it is of no benefit to overclassify such patients. Indeed, the stress induced by the diagnosis and the negative implications on insurance coverage and health care costs may outweigh any benefits.18
Nevertheless, these concerns do not invalidate the entire chronic kidney disease definition system, but have stimulated current efforts to improve it based on outcomes research.19
CASES REVISITED
Case 1: Problems with estimating GFR in a small woman
Case 1 has several points to note.
The patient’s small body size reflects low-level creatinine production. It is not atypical to find serum creatinine levels of 0.5 mg/dL in such patients. Thus, her serum creatinine level of 1.1 mg/dL may be abnormal. The fact that the MDRD equation “normalizes” the result to 1.73 m2 of body surface area in patients with very low muscle mass will lead to an overestimation of GFR. However, she has no risk factors for chronic kidney disease.
Additionally, in up to two-thirds of patients kidney function declines with age.20 Whether or not this is “normal aging” of the kidney, it is not clear that this decline in GFR reflects an underlying pathologic process.
Finally, since the patient is an older white woman, the estimated GFR tends to underestimate the true GFR. So while her body size may predispose to an overestimation of GFR, her age, race, and sex predispose to an underestimation of GFR. Many nephrologists would simply order urinalysis and ultrasonography to rule out other evidence of renal dysfunction, then recommend routine monitoring of kidney function in this case.
Case 2: Proteinuria is not normal
In case 2, because the patient is African American, young, and male, his creatinine level yields a higher estimated GFR than in case 1, despite having the same value. However, his estimated GFR still underestimates his true GFR because of his greater creatinine production due to his muscular physique.
This patient subsequently underwent iothalamate GFR testing, which yielded a GFR of 115 mL/min/1.73 m2. However, he has dipstick-positive proteinuria, which, if confirmed on further testing, would meet the criteria for chronic kidney disease and put him at a higher risk of cardiovascular events and progression to lower kidney function than the patient in case 1. He also needs to be screened for undiagnosed hypertension and underlying glomerular disease.
REFERRAL TO (AND COLLABORATION WITH) A NEPHROLOGIST
Effective co-management with a nephrologist is essential for the overall health of the patient with chronic kidney disease, as well as slowing the progression to end-stage renal disease. Exactly when and to what extent the care of a patient with chronic kidney disease should be transferred to a nephrologist depends largely on the individual nephrologist and the comfort level of the primary care provider. When the referral does occur, effective communication between providers and a mutual understanding of the goals of care (eg, the blood pressure target) are essential to optimize patient care.
Chronic kidney disease is most often discovered and diagnosed by primary care providers. The equations for estimating the glomerular filtration rate (GFR) facilitate earlier detection of this disease. However, the estimated GFR must be interpreted in the context of the individual patient. The diagnostic criteria and staging of chronic kidney disease must be understood so that it can be recognized and managed at the earliest possible stage. In this way, primary care physicians and nephrologists can better coordinate the care of these patients.
THE STAGES OF RENAL DISEASE AND THE GFR
Before 2002, an organized approach to the clinical management of patients with renal dysfunction was hampered by a lack of a standardized way to define this condition. This changed when the National Kidney Foundation, through the Kidney Disease Outcomes Quality Initiative (K/DOQI),1 defined the stages of chronic kidney disease based on the GFR as estimated by the Modification of Diet in Renal Disease (MDRD) equation.2,3
This system has increased the recognition of chronic kidney disease by the health care community and the general public. But the entire system hinges on the utility, accuracy, and reliability of the equations used to estimate the GFR.
In this article, we review the concepts of renal clearance and how to interpret the GFR in healthy patients and in those with chronic kidney disease. The following cases illustrate the interpretation of GFR in the context of patient care.
CASE 1: A 60-YEAR-OLD WOMAN WITH A ‘NORMAL’ CREATININE LEVEL
A 60-year-old white woman with no significant medical history has routine laboratory tests done as part of her annual physical examination. She weighs 135 pounds (61.2 kg) and is 64 inches (163 cm) tall. Her serum creatinine level is 1.1 mg/dL; her estimated GFR is 53 mL/min/1.73 m2. A urine dipstick test for protein and blood is normal.
CASE 2: PROTEINURIA WITH A PRESERVED GFR
A 20-year-old African American man with no medical history is undergoing routine blood testing. His serum creatinine level is 1.1 mg/dL; his estimated GFR is reported as “> 60 mL/min/1.73 m2” (calculated at 109 mL/min/1.73 m2). He is 72 inches (183 cm) tall and weighs 180 pounds (83.0 kg); he lifts weights four times a week. Urine dipstick testing reveals 3+ proteinuria.
SERUM CREATININE: AN IMPERFECT MARKER OF KIDNEY FUNCTION
Of the various functions of the kidney, the ability of the glomeruli to filter the blood, as assessed by the GFR, is considered the best index of overall kidney function.4,5 The GFR can be thought of as the clearance of a substance from the plasma by the kidney in a period of time. This is useful because no method is available to routinely and directly measure filtration across the glomerular basement membrane.
Substances that are cleared by the kidney are used to estimate the GFR. The ideal substance for this estimate is one that is cleared only by filtration and not through metabolism or excretion by other means.
The urinary clearance of the exogenous substance inulin is considered the gold standard method, but radioisotopes such as iothalamate and other markers have replaced inulin in clinical laboratories. Because these methods are expensive, time-consuming, and not widely available, alternative methods that use endogenous markers such as creatinine have been developed for clinical practice.
The serum creatinine concentration possesses many of the qualities of an ideal marker for estimating kidney function. Creatinine is produced by the body at a relatively constant rate under normal conditions and is easy and inexpensive to measure. However, it has several limitations:
Data presented in Rolin HA III, et al. Evaluation of glomerular filtration rate and renal plasma flow. In: Jacobson HR, et al, eds. The Principles and Practice of Nephrology. St. Louis: Mosby-Year Book 1995:8-13.
Figure 1. The relationship between serum creatinine concentration, creatinine clearance, and glomerular filtration rate (GFR), shown with a 95% confidence interval (blue band). Points A and B illustrate the large change in GFR that results from a small change in serum creatinine at higher levels of kidney function. Points C and D illustrate the small change in GFR that results from a large change in serum creatinine at lower levels of kidney function. Creatinine clearance tends to overestimate the GFR.Its clearance does not solely reflect glomerular filtration because the renal tubules also excrete it into the urine.6 As a result, creatinine clearance (see below) will tend to overestimate the GFR (Figure 1).
The serum creatinine concentration is directly dependent on muscle mass, which varies with sex (women tend to have less muscle mass as a percent of body weight than men), age (muscle mass decreases with age), and race (African Americans have a higher serum creatinine level for the same GFR than other Americans).6 Thus, there is no “normal” value for serum creatinine that applies to all patients.
Other factors can alter the creatinine level without changing the GFR, such as changes in dietary protein intake, exercise, and drugs such as cimetidine7 and fibrates8 (Table 1).
Another important point is that the relationship between the serum creatinine concentration and the GFR is parabolic.9 At high kidney function, large changes in the GFR are reflected by very small changes in serum creatinine—the GFR must fall quite a bit before the serum creatinine level rises very much (points A to B in Figure 1). At lower kidney function, small changes in GFR are reflected by large changes in serum creatinine (points C to D in Figure 1). This phenomenon can cause physicians to view small changes in creatinine as unimportant in patients with creatinine levels in the normal or near-normal range. Conversely, small changes may be due to random error inherent in the methods of measuring creatinine rather than to changes in kidney function.
Because the serum creatinine concentration by itself may be misleading when estimating GFR, the National Kidney Foundation and the National Kidney Disease Education Program recommend that it not be used on its own to estimate kidney function.
ESTIMATING THE GFR
Measuring 24-hour creatinine clearance
Measuring 24-hour creatinine clearance involves measuring the concentrations of creatinine in the serum and the urine and the volume of urine excreted in 24 hours.
The 24-hour creatinine clearance was long considered the best alternative to the serum creatinine concentration for assessing kidney function, as it adjusts for changes in the creatinine concentration by taking into account creatinine’s excretion in the urine. However, 24-hour urine collection is burdensome for the patient, and the results are not always reliable because of variations in collection technique. Also, using the creatinine clearance does not resolve problems with using the serum creatinine concentration, such as tubular secretion and overestimation of GFR.
In an effort to more easily estimate GFR from blood tests alone, efforts to develop mathematical equations that more closely estimate GFR began over 40 years ago. These equations take into account factors such as age, sex, and ethnicity. The best known of these are the Cockcroft and Gault10 and the MDRD equations.5
The Cockcroft-Gault equation
The Cockcroft-Gault equation is fairly simple, using serum creatinine, ideal body weight, and an adjustment factor for sex. Its main drawbacks are that it was developed to model creatinine clearance, itself an imperfect estimation of GFR, and it depends heavily on the accuracy of the value for “lean” body weight used in the equation.
The MDRD equation
The MDRD equation has now largely replaced the Cockcroft-Gault equation. Developed using iothalamate GFR measurements, it therefore estimates GFR rather than the less-accurate creatinine clearance. Also, it is normalized to a standard body surface area (1.73 m2), obviating the need to determine ideal body weight.
Since the estimated GFR can often be calculated using data available in most electronic medical record systems, it can be reported directly with any laboratory report that includes a serum creatinine value.
The main drawback of the MDRD equation is that it tends to underestimate GFR at higher ranges of kidney function, ie, higher than 60 mL/min/1.73 m2).3,11
The CKD-EPI equation
The Chronic Kidney Disease Epidemiology Collaboration study (CKD-EPI) equation,12 published in 2009, is expected to eventually replace the currently used MDRD equation, as it performs better at higher ranges of GFR.
Although the CKD-EPI equation still lacks precision and accuracy, it underestimates GFR to a lesser degree than the MDRD equation in patients with preserved renal function. Also, it was developed with the objective of reporting a specific value even when the estimated GFR is greater than 60 mL/min/1.73 m2. (In contrast, when laboratories use the MDRD equation, the recommendation is to report any value above this level as “greater than 60 mL/min/1.73 m2”).
A limitation of all equations that use the serum creatinine concentration to assess kidney function is the assumption that creatinine production is both stable over time and similar among patients. As a result, these equations should not be used in situations in which renal function is changing rapidly, such as in acute kidney injury. Also, they should be used with caution in patients at the extremes of body mass, since they underestimate GFR in very muscular patients (eg, as in case 2) and overestimate GFR in very small patients (eg, as in case 1).
As most patients with established medical problems have blood drawn periodically for routine chemistry panels, the diagnosis of chronic kidney disease often occurs through routine testing. For patients who do not yet carry this diagnosis, it is important to recognize the risk factors for chronic kidney disease (Table 2) and to determine who should be screened.
In general, anyone at higher risk of chronic kidney disease should be screened for it. This group includes US minorities and patients with hypertension, cardiovascular disease, and diabetes mellitus, among others.13 Screening includes an assessment of estimated GFR and urinalysis for proteinuria or hematuria.
CHRONIC KIDNEY DISEASE DEFINED: DAMAGE AND DURATION
The definition of chronic kidney disease contains two components—kidney damage and duration (Table 3).1
The kidney damage can be either parenchymal renal damage independent of GFR (for example, cystic disease, glomerular hematuria, or proteinuria) or depressed GFR independent of evidence of parenchymal renal disease (an estimated GFR of less than 60 mL/min/1.73 m2).
The duration component requires that the abnormality be present for at least 3 months (ie, chronic).
Concerns about the definition
This definition has not been without controversy.
An unintended consequence of not reporting estimated GFR values above 60 mL/min/1.73 m2 in absolute numbers is that providers may ignore changes in serum creatinine at estimated GFRs in this range, as they assume that the kidney function is “normal.” This may change in the future if the CKD-EPI equation is used, which produces less bias at slightly higher GFRs.
Providers may also tend to focus solely on the estimated GFR criterion and ignore other evidence of chronic kidney disease, such as abnormalities in urinalysis or imaging studies. For example, proteinuria has been shown to be more important than absolute GFR values in predicting progression of renal dysfunction and cardiovascular risk.14 Proteinuria, especially in the setting of an estimated GFR above 60 mL/min/1.73 m2, can be missed if not screened for and underappreciated once found.
Moreover, in elderly patients, the current GFR equations underperform at borderline GFR values and can yield depressed values even at impressively “normal” serum creatinine levels. As a result, there is concern that chronic kidney disease is being overdiagnosed under the current system. This is especially worrisome in elderly white women without risk factors for chronic kidney disease (eg, as in case 1).
In addition, the question arises whether the arbitrary cutoff for chronic kidney disease— 60 mL/min/1.73 m2—applies to all populations.15,16 The utility of classifying someone as having chronic kidney disease who has an estimated GFR of 55 mL/min/1.73 m2 and no risk factors for chronic kidney disease (Table 2) should be questioned if the risk of progressing to end-stage renal disease or suffering a cardiovascular event is only minimally higher than in patients with a higher estimated GFR.6,17 If the true purpose of developing the chronic kidney disease classification system is to improve patient care and outcomes, then it is of no benefit to overclassify such patients. Indeed, the stress induced by the diagnosis and the negative implications on insurance coverage and health care costs may outweigh any benefits.18
Nevertheless, these concerns do not invalidate the entire chronic kidney disease definition system, but have stimulated current efforts to improve it based on outcomes research.19
CASES REVISITED
Case 1: Problems with estimating GFR in a small woman
Case 1 has several points to note.
The patient’s small body size reflects low-level creatinine production. It is not atypical to find serum creatinine levels of 0.5 mg/dL in such patients. Thus, her serum creatinine level of 1.1 mg/dL may be abnormal. The fact that the MDRD equation “normalizes” the result to 1.73 m2 of body surface area in patients with very low muscle mass will lead to an overestimation of GFR. However, she has no risk factors for chronic kidney disease.
Additionally, in up to two-thirds of patients kidney function declines with age.20 Whether or not this is “normal aging” of the kidney, it is not clear that this decline in GFR reflects an underlying pathologic process.
Finally, since the patient is an older white woman, the estimated GFR tends to underestimate the true GFR. So while her body size may predispose to an overestimation of GFR, her age, race, and sex predispose to an underestimation of GFR. Many nephrologists would simply order urinalysis and ultrasonography to rule out other evidence of renal dysfunction, then recommend routine monitoring of kidney function in this case.
Case 2: Proteinuria is not normal
In case 2, because the patient is African American, young, and male, his creatinine level yields a higher estimated GFR than in case 1, despite having the same value. However, his estimated GFR still underestimates his true GFR because of his greater creatinine production due to his muscular physique.
This patient subsequently underwent iothalamate GFR testing, which yielded a GFR of 115 mL/min/1.73 m2. However, he has dipstick-positive proteinuria, which, if confirmed on further testing, would meet the criteria for chronic kidney disease and put him at a higher risk of cardiovascular events and progression to lower kidney function than the patient in case 1. He also needs to be screened for undiagnosed hypertension and underlying glomerular disease.
REFERRAL TO (AND COLLABORATION WITH) A NEPHROLOGIST
Effective co-management with a nephrologist is essential for the overall health of the patient with chronic kidney disease, as well as slowing the progression to end-stage renal disease. Exactly when and to what extent the care of a patient with chronic kidney disease should be transferred to a nephrologist depends largely on the individual nephrologist and the comfort level of the primary care provider. When the referral does occur, effective communication between providers and a mutual understanding of the goals of care (eg, the blood pressure target) are essential to optimize patient care.
References
National Kidney Foundation. K/DOQI clinical practice guidelines for chronic kidney disease: evaluation, classification, and stratification. Am J Kidney Dis2002; 39(2 suppl 1):S1–S266.
Levey AS, Bosch JP, Lewis JB, Greene T, Rogers N, Roth D. A more accurate method to estimate glomerular filtration rate from serum creatinine: a new prediction equation. Modification of Diet in Renal Disease Study Group. Ann Intern Med1999; 130:461–470.
Levey AS, Coresh J, Greene T, et al; Chronic Kidney Disease Epidemiology Collaboration. Using standardized serum creatinine values in the Modification of Diet in Renal Disease study equation for estimating glomerular filtration rate. Ann Intern Med2006; 145:247–254.
Levey AS. Measurement of renal function in chronic renal disease. Kidney Int1990; 38:167–184.
Toto RD. Conventional measurement of renal function utilizing serum creatinine, creatinine clearance, inulin and para-aminohippuric acid clearance. Curr Opin Nephrol Hypertens1995; 4:505–509.
Hilbrands LB, Artz MA, Wetzels JF, Koene RA. Cimetidine improves the reliability of creatinine as a marker of glomerular filtration. Kidney Int1991; 40:1171–1176.
Hottelart C, El Esper N, Rose F, Achard JM, Fournier A. Fenofibrate increases creatininemia by increasing metabolic production of creatinine. Nephron2002; 92:536–541.
Rolin HA, Hall PM. Evaluation of glomerular filtration rate and renal plasma flow. In:Jacobson HR, Striker GE, Klahr S, editors. The Principles and Practice of Nephrology, 2nd ed. St. Louis: Mosby-Year Book, Inc., 1995:8–13.
Cockcroft DW, Gault MH. Prediction of creatinine clearance from serum creatinine. Nephron1976; 16:31–41.
Stevens LA, Coresh J, Feldman HI, et al. Evaluation of the Modification of Diet in Renal Disease study equation in a large diverse population. J Am Soc Nephrol2007; 18:2749–2757.
Levey AS, Stevens LA, Schmid CH, et al; CKD-EPI (Chronic Kidney Disease Epidemiology Collaboration). A new equation to estimate glomerular filtration rate. Ann Intern Med2009; 150:604–612.
Vassalotti JA, Stevens LA, Levey AS. Testing for chronic kidney disease: a position statement from the National Kidney Foundation. Am J Kidney Dis2007; 50:169–180.
Hemmelgarn BR, Manns BJ, Lloyd A, et al; Alberta Kidney Disease Network. Relation between kidney function, proteinuria, and adverse outcomes. JAMA2010; 303:423–429.
Glassock RJ, Winearls C. Screening for CKD with eGFR: doubts and dangers. Clin J Am Soc Nephrol2008; 3:1563–1568.
Poggio ED, Rule AD. A critical evaluation of chronic kidney disease—should isolated reduced estimated glomerular filtration rate be considered a ‘disease’?Nephrol Dial Transplant2009; 24:698–700.
Go AS, Chertow GM, Fan D, McCulloch CE, Hsu CY. Chronic kidney disease and the risks of death, cardiovascular events, and hospitalization. N Engl J Med2004; 351:1296–1305.
Glassock RJ. Referrals for chronic kidney disease: real problem or nuisance?JAMA2010; 303:1201–1203.
Tonelli M, Muntner P, Lloyd A, et al; for the Alberta Kidney Disease Network. Using proteinuria and estimated glomerular filtration rate to classify risk in patients with chronic kidney disease. A cohort study. Ann Intern Med2011; 154:12–21.
Lindeman RD, Tobin JD, Shock NW. Association between blood pressure and the rate of decline in renal function with age. Kidney Int1984; 26:861–868.
References
National Kidney Foundation. K/DOQI clinical practice guidelines for chronic kidney disease: evaluation, classification, and stratification. Am J Kidney Dis2002; 39(2 suppl 1):S1–S266.
Levey AS, Bosch JP, Lewis JB, Greene T, Rogers N, Roth D. A more accurate method to estimate glomerular filtration rate from serum creatinine: a new prediction equation. Modification of Diet in Renal Disease Study Group. Ann Intern Med1999; 130:461–470.
Levey AS, Coresh J, Greene T, et al; Chronic Kidney Disease Epidemiology Collaboration. Using standardized serum creatinine values in the Modification of Diet in Renal Disease study equation for estimating glomerular filtration rate. Ann Intern Med2006; 145:247–254.
Levey AS. Measurement of renal function in chronic renal disease. Kidney Int1990; 38:167–184.
Toto RD. Conventional measurement of renal function utilizing serum creatinine, creatinine clearance, inulin and para-aminohippuric acid clearance. Curr Opin Nephrol Hypertens1995; 4:505–509.
Hilbrands LB, Artz MA, Wetzels JF, Koene RA. Cimetidine improves the reliability of creatinine as a marker of glomerular filtration. Kidney Int1991; 40:1171–1176.
Hottelart C, El Esper N, Rose F, Achard JM, Fournier A. Fenofibrate increases creatininemia by increasing metabolic production of creatinine. Nephron2002; 92:536–541.
Rolin HA, Hall PM. Evaluation of glomerular filtration rate and renal plasma flow. In:Jacobson HR, Striker GE, Klahr S, editors. The Principles and Practice of Nephrology, 2nd ed. St. Louis: Mosby-Year Book, Inc., 1995:8–13.
Cockcroft DW, Gault MH. Prediction of creatinine clearance from serum creatinine. Nephron1976; 16:31–41.
Stevens LA, Coresh J, Feldman HI, et al. Evaluation of the Modification of Diet in Renal Disease study equation in a large diverse population. J Am Soc Nephrol2007; 18:2749–2757.
Levey AS, Stevens LA, Schmid CH, et al; CKD-EPI (Chronic Kidney Disease Epidemiology Collaboration). A new equation to estimate glomerular filtration rate. Ann Intern Med2009; 150:604–612.
Vassalotti JA, Stevens LA, Levey AS. Testing for chronic kidney disease: a position statement from the National Kidney Foundation. Am J Kidney Dis2007; 50:169–180.
Hemmelgarn BR, Manns BJ, Lloyd A, et al; Alberta Kidney Disease Network. Relation between kidney function, proteinuria, and adverse outcomes. JAMA2010; 303:423–429.
Glassock RJ, Winearls C. Screening for CKD with eGFR: doubts and dangers. Clin J Am Soc Nephrol2008; 3:1563–1568.
Poggio ED, Rule AD. A critical evaluation of chronic kidney disease—should isolated reduced estimated glomerular filtration rate be considered a ‘disease’?Nephrol Dial Transplant2009; 24:698–700.
Go AS, Chertow GM, Fan D, McCulloch CE, Hsu CY. Chronic kidney disease and the risks of death, cardiovascular events, and hospitalization. N Engl J Med2004; 351:1296–1305.
Glassock RJ. Referrals for chronic kidney disease: real problem or nuisance?JAMA2010; 303:1201–1203.
Tonelli M, Muntner P, Lloyd A, et al; for the Alberta Kidney Disease Network. Using proteinuria and estimated glomerular filtration rate to classify risk in patients with chronic kidney disease. A cohort study. Ann Intern Med2011; 154:12–21.
Lindeman RD, Tobin JD, Shock NW. Association between blood pressure and the rate of decline in renal function with age. Kidney Int1984; 26:861–868.
Chronic kidney disease must be detected in its early stages so that measures can be taken to detect its complications and to delay its progression to kidney failure.
The creatinine concentration is an imperfect marker of renal function and should not be used by itself in assessing renal function.
Formulas for estimating the GFR from the serum creatinine level along with other easily obtained variables continue to be refined.
Primary care physicians and nephrologists need to collaborate to provide the optimal care for patients with chronic kidney disease.
Dronedarone (Multaq), approved by the US Food and Drug Administration in July 2009, is a congener of the antiarrhythmic drug amiodarone (Cordarone). Designed in the hope that it would be safer than amiodarone, its official indication is to lower the risk of hospitalization in patients with paroxysmal or persistent atrial fibrillation or atrial flutter. However, its precise role in the management of atrial fibrillation is yet to be defined. If dronedarone remains well tolerated, it may permit clinicians to pursue a rhythm control strategy more often. In this article, we present a progress report on this new agent.
BETTER ANTIARRHYTHMIC DRUGS ARE NEEDED
Atrial fibrillation increases the risk of stroke fivefold and accounts for 15% to 20% of all strokes.1 It also increases the risk of heart failure. Drugs are the mainstay of therapy, but many antiarrhythmic drugs are not very effective and cause cardiac and extracardiac toxicity. Thus, the need for safe and effective new drugs.2
Much effort is going into the development of drugs that target specific ion channels or proteins expressed predominantly in atrial myocardium. The rationale is to avoid the unwanted effects of ionic currents on the ventricle and thus avoid ventricular proarrhythmic effects. At the same time, alternatives to the multiple channel blocker amiodarone, the mainstay of heart rhythm control therapy in atrial fibrillation, are being developed to retain the electrophysiologic efficacy of the mother compound but avoid its extracardiac toxicity.
RATE CONTROL VS RHYTHM CONTROL
In the acute care setting, heart rate control with atrioventricular nodal agents (beta-blockers, calcium channel blockers, and digitalis) is the preferred initial strategy in most hemodynamically stable patients presenting with new-onset atrial fibrillation.3
Since we lack an effective method for maintaining sinus rhythm without incurring significant adverse effects, rate control is also often chosen for chronic management of atrial fibrillation. This is particularly true for patients who have no symptoms or only minimal symptoms and in whom adequate rate control is easily attained. Indeed, results of large clinical trials suggest that rate control is satisfactory for many patients.
The main purpose of rate control is to control symptoms as opposed to merely lowering the ventricular rate. Effective rate control often prevents hemodynamic instability in patients with underlying heart disease who present acutely with atrial fibrillation. In patients with permanent atrial fibrillation, the RACE II study4 (Rate Control Efficacy in Permanent Atrial Fibrillation: a Comparison between Lenient Versus Strict Rate Control II), during a 3-year follow-up, showed that lenient rate control (resting heart rate < 110 beats per minute) is not inferior to strict rate control (resting heart rate < 80 beats per minute) in preventing major cardiovascular events (heart failure, stroke) or arrhythmic events such as syncope and sustained ventricular tachycardia.4
As a long-term strategy, rate control also prevents tachycardia-induced cardiomyopathy, reduces the risk of worsening of underlying heart failure, and can improve symptoms and quality of life.
Although maintenance of sinus rhythm is most likely associated with a survival benefit, heart rhythm control with antiarrhythmic drugs has not shown an advantage over rate control in overall or cardiovascular death rates, thromboembolic complications, or impact on heart failure. Indeed, a rhythm control strategy has been associated only with better exercise tolerance and, although less clear, with better quality of life.5
One possible explanation as to why a rhythm control strategy has not been shown to be superior to a rate control strategy is the side effects of the presently available drugs for rhythm control.
In a subgroup analysis of the Atrial Fibrillation Follow-up Investigation of Rhythm Management (AFFIRM) trial,6 antiarrhythmic therapy was associated with a 49% increase in the mortality rate that offset the benefits of conversion and maintenance of sinus rhythm, which was associated with a 53% reduction in mortality rates.
The hope is that newer drugs with less toxicity may produce better outcomes for patients treated with rhythm control.
AN ANALOGUE OF AMIODARONE, WITHOUT THE IODINE
Dronedarone is a structurally modified version of amiodarone, the antiarrhythmic drug that has shown the greatest efficacy at maintaining sinus rhythm in patients with paroxysmal atrial fibrillation. Although historically amiodarone has been effective in maintaining sinus rhythm and has been used safely in patients with advanced heart failure, its use has been limited by cumulative and often irreversible extracardiac organ toxicity.
Dronedarone was designed to match amiodarone’s efficacy but with a better safety profile. An iodine radical makes up more than one-third of amiodarone’s molecular weight. The omission of iodine in dronedarone was intended to reduce the likelihood of toxic side effects.
Dronedarone is a benzofuran derivative pharmacologically related to amiodarone, with the addition of a methylsulfonamide group. This reduces lipophilicity and the propensity to cross the blood-brain barrier; over a 2-year period this drug has not been shown to have neurotoxic effects.7
Dronedarone has proved efficacious without toxic or proarrhythmic effects and has minimal side effects, but concerns remain regarding its use in advanced heart failure. To date, its adverse-event profile appears comparable to that of placebo. However, whether its efficacy and incidence of adverse effects are comparable to what has been reported in the literature may take time to assess.
DRONEDARONE’S PHARMACOLOGY
Dronedarone, like amiodarone, blocks multiple sodium and potassium ion channels. It also exerts an antiadrenergic effect by noncompetitive binding to beta-adrenergic receptors as well as by inhibiting an agonist-induced increase in adenylate cyclase activity.8 Compared with amiodarone, dronedarone is a more potent blocker of peak sodium current.
Dronedarone is largely metabolized by the hepatic enzyme cytochrome P450 3A4 isoform (CYP3A4). Only 6% of dronedarone is excreted renally; however, no trial has yet assessed dronedarone’s safety in patients with marked kidney dysfunction.89
Dronedarone’s steady-state terminal elimination half-life is approximately 30 hours. When taken twice a day, it achieves steady-state concentrations in 5 to 7 days.
Dronedarone is available only for oral administration at 400 mg twice daily. Dose adjustment or titration is not recommended.
CLINICAL TRIALS OF DRONEDARONE
Dronedarone vs placebo
ATHENA (A Placebo-Controlled, Double-Blind, Parallel Arm Trial to Assess the Efficacy of Dronedarone 400 mg bid for the Prevention of Cardiovascular Hospitalization or Death From any Cause in Patients With Atrial Fibrillation/Atrial Flutter)10 was a prospective, double-blind study to assess morbidity and death rates in 4,628 patients with atrial fibrillation or atrial flutter and at least one other cardiovascular risk factor.
ATHENA showed that dronedarone, in addition to standard therapy, significantly reduced the risk of a first cardiovascular hospitalization or death by 24% in patients with atrial fibrillation or atrial flutter.9 The study excluded patients with decompensated heart failure(Table 1).
EURIDIS and ADONIS. Two trials,11 EURIDIS (European Trial in Atrial Fibrillation or Flutter Patients Receiving Dronedarone for the Maintenance of Sinus Rhythm) and ADONIS (American-Australian Trial With Dronedarone in Atrial Fibrillation or Flutter Patients for the Maintenance of Sinus Rhythm), enrolled a total of more than 1,200 patients and showed that dronedarone 400 mg twice a day produced a significantly lower rate of recurrence of atrial fibrillation after electrical cardioversion compared with placebo.
Overall, treatment with dronedarone significantly reduced the risk of a first recurrence of atrial fibrillation by 22% (ADONIS) and 27.5% (EURIDIS) (Table 1).
ERATO (Efficacy and Safety of Dronedarone for the Control of Ventricular Rate During Atrial Fibrillation),12 an additional phase III study, showed that dronedarone controlled the heart rate in patients with persistently accelerated ventricular rates despite concomitant standard therapy with a beta-blocker, digitalis, or a calcium-channel blocker. Dronedarone reduced the mean 24-hour heart rate by 11.7 beats per minute and the maximal exercise ventricular rate by 24.5 beats per minute at the 14th day.
ANDROMEDA (Anti-arrhythmic Trial With Dronedarone in Moderate to Severe CHF Evaluating Morbidity Decrease)13 was a study not of patients with atrial fibrillation but rather of patients with symptomatic congestive heart failure, a left ventricular ejection fraction of 35% or less, and recent hospitalization with new or worsening heart failure. The study was terminated early because of a higher rate of death with dronedarone13 (Table 1).
Dronedarone vs amiodarone
DIONYSOS (Efficacy and Safety of Dronedarone Versus Amiodarone for the Maintenance of Sinus Rhythm in Patients With Atrial Fibrillation)14 was a randomized double-blind trial. It evaluated the efficacy and safety of dronedarone (400 mg twice daily) or amiodarone (600 mg daily for 28 days, then 200 mg daily thereafter) for at least 6 months for the maintenance of sinus rhythm in patients with atrial fibrillation. It enrolled 504 patients with persistent atrial fibrillation; patients had not previously taken amiodarone. Dronedarone was less effective than amiodarone in maintaining sinus rhythm: the rate of recurrent atrial fibrillation was 63% with dronedarone and 42% with amiodarone. But dronedarone was associated with fewer adverse effects and less need for premature discontinuation of drug treatment at a mean follow-up of 7 months (Table 1).
WHERE DOES DRONEDARONE FIT IN ATRIAL FIBRILLATION MANAGEMENT?
Dronedarone is indicated in persistent or paroxysmal atrial fibrillation, based on the observed reduction of the rate of hospitalization. It is indicated for the maintenance of sinus rhythm and may be used in patients with persistent or paroxysmal atrial fibrillation and flutter who are in sinus rhythm or will be undergoing cardioversion soon after starting the drug. Dronedarone has no role in the acute management of atrial fibrillation, such as in cardioversion to sinus rhythm in the emergency department.
We do not have substantial evidence of the efficacy of dronedarone in patients with resistant atrial fibrillation, in whom multiple antiarrhythmics have failed to maintain sinus rhythm, and no published trial has used the inclusion criterion of treatment failure with other antiarrhythmic drugs.
The role of dronedarone in heart failure with preserved systolic function is unclear. Patients taking dronedarone are twice as likely as those taking amiodarone to have a recurrence of atrial fibrillation.
The main advantage of dronedarone is its lower adverse effect profile. However, this statement is based on only a few years of observation. If the patient has developed adverse effects with amiodarone, or if the clinician is concerned about the risk of serious adverse effects, dronedarone presents an alternative for those patients without heart failure or significant left ventricular dysfunction. One such group may be younger patients, because of concerns about the cumulative effects of amiodarone taken over a lifetime.
Dronedarone may represent an acceptable alternative to many of the current antiarrhythmic drugs. Based on the results of the Cardiac Arrhythmia Suppression Trial (CAST),15 class IC antiarrhythmics such as flecainide (Tambocor) are generally avoided in patients with prior myocardial infarction or with known or even suspected coronary artery disease. Similarly, sotalol (Betapace) is generally avoided in patients with marked left ventricular hypertrophy because of adverse effects.16 Dofetilide (Tikosyn) and often sotalol require hospitalization with telemetric monitoring for QTc prolongation and the risk of proarrhythmia with torsades de pointes. Dronedarone, however, generally can be safely started in the outpatient setting.
As when considering prescribing any antiarrhythmic, the clinician must assess the patient’s thromboembolic risk, since this risk persists with a rhythm control strategy.
There is substantial evidence from the ATHENA trial,10 in which 30% of the patients had coronary artery disease, that dronedarone is safe and effective in patients with coronary artery disease. Its use in patients who have undergone coronary artery bypass surgery remains to be defined.
WHEN SHOULD WE SWITCH PATIENTS TO DRONEDARONE?
Preliminary experience suggests that dronedarone, unlike most antiarrhythmic drugs, can be safely started about 48 hours after amiodarone is discontinued. Cumulative toxicity has not been noted with dronedarone. Caution should be exercised when switching if the patient has baseline bradycardia or QT interval prolongation. No algorithm has been developed for switching from other antiarrhythmic drugs to dronedarone.
CONTRAINDICATIONS TO DRONEDARONE
Dronedarone is contraindicated in:
Patients with New York Heart Association (NYHA) class IV heart failure or NYHA class II or III heart failure with recent decompensation requiring hospitalization or referral to a specialized heart failure clinic
Patients with second- or third-degree atrioventricular block or sick sinus syndrome (except when used in conjunction with a functioning pacemaker) or bradycardia (a heart rate < 50 beats per minute)
Patients with a QTc interval of 500 ms or longer
Patients with severe hepatic impairment
Women who are pregnant, are attempting to become pregnant, or are breast-feeding
Patients taking potent CYP3A inhibitors—antifungals like ketoconazole (Nizoral), itraconazole (Sporanox), or voriconazole (VFEND); macrolide antibiotics like telithromycin (Ketek) or clarithromycin (Biaxin); protease inhibitors; or other drugs that prolong the QT interval.
In patients with new or worsening heart failure, one should consider suspending or stopping dronedarone therapy.
DRONEDARONE’S ADVERSE EFFECTS
In trials to date, dronedarone has not shown evidence of proarrhythmia (tachyarrhythmia or bradyarrythmia), torsades de pointes, or amiodarone-like organ toxicity affecting the thyroid or the lungs. Recently, rare cases of severe hepatic injury were associated with dronedarone; therefore, periodic liver function testing is advised for patients taking dronedarone, especially during the first 6 months of therapy.
Dronedarone has been associated with higher rates of diarrhea, nausea, bradycardia, QT interval prolongation, and cutaneous rash compared with placebo. In DAFNE (Dronedarone Atrial Fibrillation Study After Electrical Cardioversion),17 10.8% of patients taking dronedarone had to stop taking it because of adverse events. With 800 mg daily, the discontinuation rate was only 3.9%. The most common cause of drug discontinuation was gastrointestinal effects. Anecdotal reports suggest that the gastrointestinal side effects may be self-limited and may not always require discontinuation of the drug.
Serum creatinine levels increase by about 0.1 mg/dL after the start of treatment. This elevation occurs after 1 to 2 days, reaches a plateau after 7 days, and is reversible. The mechanism is thought to be that dronedarone partially inhibits tubular organic cation transporters, which in turn reduces renal creatinine clearance by about 18%, but with no evidence of an effect on glomerular filtration, renal plasma flow, or electrolyte exchanges.18 A limited increase in serum creatinine is, therefore, expected with dronedarone treatment, but this does not mean there is a decline in renal function.
DRONEDARONE AND POTENTIAL DRUG INTERACTIONS
Warfarin. Dronedarone does not increase the international normalized ratio when used with warfarin (Coumadin).
Verapamil, diltiazem. Dose reduction is required to avoid bradyarrhythmias with co-administration of moderate CYP3A4 inhibitors such as verapamil (Calan, Verelan) and diltiazem (Cardizem).
Simvastatin. Dronedarone increases levels of simvastatin (Zocor), a CYP3A4 substrate, two to four times, thus increasing the risk of statin-induced myopathy.
Digoxin. Dronedarone increases the serum digoxin concentration about 2.5 times, and this necessitates monitoring the digoxin level and possibly reducing the digoxin dose.13
Diuretics. Hypokalemia and hypomagnesemia may occur with concomitant administration of potassium-depleting diuretics. Potassium levels should be maintained in the normal range before and during administration of dronedarone.
Tacrolimus, sirolimus. Dronedarone may increase levels of tacrolimus (Prograf) or sirolimus (Rapamune) in posttransplantation patients. This requires dose monitoring and adjustment in concomitant therapy with these agents.
COST VARIES
The cost of dronedarone varies based on factors that include location. Dronedarone’s retail cost ranges from $3.20 to $4.00 per pill (approximately $7.20 per day). It is not available in generic form. It is presently covered by many health plans as a tier 2 drug, representing a $15 to $40 monthly copay.
MORE DATA NEEDED
Dronedarone represents the first in what may well be a number of new antiarrhythmic drugs for the treatment of patients with paroxysmal atrial fibrillation. Although less efficacious then amiodarone, dronedarone appears to be better tolerated and have less serious side effects. It is contraindicated in patients with severe systolic dysfunction and in those with recent heart failure decompensation. It appears safe in coronary artery disease and marked left ventricular hypertrophy, unlike flecainide, propafenone (Rythmol), and sotalol.
To further understand how dronedarone will fare against other antiarrhythmic drugs, more studies with longer follow-up are needed. These studies need to demonstrate superior tolerability of dronedarone, acceptable quality of life without unacceptable loss of efficacy, or a decrease in morbidity or mortality rates compared with amiodarone.
Dronedarone can be safely started in most patients on an outpatient basis. The risk of proarrhythmia with dronedarone appears to be very low.
References
Mathew ST, Patel J, Joseph S. Atrial fibrillation: mechanistic insights and treatment options. Eur J Intern Med2009; 20:672–681.
Schmitt J, Ehrlich JR, Hohnloser SH. New antiarrhythmic drugs for the treatment of atrial fibrillation. Herz2008; 33:562–567.
Wyse DG, Waldo AL, DiMarco JP, et al; Atrial Fibrillation Follow-up Investigation of Rhythm Management (AFFIRM) Investigators. A comparison of rate control and rhythm control in patients with atrial fibrillation. N Engl J Med2002; 347:1825–1833.
Van Gelder IC, Groenveld HF, Crijns HJ, et al; RACE II Investigators. Lenient versus strict rate control in patients with atrial fibrillation. N Engl J Med2010; 362:1363–1373.
Singh BN, Singh SN, Reda DJ, et al; Sotalol Amiodarone Atrial Fibrillation Efficacy Trial (SAFE-T) Investigators. Amiodarone versus sotalol for atrial fibrillation. N Engl J Med2005; 352:1861–1872.
The AFFIRM Investigators. Relationships between sinus rhythm, treatment, and survival in the Atrial Fibrillation Follow-Up Investigation of Rhythm Management (AFFIRM) study. Circulation2004; 109:1509–1513.
Van Beeren HC, Jong WM, Kaptein E, Visser TJ, Bakker O, Wiersinga WM. Dronerarone [sic] acts as a selective inhibitor of 3,5,3′-triiodothyronine binding to thyroid hormone receptor-alpha1: in vitro and in vivo evidence. Endocrinology2003; 144:552–558.
Patel C, Yan GX, Kowey PR. Dronedarone. Circulation2009; 120:636–644.
Dale KM, White CM. Dronedarone: an amiodarone analog for the treatment of atrial fibrillation and atrial flutter. Ann Pharmacother2007; 41:599–605.
Hohnloser SH, Crijns HJ, van Eickels M; ATHENA Investigators. Effect of dronedarone on cardiovascular events in atrial fibrillation. N Engl J Med2009; 360:668–678.
Singh BN, Connolly SJ, Crijns HJ, et al; EURIDIS and ADONIS Investigators. Dronedarone for maintenance of sinus rhythm in atrial fibrillation or flutter. N Engl J Med2007; 357:987–999.
Davy JM, Herold M, Hoglund CERATO Study Investigators. Dronedarone for the control of ventricular rate in permanent atrial fibrillation: the Efficacy and safety of dRonedArone for the cOntrol of ventricular rate during atrial fibrillation (ERATO) study. Am Heart J2008; 156:527.e1–e9.
Køber L, Torp-Pedersen C, McMurray JJ; Dronedarone Study Group. Increased mortality after dronedarone therapy for severe heart failure. N Engl J Med2008; 358:2678–2687.
Le Heuzey JY, De Ferrari GM, Radzik D, Santini M, Zhu J, Davy JM. A short-term, randomized, double-blind, parallel-group study to evaluate the efficacy and safety of dronedarone versus amiodarone in patients with persistent atrial fibrillation: the DIONYSOS study. J Cardiovasc Electrophysiol2010; 21:597–605.
The Cardiac Arrhythmia Suppression Trial (CAST) Investigators. Preliminary report: effect of encainide and flecainide on mortality in a randomized trial of arrhythmia suppression after myocardial infarction. N Engl J Med1989; 321:406–412.
Pratt CM. Clinical implications of the Survival With Oral D-sotalol (SWORD) trial: an investigation of patients with left ventricular dysfunction after myocardial infarction. Card Electrophysiol Rev1998; 2:28–29.
Touboul P, Brugada J, Capucci A, Crijns HJ, Edvardsson N, Hohnloser SH. Dronedarone for prevention of atrial fibrillation: a dose-ranging study. Eur Heart J2003; 24:1481–1487.
Tschuppert Y, Buclin T, Rothuizen LE, et al. Effect of dronedarone on renal function in healthy subjects. Br J Clin Pharmacol2007; 64:785–791.
Neelima Penugonda, MD Department of Internal Medicine, Lankenau Hospital, Wynnewood, PA
Adam Mohmand-Borkowski, MD Department of Internal Medicine, Division of Cardiology, Lankenau Hospital, Wynnewood, PA
James F. Burke, MD Department of Internal Medicine, Division of Cardiology, and Program Director, Fellowship in Cardiovascular Disease, Lankenau Hospital, Wynnewood, PA; Clinical Associate Professor, Thomas Jefferson University, Philadelphia
Address: Neelima Penugonda, MD, Department of Internal Medicine, Lankenau Hospital, 100 East Lancaster Avenue, 3SW, Wynnewood, PA 19096; e-mail pneelu@gmail.com
Neelima Penugonda, MD Department of Internal Medicine, Lankenau Hospital, Wynnewood, PA
Adam Mohmand-Borkowski, MD Department of Internal Medicine, Division of Cardiology, Lankenau Hospital, Wynnewood, PA
James F. Burke, MD Department of Internal Medicine, Division of Cardiology, and Program Director, Fellowship in Cardiovascular Disease, Lankenau Hospital, Wynnewood, PA; Clinical Associate Professor, Thomas Jefferson University, Philadelphia
Address: Neelima Penugonda, MD, Department of Internal Medicine, Lankenau Hospital, 100 East Lancaster Avenue, 3SW, Wynnewood, PA 19096; e-mail pneelu@gmail.com
Author and Disclosure Information
Neelima Penugonda, MD Department of Internal Medicine, Lankenau Hospital, Wynnewood, PA
Adam Mohmand-Borkowski, MD Department of Internal Medicine, Division of Cardiology, Lankenau Hospital, Wynnewood, PA
James F. Burke, MD Department of Internal Medicine, Division of Cardiology, and Program Director, Fellowship in Cardiovascular Disease, Lankenau Hospital, Wynnewood, PA; Clinical Associate Professor, Thomas Jefferson University, Philadelphia
Address: Neelima Penugonda, MD, Department of Internal Medicine, Lankenau Hospital, 100 East Lancaster Avenue, 3SW, Wynnewood, PA 19096; e-mail pneelu@gmail.com
Dronedarone (Multaq), approved by the US Food and Drug Administration in July 2009, is a congener of the antiarrhythmic drug amiodarone (Cordarone). Designed in the hope that it would be safer than amiodarone, its official indication is to lower the risk of hospitalization in patients with paroxysmal or persistent atrial fibrillation or atrial flutter. However, its precise role in the management of atrial fibrillation is yet to be defined. If dronedarone remains well tolerated, it may permit clinicians to pursue a rhythm control strategy more often. In this article, we present a progress report on this new agent.
BETTER ANTIARRHYTHMIC DRUGS ARE NEEDED
Atrial fibrillation increases the risk of stroke fivefold and accounts for 15% to 20% of all strokes.1 It also increases the risk of heart failure. Drugs are the mainstay of therapy, but many antiarrhythmic drugs are not very effective and cause cardiac and extracardiac toxicity. Thus, the need for safe and effective new drugs.2
Much effort is going into the development of drugs that target specific ion channels or proteins expressed predominantly in atrial myocardium. The rationale is to avoid the unwanted effects of ionic currents on the ventricle and thus avoid ventricular proarrhythmic effects. At the same time, alternatives to the multiple channel blocker amiodarone, the mainstay of heart rhythm control therapy in atrial fibrillation, are being developed to retain the electrophysiologic efficacy of the mother compound but avoid its extracardiac toxicity.
RATE CONTROL VS RHYTHM CONTROL
In the acute care setting, heart rate control with atrioventricular nodal agents (beta-blockers, calcium channel blockers, and digitalis) is the preferred initial strategy in most hemodynamically stable patients presenting with new-onset atrial fibrillation.3
Since we lack an effective method for maintaining sinus rhythm without incurring significant adverse effects, rate control is also often chosen for chronic management of atrial fibrillation. This is particularly true for patients who have no symptoms or only minimal symptoms and in whom adequate rate control is easily attained. Indeed, results of large clinical trials suggest that rate control is satisfactory for many patients.
The main purpose of rate control is to control symptoms as opposed to merely lowering the ventricular rate. Effective rate control often prevents hemodynamic instability in patients with underlying heart disease who present acutely with atrial fibrillation. In patients with permanent atrial fibrillation, the RACE II study4 (Rate Control Efficacy in Permanent Atrial Fibrillation: a Comparison between Lenient Versus Strict Rate Control II), during a 3-year follow-up, showed that lenient rate control (resting heart rate < 110 beats per minute) is not inferior to strict rate control (resting heart rate < 80 beats per minute) in preventing major cardiovascular events (heart failure, stroke) or arrhythmic events such as syncope and sustained ventricular tachycardia.4
As a long-term strategy, rate control also prevents tachycardia-induced cardiomyopathy, reduces the risk of worsening of underlying heart failure, and can improve symptoms and quality of life.
Although maintenance of sinus rhythm is most likely associated with a survival benefit, heart rhythm control with antiarrhythmic drugs has not shown an advantage over rate control in overall or cardiovascular death rates, thromboembolic complications, or impact on heart failure. Indeed, a rhythm control strategy has been associated only with better exercise tolerance and, although less clear, with better quality of life.5
One possible explanation as to why a rhythm control strategy has not been shown to be superior to a rate control strategy is the side effects of the presently available drugs for rhythm control.
In a subgroup analysis of the Atrial Fibrillation Follow-up Investigation of Rhythm Management (AFFIRM) trial,6 antiarrhythmic therapy was associated with a 49% increase in the mortality rate that offset the benefits of conversion and maintenance of sinus rhythm, which was associated with a 53% reduction in mortality rates.
The hope is that newer drugs with less toxicity may produce better outcomes for patients treated with rhythm control.
AN ANALOGUE OF AMIODARONE, WITHOUT THE IODINE
Dronedarone is a structurally modified version of amiodarone, the antiarrhythmic drug that has shown the greatest efficacy at maintaining sinus rhythm in patients with paroxysmal atrial fibrillation. Although historically amiodarone has been effective in maintaining sinus rhythm and has been used safely in patients with advanced heart failure, its use has been limited by cumulative and often irreversible extracardiac organ toxicity.
Dronedarone was designed to match amiodarone’s efficacy but with a better safety profile. An iodine radical makes up more than one-third of amiodarone’s molecular weight. The omission of iodine in dronedarone was intended to reduce the likelihood of toxic side effects.
Dronedarone is a benzofuran derivative pharmacologically related to amiodarone, with the addition of a methylsulfonamide group. This reduces lipophilicity and the propensity to cross the blood-brain barrier; over a 2-year period this drug has not been shown to have neurotoxic effects.7
Dronedarone has proved efficacious without toxic or proarrhythmic effects and has minimal side effects, but concerns remain regarding its use in advanced heart failure. To date, its adverse-event profile appears comparable to that of placebo. However, whether its efficacy and incidence of adverse effects are comparable to what has been reported in the literature may take time to assess.
DRONEDARONE’S PHARMACOLOGY
Dronedarone, like amiodarone, blocks multiple sodium and potassium ion channels. It also exerts an antiadrenergic effect by noncompetitive binding to beta-adrenergic receptors as well as by inhibiting an agonist-induced increase in adenylate cyclase activity.8 Compared with amiodarone, dronedarone is a more potent blocker of peak sodium current.
Dronedarone is largely metabolized by the hepatic enzyme cytochrome P450 3A4 isoform (CYP3A4). Only 6% of dronedarone is excreted renally; however, no trial has yet assessed dronedarone’s safety in patients with marked kidney dysfunction.89
Dronedarone’s steady-state terminal elimination half-life is approximately 30 hours. When taken twice a day, it achieves steady-state concentrations in 5 to 7 days.
Dronedarone is available only for oral administration at 400 mg twice daily. Dose adjustment or titration is not recommended.
CLINICAL TRIALS OF DRONEDARONE
Dronedarone vs placebo
ATHENA (A Placebo-Controlled, Double-Blind, Parallel Arm Trial to Assess the Efficacy of Dronedarone 400 mg bid for the Prevention of Cardiovascular Hospitalization or Death From any Cause in Patients With Atrial Fibrillation/Atrial Flutter)10 was a prospective, double-blind study to assess morbidity and death rates in 4,628 patients with atrial fibrillation or atrial flutter and at least one other cardiovascular risk factor.
ATHENA showed that dronedarone, in addition to standard therapy, significantly reduced the risk of a first cardiovascular hospitalization or death by 24% in patients with atrial fibrillation or atrial flutter.9 The study excluded patients with decompensated heart failure(Table 1).
EURIDIS and ADONIS. Two trials,11 EURIDIS (European Trial in Atrial Fibrillation or Flutter Patients Receiving Dronedarone for the Maintenance of Sinus Rhythm) and ADONIS (American-Australian Trial With Dronedarone in Atrial Fibrillation or Flutter Patients for the Maintenance of Sinus Rhythm), enrolled a total of more than 1,200 patients and showed that dronedarone 400 mg twice a day produced a significantly lower rate of recurrence of atrial fibrillation after electrical cardioversion compared with placebo.
Overall, treatment with dronedarone significantly reduced the risk of a first recurrence of atrial fibrillation by 22% (ADONIS) and 27.5% (EURIDIS) (Table 1).
ERATO (Efficacy and Safety of Dronedarone for the Control of Ventricular Rate During Atrial Fibrillation),12 an additional phase III study, showed that dronedarone controlled the heart rate in patients with persistently accelerated ventricular rates despite concomitant standard therapy with a beta-blocker, digitalis, or a calcium-channel blocker. Dronedarone reduced the mean 24-hour heart rate by 11.7 beats per minute and the maximal exercise ventricular rate by 24.5 beats per minute at the 14th day.
ANDROMEDA (Anti-arrhythmic Trial With Dronedarone in Moderate to Severe CHF Evaluating Morbidity Decrease)13 was a study not of patients with atrial fibrillation but rather of patients with symptomatic congestive heart failure, a left ventricular ejection fraction of 35% or less, and recent hospitalization with new or worsening heart failure. The study was terminated early because of a higher rate of death with dronedarone13 (Table 1).
Dronedarone vs amiodarone
DIONYSOS (Efficacy and Safety of Dronedarone Versus Amiodarone for the Maintenance of Sinus Rhythm in Patients With Atrial Fibrillation)14 was a randomized double-blind trial. It evaluated the efficacy and safety of dronedarone (400 mg twice daily) or amiodarone (600 mg daily for 28 days, then 200 mg daily thereafter) for at least 6 months for the maintenance of sinus rhythm in patients with atrial fibrillation. It enrolled 504 patients with persistent atrial fibrillation; patients had not previously taken amiodarone. Dronedarone was less effective than amiodarone in maintaining sinus rhythm: the rate of recurrent atrial fibrillation was 63% with dronedarone and 42% with amiodarone. But dronedarone was associated with fewer adverse effects and less need for premature discontinuation of drug treatment at a mean follow-up of 7 months (Table 1).
WHERE DOES DRONEDARONE FIT IN ATRIAL FIBRILLATION MANAGEMENT?
Dronedarone is indicated in persistent or paroxysmal atrial fibrillation, based on the observed reduction of the rate of hospitalization. It is indicated for the maintenance of sinus rhythm and may be used in patients with persistent or paroxysmal atrial fibrillation and flutter who are in sinus rhythm or will be undergoing cardioversion soon after starting the drug. Dronedarone has no role in the acute management of atrial fibrillation, such as in cardioversion to sinus rhythm in the emergency department.
We do not have substantial evidence of the efficacy of dronedarone in patients with resistant atrial fibrillation, in whom multiple antiarrhythmics have failed to maintain sinus rhythm, and no published trial has used the inclusion criterion of treatment failure with other antiarrhythmic drugs.
The role of dronedarone in heart failure with preserved systolic function is unclear. Patients taking dronedarone are twice as likely as those taking amiodarone to have a recurrence of atrial fibrillation.
The main advantage of dronedarone is its lower adverse effect profile. However, this statement is based on only a few years of observation. If the patient has developed adverse effects with amiodarone, or if the clinician is concerned about the risk of serious adverse effects, dronedarone presents an alternative for those patients without heart failure or significant left ventricular dysfunction. One such group may be younger patients, because of concerns about the cumulative effects of amiodarone taken over a lifetime.
Dronedarone may represent an acceptable alternative to many of the current antiarrhythmic drugs. Based on the results of the Cardiac Arrhythmia Suppression Trial (CAST),15 class IC antiarrhythmics such as flecainide (Tambocor) are generally avoided in patients with prior myocardial infarction or with known or even suspected coronary artery disease. Similarly, sotalol (Betapace) is generally avoided in patients with marked left ventricular hypertrophy because of adverse effects.16 Dofetilide (Tikosyn) and often sotalol require hospitalization with telemetric monitoring for QTc prolongation and the risk of proarrhythmia with torsades de pointes. Dronedarone, however, generally can be safely started in the outpatient setting.
As when considering prescribing any antiarrhythmic, the clinician must assess the patient’s thromboembolic risk, since this risk persists with a rhythm control strategy.
There is substantial evidence from the ATHENA trial,10 in which 30% of the patients had coronary artery disease, that dronedarone is safe and effective in patients with coronary artery disease. Its use in patients who have undergone coronary artery bypass surgery remains to be defined.
WHEN SHOULD WE SWITCH PATIENTS TO DRONEDARONE?
Preliminary experience suggests that dronedarone, unlike most antiarrhythmic drugs, can be safely started about 48 hours after amiodarone is discontinued. Cumulative toxicity has not been noted with dronedarone. Caution should be exercised when switching if the patient has baseline bradycardia or QT interval prolongation. No algorithm has been developed for switching from other antiarrhythmic drugs to dronedarone.
CONTRAINDICATIONS TO DRONEDARONE
Dronedarone is contraindicated in:
Patients with New York Heart Association (NYHA) class IV heart failure or NYHA class II or III heart failure with recent decompensation requiring hospitalization or referral to a specialized heart failure clinic
Patients with second- or third-degree atrioventricular block or sick sinus syndrome (except when used in conjunction with a functioning pacemaker) or bradycardia (a heart rate < 50 beats per minute)
Patients with a QTc interval of 500 ms or longer
Patients with severe hepatic impairment
Women who are pregnant, are attempting to become pregnant, or are breast-feeding
Patients taking potent CYP3A inhibitors—antifungals like ketoconazole (Nizoral), itraconazole (Sporanox), or voriconazole (VFEND); macrolide antibiotics like telithromycin (Ketek) or clarithromycin (Biaxin); protease inhibitors; or other drugs that prolong the QT interval.
In patients with new or worsening heart failure, one should consider suspending or stopping dronedarone therapy.
DRONEDARONE’S ADVERSE EFFECTS
In trials to date, dronedarone has not shown evidence of proarrhythmia (tachyarrhythmia or bradyarrythmia), torsades de pointes, or amiodarone-like organ toxicity affecting the thyroid or the lungs. Recently, rare cases of severe hepatic injury were associated with dronedarone; therefore, periodic liver function testing is advised for patients taking dronedarone, especially during the first 6 months of therapy.
Dronedarone has been associated with higher rates of diarrhea, nausea, bradycardia, QT interval prolongation, and cutaneous rash compared with placebo. In DAFNE (Dronedarone Atrial Fibrillation Study After Electrical Cardioversion),17 10.8% of patients taking dronedarone had to stop taking it because of adverse events. With 800 mg daily, the discontinuation rate was only 3.9%. The most common cause of drug discontinuation was gastrointestinal effects. Anecdotal reports suggest that the gastrointestinal side effects may be self-limited and may not always require discontinuation of the drug.
Serum creatinine levels increase by about 0.1 mg/dL after the start of treatment. This elevation occurs after 1 to 2 days, reaches a plateau after 7 days, and is reversible. The mechanism is thought to be that dronedarone partially inhibits tubular organic cation transporters, which in turn reduces renal creatinine clearance by about 18%, but with no evidence of an effect on glomerular filtration, renal plasma flow, or electrolyte exchanges.18 A limited increase in serum creatinine is, therefore, expected with dronedarone treatment, but this does not mean there is a decline in renal function.
DRONEDARONE AND POTENTIAL DRUG INTERACTIONS
Warfarin. Dronedarone does not increase the international normalized ratio when used with warfarin (Coumadin).
Verapamil, diltiazem. Dose reduction is required to avoid bradyarrhythmias with co-administration of moderate CYP3A4 inhibitors such as verapamil (Calan, Verelan) and diltiazem (Cardizem).
Simvastatin. Dronedarone increases levels of simvastatin (Zocor), a CYP3A4 substrate, two to four times, thus increasing the risk of statin-induced myopathy.
Digoxin. Dronedarone increases the serum digoxin concentration about 2.5 times, and this necessitates monitoring the digoxin level and possibly reducing the digoxin dose.13
Diuretics. Hypokalemia and hypomagnesemia may occur with concomitant administration of potassium-depleting diuretics. Potassium levels should be maintained in the normal range before and during administration of dronedarone.
Tacrolimus, sirolimus. Dronedarone may increase levels of tacrolimus (Prograf) or sirolimus (Rapamune) in posttransplantation patients. This requires dose monitoring and adjustment in concomitant therapy with these agents.
COST VARIES
The cost of dronedarone varies based on factors that include location. Dronedarone’s retail cost ranges from $3.20 to $4.00 per pill (approximately $7.20 per day). It is not available in generic form. It is presently covered by many health plans as a tier 2 drug, representing a $15 to $40 monthly copay.
MORE DATA NEEDED
Dronedarone represents the first in what may well be a number of new antiarrhythmic drugs for the treatment of patients with paroxysmal atrial fibrillation. Although less efficacious then amiodarone, dronedarone appears to be better tolerated and have less serious side effects. It is contraindicated in patients with severe systolic dysfunction and in those with recent heart failure decompensation. It appears safe in coronary artery disease and marked left ventricular hypertrophy, unlike flecainide, propafenone (Rythmol), and sotalol.
To further understand how dronedarone will fare against other antiarrhythmic drugs, more studies with longer follow-up are needed. These studies need to demonstrate superior tolerability of dronedarone, acceptable quality of life without unacceptable loss of efficacy, or a decrease in morbidity or mortality rates compared with amiodarone.
Dronedarone can be safely started in most patients on an outpatient basis. The risk of proarrhythmia with dronedarone appears to be very low.
Dronedarone (Multaq), approved by the US Food and Drug Administration in July 2009, is a congener of the antiarrhythmic drug amiodarone (Cordarone). Designed in the hope that it would be safer than amiodarone, its official indication is to lower the risk of hospitalization in patients with paroxysmal or persistent atrial fibrillation or atrial flutter. However, its precise role in the management of atrial fibrillation is yet to be defined. If dronedarone remains well tolerated, it may permit clinicians to pursue a rhythm control strategy more often. In this article, we present a progress report on this new agent.
BETTER ANTIARRHYTHMIC DRUGS ARE NEEDED
Atrial fibrillation increases the risk of stroke fivefold and accounts for 15% to 20% of all strokes.1 It also increases the risk of heart failure. Drugs are the mainstay of therapy, but many antiarrhythmic drugs are not very effective and cause cardiac and extracardiac toxicity. Thus, the need for safe and effective new drugs.2
Much effort is going into the development of drugs that target specific ion channels or proteins expressed predominantly in atrial myocardium. The rationale is to avoid the unwanted effects of ionic currents on the ventricle and thus avoid ventricular proarrhythmic effects. At the same time, alternatives to the multiple channel blocker amiodarone, the mainstay of heart rhythm control therapy in atrial fibrillation, are being developed to retain the electrophysiologic efficacy of the mother compound but avoid its extracardiac toxicity.
RATE CONTROL VS RHYTHM CONTROL
In the acute care setting, heart rate control with atrioventricular nodal agents (beta-blockers, calcium channel blockers, and digitalis) is the preferred initial strategy in most hemodynamically stable patients presenting with new-onset atrial fibrillation.3
Since we lack an effective method for maintaining sinus rhythm without incurring significant adverse effects, rate control is also often chosen for chronic management of atrial fibrillation. This is particularly true for patients who have no symptoms or only minimal symptoms and in whom adequate rate control is easily attained. Indeed, results of large clinical trials suggest that rate control is satisfactory for many patients.
The main purpose of rate control is to control symptoms as opposed to merely lowering the ventricular rate. Effective rate control often prevents hemodynamic instability in patients with underlying heart disease who present acutely with atrial fibrillation. In patients with permanent atrial fibrillation, the RACE II study4 (Rate Control Efficacy in Permanent Atrial Fibrillation: a Comparison between Lenient Versus Strict Rate Control II), during a 3-year follow-up, showed that lenient rate control (resting heart rate < 110 beats per minute) is not inferior to strict rate control (resting heart rate < 80 beats per minute) in preventing major cardiovascular events (heart failure, stroke) or arrhythmic events such as syncope and sustained ventricular tachycardia.4
As a long-term strategy, rate control also prevents tachycardia-induced cardiomyopathy, reduces the risk of worsening of underlying heart failure, and can improve symptoms and quality of life.
Although maintenance of sinus rhythm is most likely associated with a survival benefit, heart rhythm control with antiarrhythmic drugs has not shown an advantage over rate control in overall or cardiovascular death rates, thromboembolic complications, or impact on heart failure. Indeed, a rhythm control strategy has been associated only with better exercise tolerance and, although less clear, with better quality of life.5
One possible explanation as to why a rhythm control strategy has not been shown to be superior to a rate control strategy is the side effects of the presently available drugs for rhythm control.
In a subgroup analysis of the Atrial Fibrillation Follow-up Investigation of Rhythm Management (AFFIRM) trial,6 antiarrhythmic therapy was associated with a 49% increase in the mortality rate that offset the benefits of conversion and maintenance of sinus rhythm, which was associated with a 53% reduction in mortality rates.
The hope is that newer drugs with less toxicity may produce better outcomes for patients treated with rhythm control.
AN ANALOGUE OF AMIODARONE, WITHOUT THE IODINE
Dronedarone is a structurally modified version of amiodarone, the antiarrhythmic drug that has shown the greatest efficacy at maintaining sinus rhythm in patients with paroxysmal atrial fibrillation. Although historically amiodarone has been effective in maintaining sinus rhythm and has been used safely in patients with advanced heart failure, its use has been limited by cumulative and often irreversible extracardiac organ toxicity.
Dronedarone was designed to match amiodarone’s efficacy but with a better safety profile. An iodine radical makes up more than one-third of amiodarone’s molecular weight. The omission of iodine in dronedarone was intended to reduce the likelihood of toxic side effects.
Dronedarone is a benzofuran derivative pharmacologically related to amiodarone, with the addition of a methylsulfonamide group. This reduces lipophilicity and the propensity to cross the blood-brain barrier; over a 2-year period this drug has not been shown to have neurotoxic effects.7
Dronedarone has proved efficacious without toxic or proarrhythmic effects and has minimal side effects, but concerns remain regarding its use in advanced heart failure. To date, its adverse-event profile appears comparable to that of placebo. However, whether its efficacy and incidence of adverse effects are comparable to what has been reported in the literature may take time to assess.
DRONEDARONE’S PHARMACOLOGY
Dronedarone, like amiodarone, blocks multiple sodium and potassium ion channels. It also exerts an antiadrenergic effect by noncompetitive binding to beta-adrenergic receptors as well as by inhibiting an agonist-induced increase in adenylate cyclase activity.8 Compared with amiodarone, dronedarone is a more potent blocker of peak sodium current.
Dronedarone is largely metabolized by the hepatic enzyme cytochrome P450 3A4 isoform (CYP3A4). Only 6% of dronedarone is excreted renally; however, no trial has yet assessed dronedarone’s safety in patients with marked kidney dysfunction.89
Dronedarone’s steady-state terminal elimination half-life is approximately 30 hours. When taken twice a day, it achieves steady-state concentrations in 5 to 7 days.
Dronedarone is available only for oral administration at 400 mg twice daily. Dose adjustment or titration is not recommended.
CLINICAL TRIALS OF DRONEDARONE
Dronedarone vs placebo
ATHENA (A Placebo-Controlled, Double-Blind, Parallel Arm Trial to Assess the Efficacy of Dronedarone 400 mg bid for the Prevention of Cardiovascular Hospitalization or Death From any Cause in Patients With Atrial Fibrillation/Atrial Flutter)10 was a prospective, double-blind study to assess morbidity and death rates in 4,628 patients with atrial fibrillation or atrial flutter and at least one other cardiovascular risk factor.
ATHENA showed that dronedarone, in addition to standard therapy, significantly reduced the risk of a first cardiovascular hospitalization or death by 24% in patients with atrial fibrillation or atrial flutter.9 The study excluded patients with decompensated heart failure(Table 1).
EURIDIS and ADONIS. Two trials,11 EURIDIS (European Trial in Atrial Fibrillation or Flutter Patients Receiving Dronedarone for the Maintenance of Sinus Rhythm) and ADONIS (American-Australian Trial With Dronedarone in Atrial Fibrillation or Flutter Patients for the Maintenance of Sinus Rhythm), enrolled a total of more than 1,200 patients and showed that dronedarone 400 mg twice a day produced a significantly lower rate of recurrence of atrial fibrillation after electrical cardioversion compared with placebo.
Overall, treatment with dronedarone significantly reduced the risk of a first recurrence of atrial fibrillation by 22% (ADONIS) and 27.5% (EURIDIS) (Table 1).
ERATO (Efficacy and Safety of Dronedarone for the Control of Ventricular Rate During Atrial Fibrillation),12 an additional phase III study, showed that dronedarone controlled the heart rate in patients with persistently accelerated ventricular rates despite concomitant standard therapy with a beta-blocker, digitalis, or a calcium-channel blocker. Dronedarone reduced the mean 24-hour heart rate by 11.7 beats per minute and the maximal exercise ventricular rate by 24.5 beats per minute at the 14th day.
ANDROMEDA (Anti-arrhythmic Trial With Dronedarone in Moderate to Severe CHF Evaluating Morbidity Decrease)13 was a study not of patients with atrial fibrillation but rather of patients with symptomatic congestive heart failure, a left ventricular ejection fraction of 35% or less, and recent hospitalization with new or worsening heart failure. The study was terminated early because of a higher rate of death with dronedarone13 (Table 1).
Dronedarone vs amiodarone
DIONYSOS (Efficacy and Safety of Dronedarone Versus Amiodarone for the Maintenance of Sinus Rhythm in Patients With Atrial Fibrillation)14 was a randomized double-blind trial. It evaluated the efficacy and safety of dronedarone (400 mg twice daily) or amiodarone (600 mg daily for 28 days, then 200 mg daily thereafter) for at least 6 months for the maintenance of sinus rhythm in patients with atrial fibrillation. It enrolled 504 patients with persistent atrial fibrillation; patients had not previously taken amiodarone. Dronedarone was less effective than amiodarone in maintaining sinus rhythm: the rate of recurrent atrial fibrillation was 63% with dronedarone and 42% with amiodarone. But dronedarone was associated with fewer adverse effects and less need for premature discontinuation of drug treatment at a mean follow-up of 7 months (Table 1).
WHERE DOES DRONEDARONE FIT IN ATRIAL FIBRILLATION MANAGEMENT?
Dronedarone is indicated in persistent or paroxysmal atrial fibrillation, based on the observed reduction of the rate of hospitalization. It is indicated for the maintenance of sinus rhythm and may be used in patients with persistent or paroxysmal atrial fibrillation and flutter who are in sinus rhythm or will be undergoing cardioversion soon after starting the drug. Dronedarone has no role in the acute management of atrial fibrillation, such as in cardioversion to sinus rhythm in the emergency department.
We do not have substantial evidence of the efficacy of dronedarone in patients with resistant atrial fibrillation, in whom multiple antiarrhythmics have failed to maintain sinus rhythm, and no published trial has used the inclusion criterion of treatment failure with other antiarrhythmic drugs.
The role of dronedarone in heart failure with preserved systolic function is unclear. Patients taking dronedarone are twice as likely as those taking amiodarone to have a recurrence of atrial fibrillation.
The main advantage of dronedarone is its lower adverse effect profile. However, this statement is based on only a few years of observation. If the patient has developed adverse effects with amiodarone, or if the clinician is concerned about the risk of serious adverse effects, dronedarone presents an alternative for those patients without heart failure or significant left ventricular dysfunction. One such group may be younger patients, because of concerns about the cumulative effects of amiodarone taken over a lifetime.
Dronedarone may represent an acceptable alternative to many of the current antiarrhythmic drugs. Based on the results of the Cardiac Arrhythmia Suppression Trial (CAST),15 class IC antiarrhythmics such as flecainide (Tambocor) are generally avoided in patients with prior myocardial infarction or with known or even suspected coronary artery disease. Similarly, sotalol (Betapace) is generally avoided in patients with marked left ventricular hypertrophy because of adverse effects.16 Dofetilide (Tikosyn) and often sotalol require hospitalization with telemetric monitoring for QTc prolongation and the risk of proarrhythmia with torsades de pointes. Dronedarone, however, generally can be safely started in the outpatient setting.
As when considering prescribing any antiarrhythmic, the clinician must assess the patient’s thromboembolic risk, since this risk persists with a rhythm control strategy.
There is substantial evidence from the ATHENA trial,10 in which 30% of the patients had coronary artery disease, that dronedarone is safe and effective in patients with coronary artery disease. Its use in patients who have undergone coronary artery bypass surgery remains to be defined.
WHEN SHOULD WE SWITCH PATIENTS TO DRONEDARONE?
Preliminary experience suggests that dronedarone, unlike most antiarrhythmic drugs, can be safely started about 48 hours after amiodarone is discontinued. Cumulative toxicity has not been noted with dronedarone. Caution should be exercised when switching if the patient has baseline bradycardia or QT interval prolongation. No algorithm has been developed for switching from other antiarrhythmic drugs to dronedarone.
CONTRAINDICATIONS TO DRONEDARONE
Dronedarone is contraindicated in:
Patients with New York Heart Association (NYHA) class IV heart failure or NYHA class II or III heart failure with recent decompensation requiring hospitalization or referral to a specialized heart failure clinic
Patients with second- or third-degree atrioventricular block or sick sinus syndrome (except when used in conjunction with a functioning pacemaker) or bradycardia (a heart rate < 50 beats per minute)
Patients with a QTc interval of 500 ms or longer
Patients with severe hepatic impairment
Women who are pregnant, are attempting to become pregnant, or are breast-feeding
Patients taking potent CYP3A inhibitors—antifungals like ketoconazole (Nizoral), itraconazole (Sporanox), or voriconazole (VFEND); macrolide antibiotics like telithromycin (Ketek) or clarithromycin (Biaxin); protease inhibitors; or other drugs that prolong the QT interval.
In patients with new or worsening heart failure, one should consider suspending or stopping dronedarone therapy.
DRONEDARONE’S ADVERSE EFFECTS
In trials to date, dronedarone has not shown evidence of proarrhythmia (tachyarrhythmia or bradyarrythmia), torsades de pointes, or amiodarone-like organ toxicity affecting the thyroid or the lungs. Recently, rare cases of severe hepatic injury were associated with dronedarone; therefore, periodic liver function testing is advised for patients taking dronedarone, especially during the first 6 months of therapy.
Dronedarone has been associated with higher rates of diarrhea, nausea, bradycardia, QT interval prolongation, and cutaneous rash compared with placebo. In DAFNE (Dronedarone Atrial Fibrillation Study After Electrical Cardioversion),17 10.8% of patients taking dronedarone had to stop taking it because of adverse events. With 800 mg daily, the discontinuation rate was only 3.9%. The most common cause of drug discontinuation was gastrointestinal effects. Anecdotal reports suggest that the gastrointestinal side effects may be self-limited and may not always require discontinuation of the drug.
Serum creatinine levels increase by about 0.1 mg/dL after the start of treatment. This elevation occurs after 1 to 2 days, reaches a plateau after 7 days, and is reversible. The mechanism is thought to be that dronedarone partially inhibits tubular organic cation transporters, which in turn reduces renal creatinine clearance by about 18%, but with no evidence of an effect on glomerular filtration, renal plasma flow, or electrolyte exchanges.18 A limited increase in serum creatinine is, therefore, expected with dronedarone treatment, but this does not mean there is a decline in renal function.
DRONEDARONE AND POTENTIAL DRUG INTERACTIONS
Warfarin. Dronedarone does not increase the international normalized ratio when used with warfarin (Coumadin).
Verapamil, diltiazem. Dose reduction is required to avoid bradyarrhythmias with co-administration of moderate CYP3A4 inhibitors such as verapamil (Calan, Verelan) and diltiazem (Cardizem).
Simvastatin. Dronedarone increases levels of simvastatin (Zocor), a CYP3A4 substrate, two to four times, thus increasing the risk of statin-induced myopathy.
Digoxin. Dronedarone increases the serum digoxin concentration about 2.5 times, and this necessitates monitoring the digoxin level and possibly reducing the digoxin dose.13
Diuretics. Hypokalemia and hypomagnesemia may occur with concomitant administration of potassium-depleting diuretics. Potassium levels should be maintained in the normal range before and during administration of dronedarone.
Tacrolimus, sirolimus. Dronedarone may increase levels of tacrolimus (Prograf) or sirolimus (Rapamune) in posttransplantation patients. This requires dose monitoring and adjustment in concomitant therapy with these agents.
COST VARIES
The cost of dronedarone varies based on factors that include location. Dronedarone’s retail cost ranges from $3.20 to $4.00 per pill (approximately $7.20 per day). It is not available in generic form. It is presently covered by many health plans as a tier 2 drug, representing a $15 to $40 monthly copay.
MORE DATA NEEDED
Dronedarone represents the first in what may well be a number of new antiarrhythmic drugs for the treatment of patients with paroxysmal atrial fibrillation. Although less efficacious then amiodarone, dronedarone appears to be better tolerated and have less serious side effects. It is contraindicated in patients with severe systolic dysfunction and in those with recent heart failure decompensation. It appears safe in coronary artery disease and marked left ventricular hypertrophy, unlike flecainide, propafenone (Rythmol), and sotalol.
To further understand how dronedarone will fare against other antiarrhythmic drugs, more studies with longer follow-up are needed. These studies need to demonstrate superior tolerability of dronedarone, acceptable quality of life without unacceptable loss of efficacy, or a decrease in morbidity or mortality rates compared with amiodarone.
Dronedarone can be safely started in most patients on an outpatient basis. The risk of proarrhythmia with dronedarone appears to be very low.
References
Mathew ST, Patel J, Joseph S. Atrial fibrillation: mechanistic insights and treatment options. Eur J Intern Med2009; 20:672–681.
Schmitt J, Ehrlich JR, Hohnloser SH. New antiarrhythmic drugs for the treatment of atrial fibrillation. Herz2008; 33:562–567.
Wyse DG, Waldo AL, DiMarco JP, et al; Atrial Fibrillation Follow-up Investigation of Rhythm Management (AFFIRM) Investigators. A comparison of rate control and rhythm control in patients with atrial fibrillation. N Engl J Med2002; 347:1825–1833.
Van Gelder IC, Groenveld HF, Crijns HJ, et al; RACE II Investigators. Lenient versus strict rate control in patients with atrial fibrillation. N Engl J Med2010; 362:1363–1373.
Singh BN, Singh SN, Reda DJ, et al; Sotalol Amiodarone Atrial Fibrillation Efficacy Trial (SAFE-T) Investigators. Amiodarone versus sotalol for atrial fibrillation. N Engl J Med2005; 352:1861–1872.
The AFFIRM Investigators. Relationships between sinus rhythm, treatment, and survival in the Atrial Fibrillation Follow-Up Investigation of Rhythm Management (AFFIRM) study. Circulation2004; 109:1509–1513.
Van Beeren HC, Jong WM, Kaptein E, Visser TJ, Bakker O, Wiersinga WM. Dronerarone [sic] acts as a selective inhibitor of 3,5,3′-triiodothyronine binding to thyroid hormone receptor-alpha1: in vitro and in vivo evidence. Endocrinology2003; 144:552–558.
Patel C, Yan GX, Kowey PR. Dronedarone. Circulation2009; 120:636–644.
Dale KM, White CM. Dronedarone: an amiodarone analog for the treatment of atrial fibrillation and atrial flutter. Ann Pharmacother2007; 41:599–605.
Hohnloser SH, Crijns HJ, van Eickels M; ATHENA Investigators. Effect of dronedarone on cardiovascular events in atrial fibrillation. N Engl J Med2009; 360:668–678.
Singh BN, Connolly SJ, Crijns HJ, et al; EURIDIS and ADONIS Investigators. Dronedarone for maintenance of sinus rhythm in atrial fibrillation or flutter. N Engl J Med2007; 357:987–999.
Davy JM, Herold M, Hoglund CERATO Study Investigators. Dronedarone for the control of ventricular rate in permanent atrial fibrillation: the Efficacy and safety of dRonedArone for the cOntrol of ventricular rate during atrial fibrillation (ERATO) study. Am Heart J2008; 156:527.e1–e9.
Køber L, Torp-Pedersen C, McMurray JJ; Dronedarone Study Group. Increased mortality after dronedarone therapy for severe heart failure. N Engl J Med2008; 358:2678–2687.
Le Heuzey JY, De Ferrari GM, Radzik D, Santini M, Zhu J, Davy JM. A short-term, randomized, double-blind, parallel-group study to evaluate the efficacy and safety of dronedarone versus amiodarone in patients with persistent atrial fibrillation: the DIONYSOS study. J Cardiovasc Electrophysiol2010; 21:597–605.
The Cardiac Arrhythmia Suppression Trial (CAST) Investigators. Preliminary report: effect of encainide and flecainide on mortality in a randomized trial of arrhythmia suppression after myocardial infarction. N Engl J Med1989; 321:406–412.
Pratt CM. Clinical implications of the Survival With Oral D-sotalol (SWORD) trial: an investigation of patients with left ventricular dysfunction after myocardial infarction. Card Electrophysiol Rev1998; 2:28–29.
Touboul P, Brugada J, Capucci A, Crijns HJ, Edvardsson N, Hohnloser SH. Dronedarone for prevention of atrial fibrillation: a dose-ranging study. Eur Heart J2003; 24:1481–1487.
Tschuppert Y, Buclin T, Rothuizen LE, et al. Effect of dronedarone on renal function in healthy subjects. Br J Clin Pharmacol2007; 64:785–791.
References
Mathew ST, Patel J, Joseph S. Atrial fibrillation: mechanistic insights and treatment options. Eur J Intern Med2009; 20:672–681.
Schmitt J, Ehrlich JR, Hohnloser SH. New antiarrhythmic drugs for the treatment of atrial fibrillation. Herz2008; 33:562–567.
Wyse DG, Waldo AL, DiMarco JP, et al; Atrial Fibrillation Follow-up Investigation of Rhythm Management (AFFIRM) Investigators. A comparison of rate control and rhythm control in patients with atrial fibrillation. N Engl J Med2002; 347:1825–1833.
Van Gelder IC, Groenveld HF, Crijns HJ, et al; RACE II Investigators. Lenient versus strict rate control in patients with atrial fibrillation. N Engl J Med2010; 362:1363–1373.
Singh BN, Singh SN, Reda DJ, et al; Sotalol Amiodarone Atrial Fibrillation Efficacy Trial (SAFE-T) Investigators. Amiodarone versus sotalol for atrial fibrillation. N Engl J Med2005; 352:1861–1872.
The AFFIRM Investigators. Relationships between sinus rhythm, treatment, and survival in the Atrial Fibrillation Follow-Up Investigation of Rhythm Management (AFFIRM) study. Circulation2004; 109:1509–1513.
Van Beeren HC, Jong WM, Kaptein E, Visser TJ, Bakker O, Wiersinga WM. Dronerarone [sic] acts as a selective inhibitor of 3,5,3′-triiodothyronine binding to thyroid hormone receptor-alpha1: in vitro and in vivo evidence. Endocrinology2003; 144:552–558.
Patel C, Yan GX, Kowey PR. Dronedarone. Circulation2009; 120:636–644.
Dale KM, White CM. Dronedarone: an amiodarone analog for the treatment of atrial fibrillation and atrial flutter. Ann Pharmacother2007; 41:599–605.
Hohnloser SH, Crijns HJ, van Eickels M; ATHENA Investigators. Effect of dronedarone on cardiovascular events in atrial fibrillation. N Engl J Med2009; 360:668–678.
Singh BN, Connolly SJ, Crijns HJ, et al; EURIDIS and ADONIS Investigators. Dronedarone for maintenance of sinus rhythm in atrial fibrillation or flutter. N Engl J Med2007; 357:987–999.
Davy JM, Herold M, Hoglund CERATO Study Investigators. Dronedarone for the control of ventricular rate in permanent atrial fibrillation: the Efficacy and safety of dRonedArone for the cOntrol of ventricular rate during atrial fibrillation (ERATO) study. Am Heart J2008; 156:527.e1–e9.
Køber L, Torp-Pedersen C, McMurray JJ; Dronedarone Study Group. Increased mortality after dronedarone therapy for severe heart failure. N Engl J Med2008; 358:2678–2687.
Le Heuzey JY, De Ferrari GM, Radzik D, Santini M, Zhu J, Davy JM. A short-term, randomized, double-blind, parallel-group study to evaluate the efficacy and safety of dronedarone versus amiodarone in patients with persistent atrial fibrillation: the DIONYSOS study. J Cardiovasc Electrophysiol2010; 21:597–605.
The Cardiac Arrhythmia Suppression Trial (CAST) Investigators. Preliminary report: effect of encainide and flecainide on mortality in a randomized trial of arrhythmia suppression after myocardial infarction. N Engl J Med1989; 321:406–412.
Pratt CM. Clinical implications of the Survival With Oral D-sotalol (SWORD) trial: an investigation of patients with left ventricular dysfunction after myocardial infarction. Card Electrophysiol Rev1998; 2:28–29.
Touboul P, Brugada J, Capucci A, Crijns HJ, Edvardsson N, Hohnloser SH. Dronedarone for prevention of atrial fibrillation: a dose-ranging study. Eur Heart J2003; 24:1481–1487.
Tschuppert Y, Buclin T, Rothuizen LE, et al. Effect of dronedarone on renal function in healthy subjects. Br J Clin Pharmacol2007; 64:785–791.
Patients with persistent or paroxysmal atrial fibrillation are candidates for dronedarone therapy if they are in sinus rhythm or will be cardioverted soon after starting. This drug is not indicated for the acute management of atrial fibrillation, for example, in the emergency department.
Dronedarone is an option if a patient cannot tolerate amiodarone or has an underlying condition such as pulmonary or thyroid disease that is a contraindication to amiodarone.
Dronedarone is contraindicated in patients with significant left ventricular dysfunction or heart failure with recent decompensation.
The ultimate role for dronedarone is yet to be defined. Little evidence exists as to whether it will succeed when other drugs have failed.
Four months after undergoing laparoscopic cholecystectomy for symptomatic gallstones, an otherwise healthy 26-year-old woman begins to have episodes of epigastric and back pain similar to what she experienced before the surgery. The surgery was without complications, and her classic biliary colic disappeared afterward. Histologic evaluation of the surgical specimen revealed chronic cholecystitis with multiple small, mixed gallstones.
Now she describes a burning pain in her epigastrium and mid to upper back, starting about 30 minutes after a meal and lasting up to 4 hours. Sometimes it awakens her at night. She avoids eating for fear of inducing the pain. She has occasional chills but no fever, nausea, vomiting, jaundice, or changes in urine or stool color.
Three years ago she was diagnosed with a gastric ulcer induced by taking a nonsteroidal anti-inflammatory drug (NSAID). The ulcer was treated with a proton pump inhibitor for 1 month. She says the ulcer pain was dull and aching, different from her current pain.
Upper endoscopy 4 months ago (ie, before her laparoscopic cholecystectomy) showed no evidence of esophagitis or peptic ulcer disease.
Apart from her gallbladder operation, she has had no other surgery. According to the surgeon’s notes, intraoperative cholangiography was not performed, and no macroscopic changes of acute cholecystitis or difficult biliary anatomy were noted.
The patient does not smoke, does not drink alcohol, is not currently taking any medications, including NSAIDs or over-the-counter medications, and has not taken any recently. Her mother also had symptomatic gallstones requiring cholecystectomy.
On physical examination, only fever
On examination, her temperature is 101.2°F (38.4°C), blood pressure 117/80 mm Hg, heart rate 82 beats per minute, and blood oxygen saturation 99% on room air. Her weight is 138 lb (62.6 kg), height 5 feet 6 inches (168 cm).
There is no jaundice or pallor. Her heart and lung examinations are normal.
Her abdomen is soft and mildly tender to palpation of the epigastrium, with no distention or hepatosplenomegaly and no rebound tenderness or guarding. The scars from her laparoscopic surgery have healed well. Her bowel sounds are normal.
No costovertebral angle or spinal tenderness can be elicited.
Her laboratory values are shown in Table 1.
POSTCHOLECYSTECTOMY SYNDROME
1.After cholecystectomy, preoperative symptoms recur in what percentage of patients?
10% to 40%
50%
60%
80%
Postcholecystectomy syndrome—the recurrence of symptoms similar to those before the procedure—occurs in 10% to 40% of patients. The time to the onset of symptoms can range from 2 days to up to 25 years.1–4 Women may be at higher risk, with symptoms recurring in 43% vs 28% in men.5
Postcholecystectomy syndrome can have a biliary or a nonbiliary cause. Biliary causes include strictures, retained calculi, dropped calculi, tumors, sphincter of Oddi dysfunction, and calculi in the cystic duct remnant. Nonbiliary causes include functional and organic disorders such as peptic ulcer disease, gastroesophageal reflux, pancreatic disease, hepatocellular disorders, coronary artery disease, irritable bowel syndrome, and intercostal neuritis.
WHAT IS THE NEXT STEP?
2.Which is the most appropriate next step in the workup of this patient?
Ultrasonography of the right upper quadrant
Magnetic resonance cholangiopancreatography (MRCP)
Review the operative record and consult with the surgeon
Although the patient is presenting with pain and fever, two features of the classic Charcot triad (pain, fever, jaundice) seen in cholangitis (infection of a bile duct), and although cholangitis almost confirms the diagnosis of common bile duct stones in a patient with gallstones (before or after cholecystectomy), other diagnoses to consider are bile duct injury, bile leak, and biloma.
Biloma can be detected with ultrasonography. Bile duct injuries are identified intraoperatively in up to 25% of patients. For those with an unrecognized injury, the clinical presentation is variable and depends on the type of injury. If a bile leak is present, patients present early, at a median of 3 days postoperatively. However, our patient presented with symptoms 4 months after her surgery. Patients with bile duct strictures without bile leak have a longer symptom-free interval and usually present with signs of biliary obstruction. Ultrasonography can then detect biliary dilatation.6
It would be very helpful to review the operative record and to talk to the surgeon to confirm that intraoperative cholangiography had not been done and to determine the level of difficulty of the surgery. (Intraoperative cholangiography involves the introduction of contrast dye into the biliary system by cannulation of the cystic duct or by direct injection into the common bile duct. An intraoperative cholangiogram is considered normal if the entire intrahepatic and extrahepatic biliary tree is seen to be filled with contrast.) A normal cholangiogram has a negative predictive value of 99.8% for the detection of ductal stones. Thus, a normal intraoperative cholangiogram can prevent unnecessary postoperative ECRP, since it almost always indicates a clean bile duct.7
Ultrasonography of the right upper quadrant has a low sensitivity (< 50%) for detecting common bile duct stones. However, it is highly operator-dependent, and it may be twice as sensitive if done by expert radiologists than by less experienced ones. Its limitations include poor visualization of the distal portion of the duct and low sensitivity in patients in whom the common bile duct is minimally dilated and also in patients with small stones. In most studies, however, it had a very high specificity—ie, greater than 95%.8
MRCP has a sensitivity of 82.6% and a specificity of 97.5% in detecting stones in the common bile duct.9 Therefore, normal results on abdominal ultrasonography and MRCP do not completely rule out stones.
Although this patient has a high pretest probability of having common bile duct stones, ERCP should be done only after a thorough review of the previous operative procedure.
Observation and reassurance are not appropriate in a patient with cholangitis, such as this patient, because waiting increases the risk of septicemia.
The patient undergoes ERCP with stone removal
Review of the operative report and discussion with the surgeon confirm that the laparoscopic procedure was uneventful and that intraoperative cholangiography was not done.
Therefore, the patient undergoes ERCP. The major papilla is normal. Cholangiography reveals nondilated common bile and intrahepatic ducts, with faint filling defects in the mid to distal common bile duct. Endoscopic sphincterotomy is performed, and three small stones are extracted from the common bile duct. Repeat balloon-occlusion cholangiography is normal.
The patient tolerates the procedure well and resumes a normal diet and normal activities.
Her pain persists, prompting an emergency room visit
Five days after her ERCP procedure, however, the same burning epigastric pain returns. As before, the pain occurs after eating and does not occur with fasting. At this time, she has no fever or chills.
The patient continues to have recurrent episodes of pain, on one occasion so severe she visits the emergency department. During this visit she reports she has no symptoms other than pain, and the examination is normal. Laboratory tests (Table 2)show that her liver function measures have normalized.
WHAT IS CAUSING HER PAIN?
3. Which is the most likely cause of her persistent pain?
Acute pancreatitis after ERCP
Peptic ulcer disease
Sphincter of Oddi dysfunction
Biliary stones
The most likely cause is persistent biliary stones. The common bile duct was recently explored and stones were removed, but she may still have stones in the intrahepatic ducts or in the cystic duct remnant, both of which were unopacified during the ERCP procedure, indicating that either the test was incomplete or a stone is obstructing the passage of contrast. Her persistent symptoms warrant repeating her liver function tests.
Acute pancreatitis is the most common and feared complication of ERCP, and it should be suspected in any patient who develops abdominal pain within 6 hours of the procedure. It is much less likely to develop after 12 hours, however. Risk factors for post-ERCP pancreatitis include patient factors (young age, female sex, history of recurrent pancreatitis), procedural factors (difficult cannulation, minor papilla sphincterotomy), and, less likely, operator-related factors.10–13 In general, the more likely a patient is to have an abnormal and irregular common bile duct or pancreatic duct, the lower the risk of post-ERCP pancreatitis. The importance of operator-dependent factors is not yet clear.10–13
Despite the postprandial pattern of our patient’s pain and her history of gastric ulcer, peptic ulcer disease is unlikely in view of a normal esophagogastroduodenoscopic examination done 4 months earlier, and since she has no recent exposure to NSAIDs.
Sphincter of Oddi dysfunction may explain her symptoms, but she recently underwent endoscopic sphincterotomy, which is regarded as the most definitive treatment.14
WHAT SHOULD BE DONE NEXT?
4. What would be the best next step in her management?
Repeat ERCP
MRCP
Endoscopic ultrasonography
Observation and reassurance
MRCP is the most appropriate next step, given her recurrent symptoms. Repeat ERCP is not appropriate, since there is no evidence of cholangitis, and since her liver function tests had completely normalized.
A recent systematic review of endoscopic ultrasonography and MRCP for diagnosing choledocholithiasis found both tests to be highly accurate, with no statistically significant differences in sensitivity or specificity between the two.15 However, MRCP has the advantage of being noninvasive and of being able to show intrahepatic stones.
Park et al,16 in a prospective study of 66 patients with primary intrahepatic stones, concluded that MRCP findings were comparable to those of percutaneous transhepatic cholangioscopy, the reference standard for locating intrahepatic stones. The sensitivity, specificity, and accuracy of MRCP for detecting and locating intrahepatic stones were high (97%, 99%, and 98%, respectively).16 However, after sphincterotomy, pneumobilia may create an appearance that can be mistaken for intraductal stones.
Figure 1. Magnetic resonance cholangiopancreatography shows a normal biliary tree (arrow) and pancreatic duct. The cystic duct cannot be seen.Merely reassuring the patient is not appropriate at this point, given her level of pain.
She undergoes MRCP
Figure 2. Ultrasonography of the right upper quadrant shows a nondilated common bile duct 4 mm in diameter (arrow). No stones are visible.MRCP shows a normal biliary tree without stones (Figure 1). Similarly, ultrasonography of the right upper quadrant shows no stones and a nondilated common bile duct (Figure 2).
The patient continues to have pain, and she has lost 5 pounds because she is still avoiding eating. At this point, she is beginning to wonder if her symptoms are psychogenic, since all the test results have been normal.
ERCP, MRCP, ULTRASONOGRAPHY?
5.What would be the best next step?
Reassurance
Referral to a psychiatrist
Referral to a pain management clinic
Endoscopic ultrasonography
Repeat ERCP
Endoscopic ultrasonography is needed to look for cystic duct stones. Although several tests have shown normal results, the patient’s pain continues as in the previous episodes, making stone disease the most likely cause.
Although no stones were seen on MRCP and ultrasonography, a detailed evaluation for stones in a cystic duct or retained gallbladder remnant was not done satisfactorily.
Reassurance and referral to a psychiatrist or pain management clinic are not appropriate, since an organic cause of her pain has not been completely ruled out.
Figure 3. Endoscopic ultrasonography from the duodenal bulb shows a 7-mm stone (arrow) in the cystic duct remnant or gallbladder remnant.ERCP should not be used as a diagnostic test in a situation such as this.
Findings on endoscopic ultrasonography
Endoscopic ultrasonography is performed and reveals a large (7-mm) stone in the area of the cystic duct remnant or gallbladder remnant (Figure 3). The common bile duct is normal.
CAUSES OF RETAINED GALLBLADDER AND CYSTIC DUCT REMNANT
6.What may have predisposed this patient to a retained gallbladder or cystic duct remnant after her surgery?
Laparoscopic cholecystectomy
Not doing intraoperative cholangiography
Cholecystectomy for acute cholecystitis
All of the above
All of the above may have contributed.
Postcholecystectomy syndrome can pose a diagnostic and therapeutic challenge, as in our patient. Although it has been reported since the advent of the operation, it is more common after laparoscopic cholecystectomy than after open surgery. One possible cause is stones in a cystic duct remnant, ie, a stub longer than 1 cm.
During open cholecystectomy, the cystic duct is ligated and cut as close to the common bile duct as possible, leaving only a small remnant. In laparoscopic cholecystectomy, it is divided closer to the gallbladder to avoid iatrogenic injury to the common bile duct, leaving a longer remnant. A long cystic duct remnant can be prevented by accurately locating the junction of the gallbladder and the cystic duct during cholecystectomy and by routinely doing intraoperative cholangiography. The presence of stones in a cystic duct or retained gallbladder remnant is a rare cause of postcholecystectomy syndrome, and suspicion is required to make the diagnosis.17–19
We should note that stones may also lurk in the short cystic duct remnant left after open cholecystectomy. In fact, the first case of cystic duct remnant, the so-called reformed gallbladder containing stones, was described in 1912 by Flörcken.20
Intraoperative cholangiography was introduced in 1931 by Mirizzi,21 who recommended its routine use. Since the advent of laparoscopic cholecystectomy in 1988, the routine use of intraoperative cholangiography has been debated. Advocates point to its ability to detect unsuspected calculi and to delineate the biliary anatomy, thus reducing the risk of biliary duct injury.7,22–25 Those who argue against its routine use emphasize the low reported rates of unsuspected stones in the common bile duct (2% to 3%), a longer operative time, the additional cost, and false-positive results that may lead to unnecessary common bile duct exploration. Another argument against its routine use is that most small ductal stones pass spontaneously without significant sequelae.26–28 Surgeons who use intraoperative cholangiography only selectively use it in patients with unclear biliary anatomy and preoperative biochemical or radiologic evidence of choledocholithiasis.
Figure 4. Endoscopic retrograde cholangiopancreatography shows an oval filling defect in the cystic duct remnant at its insertion into the common bile duct (arrow).Another potential explanation for the retained gallbladder remnant is that the cholecystectomy was done while the patient had acute cholecystitis, in which inflammation may obscure anatomic landmarks. Hence, cholangiography during laparoscopic cholecystectomy has been widely recognized as a means of delineating the biliary anatomy.
Case continued: She undergoes repeat ERCP
Figure 5. Endoscopic retrograde cholangiopancreatography reveals a long duct remnant (red arrow) and a small gallbladder remnant (black arrow). The stone has already been extracted.The patient undergoes ERCP again (Figure 4 and Figure 5). Cholangiography shows a normal common bile duct with low insertion of the cystic duct and an oval filling defect in the cystic duct just proximal to its insertion into the common bile duct. Cystic duct opacification reveals a long cystic duct remnant and a small gallbladder remnant. The stone in the cystic duct is successfully removed.
IF STONES ARE DIFFICULT TO EXTRACT
7. If the cystic duct stone were not amenable to endoscopic extraction, what would be the best alternative?
Extracorporeal shock-wave lithotripsy (ESWL)
Endoscopic biliary laser lithotripsy
Repeat laparoscopic cholecystectomy
All of the above
All of the above are alternatives.
A symptomatic stone in a cystic duct remnant is uncommon and is mentioned in the literature only in case series and case reports.
ESWL is effective for treating bile duct calculi.29 In a cohort of 239 patients with bile duct stones treated by ESWL, Benninger et al30 concluded that endoscopy plus ESWL was a definitive treatment for all patients except one, who subsequently underwent cholecystectomy. Once fragmented, the stones are extracted endoscopically.
Another fragmentation technique that can be offered to patients with stones in the cystic duct that are difficult to extract is contact fragmentation with a holmium laser placed in a transpapillary position under visual guidance.17
Repeat cholecystectomy with removal of stones in the cystic duct remnant (and removal of retained gallbladder remnants and reduction of the cystic duct remnant) has good postoperative results.17,18,31,32
After incomplete cholecystectomy, the cystic duct remnant and the Calot (cystohepatic) triangle are surrounded by inflamed scar tissue, and this was thought to make laparoscopic reoperation difficult.33 However, with advances in surgical technique and increasing experience of surgeons, repeat cholecystectomy can be done laparoscopically. It has now been suggested that laparoscopic exploration to remove the gallbladder remnants is safe and feasible in such patients.34,35
Discharge and follow-up
The patient is discharged home after the procedure. She is still free of symptoms 31 months later.
LESSONS LEARNED
Remnant cystic duct stones are uncommon
The estimated incidence of a retained calculus within the cystic duct remnant after cholecystectomy is less than 2.5%.2,36 In a series of 322 patients who underwent repeat surgery because of postcholecystectomy syndrome, Rogy et al36 found only 8 who had a stone in the cystic duct or gallbladder remnant, and in a series of 371 patients, Zhou el al2 found 4 who had a stone in the cystic duct remnant.
Stones in the cystic duct remnant are difficult to diagnose
Diagnosing stones in surgical remnants of the cystic duct or gallbladder can be difficult. The sensitivity of abdominal ultrasonography in detecting cystic duct stones is low—only 27% in one study, with a specificity of 100% and an accuracy of 75%.37 Ultrasonography may occasionally suggest cystic duct stones by showing an acoustic shadow in the anatomic region of the cystic duct. However, the results should be interpreted with caution.
Determining the accuracy of ERCP and MRCP in detecting cystic duct remnant stones is also difficult, as few cases have been reported and data may be conflicting. In a review of seven patients confirmed to have retained stones in a surgical remnant, Walsh et al17 found that ERCP correctly diagnosed the retained stone in only four out of six patients; MRCP was done in one patient, and it was read as normal.
In three cases of stones in a postsurgical gallbladder remnant, Hassan and Vilmann38 reported that ERCP and MRCP failed to identify the gallbladder remnant in two out of three cases, likely because the remaining structures are small. The diagnosis was finally made by endoscopic ultrasonography, which the authors concluded was a valuable method to visualize a small gallbladder remnant with stones.
Greater suspicion is needed in patients with typical biliary colic after cholecystectomy
Retained gallbladder remnant is described in the literature as a latent complication. The main problem is not the remnant itself but the chance that it harbors retained stones, which can lead to dilatation and inflammation of the remnant.
The patient can develop symptoms of acute cholecystitis or even acute cholangitis if the stone migrates to the common bile duct. Symptoms can develop as early as 2 weeks or as late as 25 years after laparoscopic cholecystectomy.
Endoscopic ultrasonography may be the best way to look for these remnant stones and to evaluate the bile duct and pancreas. Therefore, it should be part of the diagnostic algorithm in the evaluation of postcholecystectomy pain.
Mixed results with ERCP for extracting cystic duct stones
In case reports of cystic duct calculi after cholecystectomy, ERCP by itself has had mixed results. This traditional means of removing stones may succeed, as in our case. However, the success rate depends largely on anatomic factors such as the position of the stone in the cystic duct, the degree of stone impaction, the diameter of the cystic duct, and the number of valves in the duct.17
Stones in the cystic duct that cannot be extracted with ERCP may benefit from fragmentation techniques in situ via holmium laser followed by endoscopic extraction.
Repeat cholecystectomy is generally advised for any residual gallbladder, and it can be done laparoscopically.
References
Lehman GA, Sherman S. Sphincter of Oddi dysfunction (postcholecystectomy syndrome). In:Yamada T, editor. Textbook of Gastroenterology. 2nd ed. Philadelphia: Lippincott; 1995:2251–2262.
Zhou PH, Liu FL, Yao LQ, Qin XY. Endoscopic diagnosis and treatment of post-cholecystectomy syndrome. Hepatobiliary Pancreat Dis Int2003; 2:117–120.
Mergener K, Clavien PA, Branch MS, Baillie J. A stone in a grossly dilated cystic duct stump: a rare cause of postcholecystectomy pain. Am J Gastroenterol1999; 94:229–231.
Goenka MK, Kochhar R, Nagi B, Bhasin DK, Chowdhury A, Singh K. Endoscopic retrograde cholangiopancreatography in postcholecystectomy syndrome. J Assoc Physicians India1996; 44:119–122.
Bodvall B, Overgaard B. Cystic duct remnant after cholecystectomy: incidence studied by cholegraphy in 500 cases, and significance in 103 reoperations. Ann Surg1966; 163:382–390.
Bergman JJ, van den Brink GR, Rauws EA, et al. Treatment of bile duct lesions after laparoscopic cholecystectomy. Gut1996; 38:141–147.
Nickkholgh A, Soltaniyekta S, Kalbasi H. Routine versus selective intraoperative cholangiography during laparoscopic cholecystectomy: a survey of 2,130 patients undergoing laparoscopic cholecystectomy. Surg Endosc2006; 20:868–874.
Gandolfi L, Torresan F, Solmi L, Puccetti A. The role of ultrasound in biliary and pancreatic diseases. Eur J Ultrasound2003; 16:141–159.
Al Samaraee A, Khan U, Almashta Z, Yiannakou Y. Preoperative diagnosis of choledocholithiasis: the role of MRCP. Br J Hosp Med (Lond)2009; 70:339–343.
Freeman ML, DiSario JA, Nelson DB, et al. Risk factors for post-ERCP pancreatitis: a prospective, multicenter study. Gastrointest Endosc2001; 54:425–434.
Cheng CL, Sherman S, Watkins JL, et al. Risk factors for post-ERCP pancreatitis: a prospective multicenter study. Am J Gastroenterol2006; 101:139–147.
Mehta SN, Pavone E, Barkun JS, Bouchard S, Barkun AN. Predictors of post-ERCP complications in patients with suspected choledocholithiasis. Endoscopy1998; 30:457–463.
Badalov N, Tenner S, Baillie J. The prevention, recognition and treatment of post-ERCP pancreatitis. JOP2009; 10:88–97.
Geenen JE, Hogan WJ, Dodds WJ, Toouli J, Venu RP. The efficacy of endoscopic sphincterotomy after cholecystectomy in patients with sphincter-of-Oddi dysfunction. N Engl J Med1989; 320:82–87.
Verma D, Kapadia A, Eisen GM, Adler DG. EUS vs MRCP for detection of choledocholithiasis. Gastrointest Endosc2006; 64:248–254.
Park DH, Kim MH, Lee SS, et al. Accuracy of magnetic resonance cholangiopancreatography for locating hepatolithiasis and detecting accompanying biliary strictures. Endoscopy2004; 36:987–992.
Walsh RM, Ponsky JL, Dumot J. Retained gallbladder/cystic duct remnant calculi as a cause of postcholecystectomy pain. Surg Endosc2002; 16:981–984.
Tantia O, Jain M, Khanna S, Sen B. Post cholecystectomy syndrome: role of cystic duct stump and re-intervention by laparoscopic surgery. J Minim Access Surg2008; 4:71–75.
Palanivelu C, Rangarajan M, Jategaonkar PA, Madankumar MV, Anand NV. Laparoscopic management of remnant cystic duct calculi: a retrospective study. Ann R Coll Surg Engl2009; 91:25–29.
Flörcken H. Gallenblasenregeneration mit Steinrecidiv nach Cholecystectomie. Deutsch Z Chir1912; 113:604.
Mirizzi PL. La colangiografía durante las operaciones de las vias biliares. Bol Soc Cirug Buenos Aires1932; 16:1113.
Soper NJ, Brunt LM. The case for routine operative cholangiography during laparoscopic cholecystectomy. Surg Clin North Am1994; 74:953–959.
Cuschieri A, Shimi S, Banting S, Nathanson LK, Pietrabissa A. Intraoperative cholangiography during laparoscopic cholecystectomy. Routine vs selective policy. Surg Endosc1994; 8:302–305.
Woods MS, Traverso LW, Kozarek RA, et al. Biliary tract complications of laparoscopic cholecystectomy are detected more frequently with routine intraoperative cholangiography. Surg Endosc1995; 9:1076–1080.
Vezakis A, Davides D, Ammori BJ, Martin IG, Larvin M, McMahon MJ. Intraoperative cholangiography during laparoscopic cholecystectomy. Surg Endosc2000; 14:1118–1122.
Ladocsi LT, Benitez LD, Filippone DR, Nance FC. Intraoperative cholangiography in laparoscopic cholecystectomy: a review of 734 consecutive cases. Am Surg1997; 63:150–156.
Clair DG, Brooks DC. Laparoscopic cholangiography. The case for a selective approach. Surg Clin North Am1994; 74:961–966.
Collins C, Maguire D, Ireland A, Fitzgerald E, O’Sullivan GC. A prospective study of common bile duct calculi in patients undergoing laparoscopic cholecystectomy: natural history of choledocholithiasis revisited. Ann Surg2004; 239:28–33.
Ponsky LE, Geisinger MA, Ponsky JL, Streem SB. Contemporary ‘urologic’ intervention in the pancreaticobiliary tree. Urology2001; 57:21–25.
Benninger J, Rabenstein T, Farnbacher M, Keppler J, Hahn EG, Schneider HT. Extracorporeal shockwave lithotripsy of gallstones in cystic duct remnants and Mirizzi syndrome. Gastrointest Endosc2004; 60:454–459.
Demetriades H, Pramateftakis MG, Kanellos I, Angelopoulos S, Mantzoros I, Betsis D. Retained gallbladder remnant after laparoscopic cholecystectomy. J Laparoendosc Adv Surg Tech A2008; 18:276–279.
Shaw C, O’Hanlon DM, Fenlon HM, McEntee GP. Cystic duct remnant and the ‘post-cholecystectomy syndrome. ’ Hepatogastroenterology2004; 51:36–38.
Rozsos I, Magyaródi Z, Orbán P. Cystic duct syndrome and minimally invasive surgery. [Hungarian] Orv Hetil1997; 138:2397–2401.
Chowbey PK, Bandyopadhyay SK, Sharma A, Khullar R, Soni V, Baijal M. Laparoscopic reintervention for residual gallstone disease. Surg Laparosc Endosc Percutan Tech2003; 13:31–35.
Clemente G, Giuliante F, Cadeddu F, Nuzzo G. Laparoscopic removal of gallbladder remnant and long cystic stump. Endoscopy2001; 33:814–815.
Rogy MA, Függer R, Herbst F, Schulz F. Reoperation after cholecystectomy. The role of the cystic duct stump. HPB Surg1991; 4:129–134.
CArole Macaron, MD Department of Internal Medicine, Cleveland Clinic
Mohammed A. Qadeer, MD, MPH Department of Gastroenterology and Hepatology, Cleveland Clinic
John J. Vargo, MD, MPH Department of Gastroenterology and Hepatology, Cleveland Clinic
Address: John J. Vargo, MD, MPH, Section of Endoscopy, Department of Gastroenterology and Hepatology, Digestive Disease Institute, A30, Cleveland Clinic, 9500 Euclid Avenue, Cleveland, OH 44195; e-mail vargoj@ccf.org
CArole Macaron, MD Department of Internal Medicine, Cleveland Clinic
Mohammed A. Qadeer, MD, MPH Department of Gastroenterology and Hepatology, Cleveland Clinic
John J. Vargo, MD, MPH Department of Gastroenterology and Hepatology, Cleveland Clinic
Address: John J. Vargo, MD, MPH, Section of Endoscopy, Department of Gastroenterology and Hepatology, Digestive Disease Institute, A30, Cleveland Clinic, 9500 Euclid Avenue, Cleveland, OH 44195; e-mail vargoj@ccf.org
Author and Disclosure Information
CArole Macaron, MD Department of Internal Medicine, Cleveland Clinic
Mohammed A. Qadeer, MD, MPH Department of Gastroenterology and Hepatology, Cleveland Clinic
John J. Vargo, MD, MPH Department of Gastroenterology and Hepatology, Cleveland Clinic
Address: John J. Vargo, MD, MPH, Section of Endoscopy, Department of Gastroenterology and Hepatology, Digestive Disease Institute, A30, Cleveland Clinic, 9500 Euclid Avenue, Cleveland, OH 44195; e-mail vargoj@ccf.org
Four months after undergoing laparoscopic cholecystectomy for symptomatic gallstones, an otherwise healthy 26-year-old woman begins to have episodes of epigastric and back pain similar to what she experienced before the surgery. The surgery was without complications, and her classic biliary colic disappeared afterward. Histologic evaluation of the surgical specimen revealed chronic cholecystitis with multiple small, mixed gallstones.
Now she describes a burning pain in her epigastrium and mid to upper back, starting about 30 minutes after a meal and lasting up to 4 hours. Sometimes it awakens her at night. She avoids eating for fear of inducing the pain. She has occasional chills but no fever, nausea, vomiting, jaundice, or changes in urine or stool color.
Three years ago she was diagnosed with a gastric ulcer induced by taking a nonsteroidal anti-inflammatory drug (NSAID). The ulcer was treated with a proton pump inhibitor for 1 month. She says the ulcer pain was dull and aching, different from her current pain.
Upper endoscopy 4 months ago (ie, before her laparoscopic cholecystectomy) showed no evidence of esophagitis or peptic ulcer disease.
Apart from her gallbladder operation, she has had no other surgery. According to the surgeon’s notes, intraoperative cholangiography was not performed, and no macroscopic changes of acute cholecystitis or difficult biliary anatomy were noted.
The patient does not smoke, does not drink alcohol, is not currently taking any medications, including NSAIDs or over-the-counter medications, and has not taken any recently. Her mother also had symptomatic gallstones requiring cholecystectomy.
On physical examination, only fever
On examination, her temperature is 101.2°F (38.4°C), blood pressure 117/80 mm Hg, heart rate 82 beats per minute, and blood oxygen saturation 99% on room air. Her weight is 138 lb (62.6 kg), height 5 feet 6 inches (168 cm).
There is no jaundice or pallor. Her heart and lung examinations are normal.
Her abdomen is soft and mildly tender to palpation of the epigastrium, with no distention or hepatosplenomegaly and no rebound tenderness or guarding. The scars from her laparoscopic surgery have healed well. Her bowel sounds are normal.
No costovertebral angle or spinal tenderness can be elicited.
Her laboratory values are shown in Table 1.
POSTCHOLECYSTECTOMY SYNDROME
1.After cholecystectomy, preoperative symptoms recur in what percentage of patients?
10% to 40%
50%
60%
80%
Postcholecystectomy syndrome—the recurrence of symptoms similar to those before the procedure—occurs in 10% to 40% of patients. The time to the onset of symptoms can range from 2 days to up to 25 years.1–4 Women may be at higher risk, with symptoms recurring in 43% vs 28% in men.5
Postcholecystectomy syndrome can have a biliary or a nonbiliary cause. Biliary causes include strictures, retained calculi, dropped calculi, tumors, sphincter of Oddi dysfunction, and calculi in the cystic duct remnant. Nonbiliary causes include functional and organic disorders such as peptic ulcer disease, gastroesophageal reflux, pancreatic disease, hepatocellular disorders, coronary artery disease, irritable bowel syndrome, and intercostal neuritis.
WHAT IS THE NEXT STEP?
2.Which is the most appropriate next step in the workup of this patient?
Ultrasonography of the right upper quadrant
Magnetic resonance cholangiopancreatography (MRCP)
Review the operative record and consult with the surgeon
Although the patient is presenting with pain and fever, two features of the classic Charcot triad (pain, fever, jaundice) seen in cholangitis (infection of a bile duct), and although cholangitis almost confirms the diagnosis of common bile duct stones in a patient with gallstones (before or after cholecystectomy), other diagnoses to consider are bile duct injury, bile leak, and biloma.
Biloma can be detected with ultrasonography. Bile duct injuries are identified intraoperatively in up to 25% of patients. For those with an unrecognized injury, the clinical presentation is variable and depends on the type of injury. If a bile leak is present, patients present early, at a median of 3 days postoperatively. However, our patient presented with symptoms 4 months after her surgery. Patients with bile duct strictures without bile leak have a longer symptom-free interval and usually present with signs of biliary obstruction. Ultrasonography can then detect biliary dilatation.6
It would be very helpful to review the operative record and to talk to the surgeon to confirm that intraoperative cholangiography had not been done and to determine the level of difficulty of the surgery. (Intraoperative cholangiography involves the introduction of contrast dye into the biliary system by cannulation of the cystic duct or by direct injection into the common bile duct. An intraoperative cholangiogram is considered normal if the entire intrahepatic and extrahepatic biliary tree is seen to be filled with contrast.) A normal cholangiogram has a negative predictive value of 99.8% for the detection of ductal stones. Thus, a normal intraoperative cholangiogram can prevent unnecessary postoperative ECRP, since it almost always indicates a clean bile duct.7
Ultrasonography of the right upper quadrant has a low sensitivity (< 50%) for detecting common bile duct stones. However, it is highly operator-dependent, and it may be twice as sensitive if done by expert radiologists than by less experienced ones. Its limitations include poor visualization of the distal portion of the duct and low sensitivity in patients in whom the common bile duct is minimally dilated and also in patients with small stones. In most studies, however, it had a very high specificity—ie, greater than 95%.8
MRCP has a sensitivity of 82.6% and a specificity of 97.5% in detecting stones in the common bile duct.9 Therefore, normal results on abdominal ultrasonography and MRCP do not completely rule out stones.
Although this patient has a high pretest probability of having common bile duct stones, ERCP should be done only after a thorough review of the previous operative procedure.
Observation and reassurance are not appropriate in a patient with cholangitis, such as this patient, because waiting increases the risk of septicemia.
The patient undergoes ERCP with stone removal
Review of the operative report and discussion with the surgeon confirm that the laparoscopic procedure was uneventful and that intraoperative cholangiography was not done.
Therefore, the patient undergoes ERCP. The major papilla is normal. Cholangiography reveals nondilated common bile and intrahepatic ducts, with faint filling defects in the mid to distal common bile duct. Endoscopic sphincterotomy is performed, and three small stones are extracted from the common bile duct. Repeat balloon-occlusion cholangiography is normal.
The patient tolerates the procedure well and resumes a normal diet and normal activities.
Her pain persists, prompting an emergency room visit
Five days after her ERCP procedure, however, the same burning epigastric pain returns. As before, the pain occurs after eating and does not occur with fasting. At this time, she has no fever or chills.
The patient continues to have recurrent episodes of pain, on one occasion so severe she visits the emergency department. During this visit she reports she has no symptoms other than pain, and the examination is normal. Laboratory tests (Table 2)show that her liver function measures have normalized.
WHAT IS CAUSING HER PAIN?
3. Which is the most likely cause of her persistent pain?
Acute pancreatitis after ERCP
Peptic ulcer disease
Sphincter of Oddi dysfunction
Biliary stones
The most likely cause is persistent biliary stones. The common bile duct was recently explored and stones were removed, but she may still have stones in the intrahepatic ducts or in the cystic duct remnant, both of which were unopacified during the ERCP procedure, indicating that either the test was incomplete or a stone is obstructing the passage of contrast. Her persistent symptoms warrant repeating her liver function tests.
Acute pancreatitis is the most common and feared complication of ERCP, and it should be suspected in any patient who develops abdominal pain within 6 hours of the procedure. It is much less likely to develop after 12 hours, however. Risk factors for post-ERCP pancreatitis include patient factors (young age, female sex, history of recurrent pancreatitis), procedural factors (difficult cannulation, minor papilla sphincterotomy), and, less likely, operator-related factors.10–13 In general, the more likely a patient is to have an abnormal and irregular common bile duct or pancreatic duct, the lower the risk of post-ERCP pancreatitis. The importance of operator-dependent factors is not yet clear.10–13
Despite the postprandial pattern of our patient’s pain and her history of gastric ulcer, peptic ulcer disease is unlikely in view of a normal esophagogastroduodenoscopic examination done 4 months earlier, and since she has no recent exposure to NSAIDs.
Sphincter of Oddi dysfunction may explain her symptoms, but she recently underwent endoscopic sphincterotomy, which is regarded as the most definitive treatment.14
WHAT SHOULD BE DONE NEXT?
4. What would be the best next step in her management?
Repeat ERCP
MRCP
Endoscopic ultrasonography
Observation and reassurance
MRCP is the most appropriate next step, given her recurrent symptoms. Repeat ERCP is not appropriate, since there is no evidence of cholangitis, and since her liver function tests had completely normalized.
A recent systematic review of endoscopic ultrasonography and MRCP for diagnosing choledocholithiasis found both tests to be highly accurate, with no statistically significant differences in sensitivity or specificity between the two.15 However, MRCP has the advantage of being noninvasive and of being able to show intrahepatic stones.
Park et al,16 in a prospective study of 66 patients with primary intrahepatic stones, concluded that MRCP findings were comparable to those of percutaneous transhepatic cholangioscopy, the reference standard for locating intrahepatic stones. The sensitivity, specificity, and accuracy of MRCP for detecting and locating intrahepatic stones were high (97%, 99%, and 98%, respectively).16 However, after sphincterotomy, pneumobilia may create an appearance that can be mistaken for intraductal stones.
Figure 1. Magnetic resonance cholangiopancreatography shows a normal biliary tree (arrow) and pancreatic duct. The cystic duct cannot be seen.Merely reassuring the patient is not appropriate at this point, given her level of pain.
She undergoes MRCP
Figure 2. Ultrasonography of the right upper quadrant shows a nondilated common bile duct 4 mm in diameter (arrow). No stones are visible.MRCP shows a normal biliary tree without stones (Figure 1). Similarly, ultrasonography of the right upper quadrant shows no stones and a nondilated common bile duct (Figure 2).
The patient continues to have pain, and she has lost 5 pounds because she is still avoiding eating. At this point, she is beginning to wonder if her symptoms are psychogenic, since all the test results have been normal.
ERCP, MRCP, ULTRASONOGRAPHY?
5.What would be the best next step?
Reassurance
Referral to a psychiatrist
Referral to a pain management clinic
Endoscopic ultrasonography
Repeat ERCP
Endoscopic ultrasonography is needed to look for cystic duct stones. Although several tests have shown normal results, the patient’s pain continues as in the previous episodes, making stone disease the most likely cause.
Although no stones were seen on MRCP and ultrasonography, a detailed evaluation for stones in a cystic duct or retained gallbladder remnant was not done satisfactorily.
Reassurance and referral to a psychiatrist or pain management clinic are not appropriate, since an organic cause of her pain has not been completely ruled out.
Figure 3. Endoscopic ultrasonography from the duodenal bulb shows a 7-mm stone (arrow) in the cystic duct remnant or gallbladder remnant.ERCP should not be used as a diagnostic test in a situation such as this.
Findings on endoscopic ultrasonography
Endoscopic ultrasonography is performed and reveals a large (7-mm) stone in the area of the cystic duct remnant or gallbladder remnant (Figure 3). The common bile duct is normal.
CAUSES OF RETAINED GALLBLADDER AND CYSTIC DUCT REMNANT
6.What may have predisposed this patient to a retained gallbladder or cystic duct remnant after her surgery?
Laparoscopic cholecystectomy
Not doing intraoperative cholangiography
Cholecystectomy for acute cholecystitis
All of the above
All of the above may have contributed.
Postcholecystectomy syndrome can pose a diagnostic and therapeutic challenge, as in our patient. Although it has been reported since the advent of the operation, it is more common after laparoscopic cholecystectomy than after open surgery. One possible cause is stones in a cystic duct remnant, ie, a stub longer than 1 cm.
During open cholecystectomy, the cystic duct is ligated and cut as close to the common bile duct as possible, leaving only a small remnant. In laparoscopic cholecystectomy, it is divided closer to the gallbladder to avoid iatrogenic injury to the common bile duct, leaving a longer remnant. A long cystic duct remnant can be prevented by accurately locating the junction of the gallbladder and the cystic duct during cholecystectomy and by routinely doing intraoperative cholangiography. The presence of stones in a cystic duct or retained gallbladder remnant is a rare cause of postcholecystectomy syndrome, and suspicion is required to make the diagnosis.17–19
We should note that stones may also lurk in the short cystic duct remnant left after open cholecystectomy. In fact, the first case of cystic duct remnant, the so-called reformed gallbladder containing stones, was described in 1912 by Flörcken.20
Intraoperative cholangiography was introduced in 1931 by Mirizzi,21 who recommended its routine use. Since the advent of laparoscopic cholecystectomy in 1988, the routine use of intraoperative cholangiography has been debated. Advocates point to its ability to detect unsuspected calculi and to delineate the biliary anatomy, thus reducing the risk of biliary duct injury.7,22–25 Those who argue against its routine use emphasize the low reported rates of unsuspected stones in the common bile duct (2% to 3%), a longer operative time, the additional cost, and false-positive results that may lead to unnecessary common bile duct exploration. Another argument against its routine use is that most small ductal stones pass spontaneously without significant sequelae.26–28 Surgeons who use intraoperative cholangiography only selectively use it in patients with unclear biliary anatomy and preoperative biochemical or radiologic evidence of choledocholithiasis.
Figure 4. Endoscopic retrograde cholangiopancreatography shows an oval filling defect in the cystic duct remnant at its insertion into the common bile duct (arrow).Another potential explanation for the retained gallbladder remnant is that the cholecystectomy was done while the patient had acute cholecystitis, in which inflammation may obscure anatomic landmarks. Hence, cholangiography during laparoscopic cholecystectomy has been widely recognized as a means of delineating the biliary anatomy.
Case continued: She undergoes repeat ERCP
Figure 5. Endoscopic retrograde cholangiopancreatography reveals a long duct remnant (red arrow) and a small gallbladder remnant (black arrow). The stone has already been extracted.The patient undergoes ERCP again (Figure 4 and Figure 5). Cholangiography shows a normal common bile duct with low insertion of the cystic duct and an oval filling defect in the cystic duct just proximal to its insertion into the common bile duct. Cystic duct opacification reveals a long cystic duct remnant and a small gallbladder remnant. The stone in the cystic duct is successfully removed.
IF STONES ARE DIFFICULT TO EXTRACT
7. If the cystic duct stone were not amenable to endoscopic extraction, what would be the best alternative?
Extracorporeal shock-wave lithotripsy (ESWL)
Endoscopic biliary laser lithotripsy
Repeat laparoscopic cholecystectomy
All of the above
All of the above are alternatives.
A symptomatic stone in a cystic duct remnant is uncommon and is mentioned in the literature only in case series and case reports.
ESWL is effective for treating bile duct calculi.29 In a cohort of 239 patients with bile duct stones treated by ESWL, Benninger et al30 concluded that endoscopy plus ESWL was a definitive treatment for all patients except one, who subsequently underwent cholecystectomy. Once fragmented, the stones are extracted endoscopically.
Another fragmentation technique that can be offered to patients with stones in the cystic duct that are difficult to extract is contact fragmentation with a holmium laser placed in a transpapillary position under visual guidance.17
Repeat cholecystectomy with removal of stones in the cystic duct remnant (and removal of retained gallbladder remnants and reduction of the cystic duct remnant) has good postoperative results.17,18,31,32
After incomplete cholecystectomy, the cystic duct remnant and the Calot (cystohepatic) triangle are surrounded by inflamed scar tissue, and this was thought to make laparoscopic reoperation difficult.33 However, with advances in surgical technique and increasing experience of surgeons, repeat cholecystectomy can be done laparoscopically. It has now been suggested that laparoscopic exploration to remove the gallbladder remnants is safe and feasible in such patients.34,35
Discharge and follow-up
The patient is discharged home after the procedure. She is still free of symptoms 31 months later.
LESSONS LEARNED
Remnant cystic duct stones are uncommon
The estimated incidence of a retained calculus within the cystic duct remnant after cholecystectomy is less than 2.5%.2,36 In a series of 322 patients who underwent repeat surgery because of postcholecystectomy syndrome, Rogy et al36 found only 8 who had a stone in the cystic duct or gallbladder remnant, and in a series of 371 patients, Zhou el al2 found 4 who had a stone in the cystic duct remnant.
Stones in the cystic duct remnant are difficult to diagnose
Diagnosing stones in surgical remnants of the cystic duct or gallbladder can be difficult. The sensitivity of abdominal ultrasonography in detecting cystic duct stones is low—only 27% in one study, with a specificity of 100% and an accuracy of 75%.37 Ultrasonography may occasionally suggest cystic duct stones by showing an acoustic shadow in the anatomic region of the cystic duct. However, the results should be interpreted with caution.
Determining the accuracy of ERCP and MRCP in detecting cystic duct remnant stones is also difficult, as few cases have been reported and data may be conflicting. In a review of seven patients confirmed to have retained stones in a surgical remnant, Walsh et al17 found that ERCP correctly diagnosed the retained stone in only four out of six patients; MRCP was done in one patient, and it was read as normal.
In three cases of stones in a postsurgical gallbladder remnant, Hassan and Vilmann38 reported that ERCP and MRCP failed to identify the gallbladder remnant in two out of three cases, likely because the remaining structures are small. The diagnosis was finally made by endoscopic ultrasonography, which the authors concluded was a valuable method to visualize a small gallbladder remnant with stones.
Greater suspicion is needed in patients with typical biliary colic after cholecystectomy
Retained gallbladder remnant is described in the literature as a latent complication. The main problem is not the remnant itself but the chance that it harbors retained stones, which can lead to dilatation and inflammation of the remnant.
The patient can develop symptoms of acute cholecystitis or even acute cholangitis if the stone migrates to the common bile duct. Symptoms can develop as early as 2 weeks or as late as 25 years after laparoscopic cholecystectomy.
Endoscopic ultrasonography may be the best way to look for these remnant stones and to evaluate the bile duct and pancreas. Therefore, it should be part of the diagnostic algorithm in the evaluation of postcholecystectomy pain.
Mixed results with ERCP for extracting cystic duct stones
In case reports of cystic duct calculi after cholecystectomy, ERCP by itself has had mixed results. This traditional means of removing stones may succeed, as in our case. However, the success rate depends largely on anatomic factors such as the position of the stone in the cystic duct, the degree of stone impaction, the diameter of the cystic duct, and the number of valves in the duct.17
Stones in the cystic duct that cannot be extracted with ERCP may benefit from fragmentation techniques in situ via holmium laser followed by endoscopic extraction.
Repeat cholecystectomy is generally advised for any residual gallbladder, and it can be done laparoscopically.
Four months after undergoing laparoscopic cholecystectomy for symptomatic gallstones, an otherwise healthy 26-year-old woman begins to have episodes of epigastric and back pain similar to what she experienced before the surgery. The surgery was without complications, and her classic biliary colic disappeared afterward. Histologic evaluation of the surgical specimen revealed chronic cholecystitis with multiple small, mixed gallstones.
Now she describes a burning pain in her epigastrium and mid to upper back, starting about 30 minutes after a meal and lasting up to 4 hours. Sometimes it awakens her at night. She avoids eating for fear of inducing the pain. She has occasional chills but no fever, nausea, vomiting, jaundice, or changes in urine or stool color.
Three years ago she was diagnosed with a gastric ulcer induced by taking a nonsteroidal anti-inflammatory drug (NSAID). The ulcer was treated with a proton pump inhibitor for 1 month. She says the ulcer pain was dull and aching, different from her current pain.
Upper endoscopy 4 months ago (ie, before her laparoscopic cholecystectomy) showed no evidence of esophagitis or peptic ulcer disease.
Apart from her gallbladder operation, she has had no other surgery. According to the surgeon’s notes, intraoperative cholangiography was not performed, and no macroscopic changes of acute cholecystitis or difficult biliary anatomy were noted.
The patient does not smoke, does not drink alcohol, is not currently taking any medications, including NSAIDs or over-the-counter medications, and has not taken any recently. Her mother also had symptomatic gallstones requiring cholecystectomy.
On physical examination, only fever
On examination, her temperature is 101.2°F (38.4°C), blood pressure 117/80 mm Hg, heart rate 82 beats per minute, and blood oxygen saturation 99% on room air. Her weight is 138 lb (62.6 kg), height 5 feet 6 inches (168 cm).
There is no jaundice or pallor. Her heart and lung examinations are normal.
Her abdomen is soft and mildly tender to palpation of the epigastrium, with no distention or hepatosplenomegaly and no rebound tenderness or guarding. The scars from her laparoscopic surgery have healed well. Her bowel sounds are normal.
No costovertebral angle or spinal tenderness can be elicited.
Her laboratory values are shown in Table 1.
POSTCHOLECYSTECTOMY SYNDROME
1.After cholecystectomy, preoperative symptoms recur in what percentage of patients?
10% to 40%
50%
60%
80%
Postcholecystectomy syndrome—the recurrence of symptoms similar to those before the procedure—occurs in 10% to 40% of patients. The time to the onset of symptoms can range from 2 days to up to 25 years.1–4 Women may be at higher risk, with symptoms recurring in 43% vs 28% in men.5
Postcholecystectomy syndrome can have a biliary or a nonbiliary cause. Biliary causes include strictures, retained calculi, dropped calculi, tumors, sphincter of Oddi dysfunction, and calculi in the cystic duct remnant. Nonbiliary causes include functional and organic disorders such as peptic ulcer disease, gastroesophageal reflux, pancreatic disease, hepatocellular disorders, coronary artery disease, irritable bowel syndrome, and intercostal neuritis.
WHAT IS THE NEXT STEP?
2.Which is the most appropriate next step in the workup of this patient?
Ultrasonography of the right upper quadrant
Magnetic resonance cholangiopancreatography (MRCP)
Review the operative record and consult with the surgeon
Although the patient is presenting with pain and fever, two features of the classic Charcot triad (pain, fever, jaundice) seen in cholangitis (infection of a bile duct), and although cholangitis almost confirms the diagnosis of common bile duct stones in a patient with gallstones (before or after cholecystectomy), other diagnoses to consider are bile duct injury, bile leak, and biloma.
Biloma can be detected with ultrasonography. Bile duct injuries are identified intraoperatively in up to 25% of patients. For those with an unrecognized injury, the clinical presentation is variable and depends on the type of injury. If a bile leak is present, patients present early, at a median of 3 days postoperatively. However, our patient presented with symptoms 4 months after her surgery. Patients with bile duct strictures without bile leak have a longer symptom-free interval and usually present with signs of biliary obstruction. Ultrasonography can then detect biliary dilatation.6
It would be very helpful to review the operative record and to talk to the surgeon to confirm that intraoperative cholangiography had not been done and to determine the level of difficulty of the surgery. (Intraoperative cholangiography involves the introduction of contrast dye into the biliary system by cannulation of the cystic duct or by direct injection into the common bile duct. An intraoperative cholangiogram is considered normal if the entire intrahepatic and extrahepatic biliary tree is seen to be filled with contrast.) A normal cholangiogram has a negative predictive value of 99.8% for the detection of ductal stones. Thus, a normal intraoperative cholangiogram can prevent unnecessary postoperative ECRP, since it almost always indicates a clean bile duct.7
Ultrasonography of the right upper quadrant has a low sensitivity (< 50%) for detecting common bile duct stones. However, it is highly operator-dependent, and it may be twice as sensitive if done by expert radiologists than by less experienced ones. Its limitations include poor visualization of the distal portion of the duct and low sensitivity in patients in whom the common bile duct is minimally dilated and also in patients with small stones. In most studies, however, it had a very high specificity—ie, greater than 95%.8
MRCP has a sensitivity of 82.6% and a specificity of 97.5% in detecting stones in the common bile duct.9 Therefore, normal results on abdominal ultrasonography and MRCP do not completely rule out stones.
Although this patient has a high pretest probability of having common bile duct stones, ERCP should be done only after a thorough review of the previous operative procedure.
Observation and reassurance are not appropriate in a patient with cholangitis, such as this patient, because waiting increases the risk of septicemia.
The patient undergoes ERCP with stone removal
Review of the operative report and discussion with the surgeon confirm that the laparoscopic procedure was uneventful and that intraoperative cholangiography was not done.
Therefore, the patient undergoes ERCP. The major papilla is normal. Cholangiography reveals nondilated common bile and intrahepatic ducts, with faint filling defects in the mid to distal common bile duct. Endoscopic sphincterotomy is performed, and three small stones are extracted from the common bile duct. Repeat balloon-occlusion cholangiography is normal.
The patient tolerates the procedure well and resumes a normal diet and normal activities.
Her pain persists, prompting an emergency room visit
Five days after her ERCP procedure, however, the same burning epigastric pain returns. As before, the pain occurs after eating and does not occur with fasting. At this time, she has no fever or chills.
The patient continues to have recurrent episodes of pain, on one occasion so severe she visits the emergency department. During this visit she reports she has no symptoms other than pain, and the examination is normal. Laboratory tests (Table 2)show that her liver function measures have normalized.
WHAT IS CAUSING HER PAIN?
3. Which is the most likely cause of her persistent pain?
Acute pancreatitis after ERCP
Peptic ulcer disease
Sphincter of Oddi dysfunction
Biliary stones
The most likely cause is persistent biliary stones. The common bile duct was recently explored and stones were removed, but she may still have stones in the intrahepatic ducts or in the cystic duct remnant, both of which were unopacified during the ERCP procedure, indicating that either the test was incomplete or a stone is obstructing the passage of contrast. Her persistent symptoms warrant repeating her liver function tests.
Acute pancreatitis is the most common and feared complication of ERCP, and it should be suspected in any patient who develops abdominal pain within 6 hours of the procedure. It is much less likely to develop after 12 hours, however. Risk factors for post-ERCP pancreatitis include patient factors (young age, female sex, history of recurrent pancreatitis), procedural factors (difficult cannulation, minor papilla sphincterotomy), and, less likely, operator-related factors.10–13 In general, the more likely a patient is to have an abnormal and irregular common bile duct or pancreatic duct, the lower the risk of post-ERCP pancreatitis. The importance of operator-dependent factors is not yet clear.10–13
Despite the postprandial pattern of our patient’s pain and her history of gastric ulcer, peptic ulcer disease is unlikely in view of a normal esophagogastroduodenoscopic examination done 4 months earlier, and since she has no recent exposure to NSAIDs.
Sphincter of Oddi dysfunction may explain her symptoms, but she recently underwent endoscopic sphincterotomy, which is regarded as the most definitive treatment.14
WHAT SHOULD BE DONE NEXT?
4. What would be the best next step in her management?
Repeat ERCP
MRCP
Endoscopic ultrasonography
Observation and reassurance
MRCP is the most appropriate next step, given her recurrent symptoms. Repeat ERCP is not appropriate, since there is no evidence of cholangitis, and since her liver function tests had completely normalized.
A recent systematic review of endoscopic ultrasonography and MRCP for diagnosing choledocholithiasis found both tests to be highly accurate, with no statistically significant differences in sensitivity or specificity between the two.15 However, MRCP has the advantage of being noninvasive and of being able to show intrahepatic stones.
Park et al,16 in a prospective study of 66 patients with primary intrahepatic stones, concluded that MRCP findings were comparable to those of percutaneous transhepatic cholangioscopy, the reference standard for locating intrahepatic stones. The sensitivity, specificity, and accuracy of MRCP for detecting and locating intrahepatic stones were high (97%, 99%, and 98%, respectively).16 However, after sphincterotomy, pneumobilia may create an appearance that can be mistaken for intraductal stones.
Figure 1. Magnetic resonance cholangiopancreatography shows a normal biliary tree (arrow) and pancreatic duct. The cystic duct cannot be seen.Merely reassuring the patient is not appropriate at this point, given her level of pain.
She undergoes MRCP
Figure 2. Ultrasonography of the right upper quadrant shows a nondilated common bile duct 4 mm in diameter (arrow). No stones are visible.MRCP shows a normal biliary tree without stones (Figure 1). Similarly, ultrasonography of the right upper quadrant shows no stones and a nondilated common bile duct (Figure 2).
The patient continues to have pain, and she has lost 5 pounds because she is still avoiding eating. At this point, she is beginning to wonder if her symptoms are psychogenic, since all the test results have been normal.
ERCP, MRCP, ULTRASONOGRAPHY?
5.What would be the best next step?
Reassurance
Referral to a psychiatrist
Referral to a pain management clinic
Endoscopic ultrasonography
Repeat ERCP
Endoscopic ultrasonography is needed to look for cystic duct stones. Although several tests have shown normal results, the patient’s pain continues as in the previous episodes, making stone disease the most likely cause.
Although no stones were seen on MRCP and ultrasonography, a detailed evaluation for stones in a cystic duct or retained gallbladder remnant was not done satisfactorily.
Reassurance and referral to a psychiatrist or pain management clinic are not appropriate, since an organic cause of her pain has not been completely ruled out.
Figure 3. Endoscopic ultrasonography from the duodenal bulb shows a 7-mm stone (arrow) in the cystic duct remnant or gallbladder remnant.ERCP should not be used as a diagnostic test in a situation such as this.
Findings on endoscopic ultrasonography
Endoscopic ultrasonography is performed and reveals a large (7-mm) stone in the area of the cystic duct remnant or gallbladder remnant (Figure 3). The common bile duct is normal.
CAUSES OF RETAINED GALLBLADDER AND CYSTIC DUCT REMNANT
6.What may have predisposed this patient to a retained gallbladder or cystic duct remnant after her surgery?
Laparoscopic cholecystectomy
Not doing intraoperative cholangiography
Cholecystectomy for acute cholecystitis
All of the above
All of the above may have contributed.
Postcholecystectomy syndrome can pose a diagnostic and therapeutic challenge, as in our patient. Although it has been reported since the advent of the operation, it is more common after laparoscopic cholecystectomy than after open surgery. One possible cause is stones in a cystic duct remnant, ie, a stub longer than 1 cm.
During open cholecystectomy, the cystic duct is ligated and cut as close to the common bile duct as possible, leaving only a small remnant. In laparoscopic cholecystectomy, it is divided closer to the gallbladder to avoid iatrogenic injury to the common bile duct, leaving a longer remnant. A long cystic duct remnant can be prevented by accurately locating the junction of the gallbladder and the cystic duct during cholecystectomy and by routinely doing intraoperative cholangiography. The presence of stones in a cystic duct or retained gallbladder remnant is a rare cause of postcholecystectomy syndrome, and suspicion is required to make the diagnosis.17–19
We should note that stones may also lurk in the short cystic duct remnant left after open cholecystectomy. In fact, the first case of cystic duct remnant, the so-called reformed gallbladder containing stones, was described in 1912 by Flörcken.20
Intraoperative cholangiography was introduced in 1931 by Mirizzi,21 who recommended its routine use. Since the advent of laparoscopic cholecystectomy in 1988, the routine use of intraoperative cholangiography has been debated. Advocates point to its ability to detect unsuspected calculi and to delineate the biliary anatomy, thus reducing the risk of biliary duct injury.7,22–25 Those who argue against its routine use emphasize the low reported rates of unsuspected stones in the common bile duct (2% to 3%), a longer operative time, the additional cost, and false-positive results that may lead to unnecessary common bile duct exploration. Another argument against its routine use is that most small ductal stones pass spontaneously without significant sequelae.26–28 Surgeons who use intraoperative cholangiography only selectively use it in patients with unclear biliary anatomy and preoperative biochemical or radiologic evidence of choledocholithiasis.
Figure 4. Endoscopic retrograde cholangiopancreatography shows an oval filling defect in the cystic duct remnant at its insertion into the common bile duct (arrow).Another potential explanation for the retained gallbladder remnant is that the cholecystectomy was done while the patient had acute cholecystitis, in which inflammation may obscure anatomic landmarks. Hence, cholangiography during laparoscopic cholecystectomy has been widely recognized as a means of delineating the biliary anatomy.
Case continued: She undergoes repeat ERCP
Figure 5. Endoscopic retrograde cholangiopancreatography reveals a long duct remnant (red arrow) and a small gallbladder remnant (black arrow). The stone has already been extracted.The patient undergoes ERCP again (Figure 4 and Figure 5). Cholangiography shows a normal common bile duct with low insertion of the cystic duct and an oval filling defect in the cystic duct just proximal to its insertion into the common bile duct. Cystic duct opacification reveals a long cystic duct remnant and a small gallbladder remnant. The stone in the cystic duct is successfully removed.
IF STONES ARE DIFFICULT TO EXTRACT
7. If the cystic duct stone were not amenable to endoscopic extraction, what would be the best alternative?
Extracorporeal shock-wave lithotripsy (ESWL)
Endoscopic biliary laser lithotripsy
Repeat laparoscopic cholecystectomy
All of the above
All of the above are alternatives.
A symptomatic stone in a cystic duct remnant is uncommon and is mentioned in the literature only in case series and case reports.
ESWL is effective for treating bile duct calculi.29 In a cohort of 239 patients with bile duct stones treated by ESWL, Benninger et al30 concluded that endoscopy plus ESWL was a definitive treatment for all patients except one, who subsequently underwent cholecystectomy. Once fragmented, the stones are extracted endoscopically.
Another fragmentation technique that can be offered to patients with stones in the cystic duct that are difficult to extract is contact fragmentation with a holmium laser placed in a transpapillary position under visual guidance.17
Repeat cholecystectomy with removal of stones in the cystic duct remnant (and removal of retained gallbladder remnants and reduction of the cystic duct remnant) has good postoperative results.17,18,31,32
After incomplete cholecystectomy, the cystic duct remnant and the Calot (cystohepatic) triangle are surrounded by inflamed scar tissue, and this was thought to make laparoscopic reoperation difficult.33 However, with advances in surgical technique and increasing experience of surgeons, repeat cholecystectomy can be done laparoscopically. It has now been suggested that laparoscopic exploration to remove the gallbladder remnants is safe and feasible in such patients.34,35
Discharge and follow-up
The patient is discharged home after the procedure. She is still free of symptoms 31 months later.
LESSONS LEARNED
Remnant cystic duct stones are uncommon
The estimated incidence of a retained calculus within the cystic duct remnant after cholecystectomy is less than 2.5%.2,36 In a series of 322 patients who underwent repeat surgery because of postcholecystectomy syndrome, Rogy et al36 found only 8 who had a stone in the cystic duct or gallbladder remnant, and in a series of 371 patients, Zhou el al2 found 4 who had a stone in the cystic duct remnant.
Stones in the cystic duct remnant are difficult to diagnose
Diagnosing stones in surgical remnants of the cystic duct or gallbladder can be difficult. The sensitivity of abdominal ultrasonography in detecting cystic duct stones is low—only 27% in one study, with a specificity of 100% and an accuracy of 75%.37 Ultrasonography may occasionally suggest cystic duct stones by showing an acoustic shadow in the anatomic region of the cystic duct. However, the results should be interpreted with caution.
Determining the accuracy of ERCP and MRCP in detecting cystic duct remnant stones is also difficult, as few cases have been reported and data may be conflicting. In a review of seven patients confirmed to have retained stones in a surgical remnant, Walsh et al17 found that ERCP correctly diagnosed the retained stone in only four out of six patients; MRCP was done in one patient, and it was read as normal.
In three cases of stones in a postsurgical gallbladder remnant, Hassan and Vilmann38 reported that ERCP and MRCP failed to identify the gallbladder remnant in two out of three cases, likely because the remaining structures are small. The diagnosis was finally made by endoscopic ultrasonography, which the authors concluded was a valuable method to visualize a small gallbladder remnant with stones.
Greater suspicion is needed in patients with typical biliary colic after cholecystectomy
Retained gallbladder remnant is described in the literature as a latent complication. The main problem is not the remnant itself but the chance that it harbors retained stones, which can lead to dilatation and inflammation of the remnant.
The patient can develop symptoms of acute cholecystitis or even acute cholangitis if the stone migrates to the common bile duct. Symptoms can develop as early as 2 weeks or as late as 25 years after laparoscopic cholecystectomy.
Endoscopic ultrasonography may be the best way to look for these remnant stones and to evaluate the bile duct and pancreas. Therefore, it should be part of the diagnostic algorithm in the evaluation of postcholecystectomy pain.
Mixed results with ERCP for extracting cystic duct stones
In case reports of cystic duct calculi after cholecystectomy, ERCP by itself has had mixed results. This traditional means of removing stones may succeed, as in our case. However, the success rate depends largely on anatomic factors such as the position of the stone in the cystic duct, the degree of stone impaction, the diameter of the cystic duct, and the number of valves in the duct.17
Stones in the cystic duct that cannot be extracted with ERCP may benefit from fragmentation techniques in situ via holmium laser followed by endoscopic extraction.
Repeat cholecystectomy is generally advised for any residual gallbladder, and it can be done laparoscopically.
References
Lehman GA, Sherman S. Sphincter of Oddi dysfunction (postcholecystectomy syndrome). In:Yamada T, editor. Textbook of Gastroenterology. 2nd ed. Philadelphia: Lippincott; 1995:2251–2262.
Zhou PH, Liu FL, Yao LQ, Qin XY. Endoscopic diagnosis and treatment of post-cholecystectomy syndrome. Hepatobiliary Pancreat Dis Int2003; 2:117–120.
Mergener K, Clavien PA, Branch MS, Baillie J. A stone in a grossly dilated cystic duct stump: a rare cause of postcholecystectomy pain. Am J Gastroenterol1999; 94:229–231.
Goenka MK, Kochhar R, Nagi B, Bhasin DK, Chowdhury A, Singh K. Endoscopic retrograde cholangiopancreatography in postcholecystectomy syndrome. J Assoc Physicians India1996; 44:119–122.
Bodvall B, Overgaard B. Cystic duct remnant after cholecystectomy: incidence studied by cholegraphy in 500 cases, and significance in 103 reoperations. Ann Surg1966; 163:382–390.
Bergman JJ, van den Brink GR, Rauws EA, et al. Treatment of bile duct lesions after laparoscopic cholecystectomy. Gut1996; 38:141–147.
Nickkholgh A, Soltaniyekta S, Kalbasi H. Routine versus selective intraoperative cholangiography during laparoscopic cholecystectomy: a survey of 2,130 patients undergoing laparoscopic cholecystectomy. Surg Endosc2006; 20:868–874.
Gandolfi L, Torresan F, Solmi L, Puccetti A. The role of ultrasound in biliary and pancreatic diseases. Eur J Ultrasound2003; 16:141–159.
Al Samaraee A, Khan U, Almashta Z, Yiannakou Y. Preoperative diagnosis of choledocholithiasis: the role of MRCP. Br J Hosp Med (Lond)2009; 70:339–343.
Freeman ML, DiSario JA, Nelson DB, et al. Risk factors for post-ERCP pancreatitis: a prospective, multicenter study. Gastrointest Endosc2001; 54:425–434.
Cheng CL, Sherman S, Watkins JL, et al. Risk factors for post-ERCP pancreatitis: a prospective multicenter study. Am J Gastroenterol2006; 101:139–147.
Mehta SN, Pavone E, Barkun JS, Bouchard S, Barkun AN. Predictors of post-ERCP complications in patients with suspected choledocholithiasis. Endoscopy1998; 30:457–463.
Badalov N, Tenner S, Baillie J. The prevention, recognition and treatment of post-ERCP pancreatitis. JOP2009; 10:88–97.
Geenen JE, Hogan WJ, Dodds WJ, Toouli J, Venu RP. The efficacy of endoscopic sphincterotomy after cholecystectomy in patients with sphincter-of-Oddi dysfunction. N Engl J Med1989; 320:82–87.
Verma D, Kapadia A, Eisen GM, Adler DG. EUS vs MRCP for detection of choledocholithiasis. Gastrointest Endosc2006; 64:248–254.
Park DH, Kim MH, Lee SS, et al. Accuracy of magnetic resonance cholangiopancreatography for locating hepatolithiasis and detecting accompanying biliary strictures. Endoscopy2004; 36:987–992.
Walsh RM, Ponsky JL, Dumot J. Retained gallbladder/cystic duct remnant calculi as a cause of postcholecystectomy pain. Surg Endosc2002; 16:981–984.
Tantia O, Jain M, Khanna S, Sen B. Post cholecystectomy syndrome: role of cystic duct stump and re-intervention by laparoscopic surgery. J Minim Access Surg2008; 4:71–75.
Palanivelu C, Rangarajan M, Jategaonkar PA, Madankumar MV, Anand NV. Laparoscopic management of remnant cystic duct calculi: a retrospective study. Ann R Coll Surg Engl2009; 91:25–29.
Flörcken H. Gallenblasenregeneration mit Steinrecidiv nach Cholecystectomie. Deutsch Z Chir1912; 113:604.
Mirizzi PL. La colangiografía durante las operaciones de las vias biliares. Bol Soc Cirug Buenos Aires1932; 16:1113.
Soper NJ, Brunt LM. The case for routine operative cholangiography during laparoscopic cholecystectomy. Surg Clin North Am1994; 74:953–959.
Cuschieri A, Shimi S, Banting S, Nathanson LK, Pietrabissa A. Intraoperative cholangiography during laparoscopic cholecystectomy. Routine vs selective policy. Surg Endosc1994; 8:302–305.
Woods MS, Traverso LW, Kozarek RA, et al. Biliary tract complications of laparoscopic cholecystectomy are detected more frequently with routine intraoperative cholangiography. Surg Endosc1995; 9:1076–1080.
Vezakis A, Davides D, Ammori BJ, Martin IG, Larvin M, McMahon MJ. Intraoperative cholangiography during laparoscopic cholecystectomy. Surg Endosc2000; 14:1118–1122.
Ladocsi LT, Benitez LD, Filippone DR, Nance FC. Intraoperative cholangiography in laparoscopic cholecystectomy: a review of 734 consecutive cases. Am Surg1997; 63:150–156.
Clair DG, Brooks DC. Laparoscopic cholangiography. The case for a selective approach. Surg Clin North Am1994; 74:961–966.
Collins C, Maguire D, Ireland A, Fitzgerald E, O’Sullivan GC. A prospective study of common bile duct calculi in patients undergoing laparoscopic cholecystectomy: natural history of choledocholithiasis revisited. Ann Surg2004; 239:28–33.
Ponsky LE, Geisinger MA, Ponsky JL, Streem SB. Contemporary ‘urologic’ intervention in the pancreaticobiliary tree. Urology2001; 57:21–25.
Benninger J, Rabenstein T, Farnbacher M, Keppler J, Hahn EG, Schneider HT. Extracorporeal shockwave lithotripsy of gallstones in cystic duct remnants and Mirizzi syndrome. Gastrointest Endosc2004; 60:454–459.
Demetriades H, Pramateftakis MG, Kanellos I, Angelopoulos S, Mantzoros I, Betsis D. Retained gallbladder remnant after laparoscopic cholecystectomy. J Laparoendosc Adv Surg Tech A2008; 18:276–279.
Shaw C, O’Hanlon DM, Fenlon HM, McEntee GP. Cystic duct remnant and the ‘post-cholecystectomy syndrome. ’ Hepatogastroenterology2004; 51:36–38.
Rozsos I, Magyaródi Z, Orbán P. Cystic duct syndrome and minimally invasive surgery. [Hungarian] Orv Hetil1997; 138:2397–2401.
Chowbey PK, Bandyopadhyay SK, Sharma A, Khullar R, Soni V, Baijal M. Laparoscopic reintervention for residual gallstone disease. Surg Laparosc Endosc Percutan Tech2003; 13:31–35.
Clemente G, Giuliante F, Cadeddu F, Nuzzo G. Laparoscopic removal of gallbladder remnant and long cystic stump. Endoscopy2001; 33:814–815.
Rogy MA, Függer R, Herbst F, Schulz F. Reoperation after cholecystectomy. The role of the cystic duct stump. HPB Surg1991; 4:129–134.
Hassan H, Vilmann P. Insufficient cholecystectomy diagnosed by endoscopic ultrasonography. Endoscopy2004; 36:236–238.
References
Lehman GA, Sherman S. Sphincter of Oddi dysfunction (postcholecystectomy syndrome). In:Yamada T, editor. Textbook of Gastroenterology. 2nd ed. Philadelphia: Lippincott; 1995:2251–2262.
Zhou PH, Liu FL, Yao LQ, Qin XY. Endoscopic diagnosis and treatment of post-cholecystectomy syndrome. Hepatobiliary Pancreat Dis Int2003; 2:117–120.
Mergener K, Clavien PA, Branch MS, Baillie J. A stone in a grossly dilated cystic duct stump: a rare cause of postcholecystectomy pain. Am J Gastroenterol1999; 94:229–231.
Goenka MK, Kochhar R, Nagi B, Bhasin DK, Chowdhury A, Singh K. Endoscopic retrograde cholangiopancreatography in postcholecystectomy syndrome. J Assoc Physicians India1996; 44:119–122.
Bodvall B, Overgaard B. Cystic duct remnant after cholecystectomy: incidence studied by cholegraphy in 500 cases, and significance in 103 reoperations. Ann Surg1966; 163:382–390.
Bergman JJ, van den Brink GR, Rauws EA, et al. Treatment of bile duct lesions after laparoscopic cholecystectomy. Gut1996; 38:141–147.
Nickkholgh A, Soltaniyekta S, Kalbasi H. Routine versus selective intraoperative cholangiography during laparoscopic cholecystectomy: a survey of 2,130 patients undergoing laparoscopic cholecystectomy. Surg Endosc2006; 20:868–874.
Gandolfi L, Torresan F, Solmi L, Puccetti A. The role of ultrasound in biliary and pancreatic diseases. Eur J Ultrasound2003; 16:141–159.
Al Samaraee A, Khan U, Almashta Z, Yiannakou Y. Preoperative diagnosis of choledocholithiasis: the role of MRCP. Br J Hosp Med (Lond)2009; 70:339–343.
Freeman ML, DiSario JA, Nelson DB, et al. Risk factors for post-ERCP pancreatitis: a prospective, multicenter study. Gastrointest Endosc2001; 54:425–434.
Cheng CL, Sherman S, Watkins JL, et al. Risk factors for post-ERCP pancreatitis: a prospective multicenter study. Am J Gastroenterol2006; 101:139–147.
Mehta SN, Pavone E, Barkun JS, Bouchard S, Barkun AN. Predictors of post-ERCP complications in patients with suspected choledocholithiasis. Endoscopy1998; 30:457–463.
Badalov N, Tenner S, Baillie J. The prevention, recognition and treatment of post-ERCP pancreatitis. JOP2009; 10:88–97.
Geenen JE, Hogan WJ, Dodds WJ, Toouli J, Venu RP. The efficacy of endoscopic sphincterotomy after cholecystectomy in patients with sphincter-of-Oddi dysfunction. N Engl J Med1989; 320:82–87.
Verma D, Kapadia A, Eisen GM, Adler DG. EUS vs MRCP for detection of choledocholithiasis. Gastrointest Endosc2006; 64:248–254.
Park DH, Kim MH, Lee SS, et al. Accuracy of magnetic resonance cholangiopancreatography for locating hepatolithiasis and detecting accompanying biliary strictures. Endoscopy2004; 36:987–992.
Walsh RM, Ponsky JL, Dumot J. Retained gallbladder/cystic duct remnant calculi as a cause of postcholecystectomy pain. Surg Endosc2002; 16:981–984.
Tantia O, Jain M, Khanna S, Sen B. Post cholecystectomy syndrome: role of cystic duct stump and re-intervention by laparoscopic surgery. J Minim Access Surg2008; 4:71–75.
Palanivelu C, Rangarajan M, Jategaonkar PA, Madankumar MV, Anand NV. Laparoscopic management of remnant cystic duct calculi: a retrospective study. Ann R Coll Surg Engl2009; 91:25–29.
Flörcken H. Gallenblasenregeneration mit Steinrecidiv nach Cholecystectomie. Deutsch Z Chir1912; 113:604.
Mirizzi PL. La colangiografía durante las operaciones de las vias biliares. Bol Soc Cirug Buenos Aires1932; 16:1113.
Soper NJ, Brunt LM. The case for routine operative cholangiography during laparoscopic cholecystectomy. Surg Clin North Am1994; 74:953–959.
Cuschieri A, Shimi S, Banting S, Nathanson LK, Pietrabissa A. Intraoperative cholangiography during laparoscopic cholecystectomy. Routine vs selective policy. Surg Endosc1994; 8:302–305.
Woods MS, Traverso LW, Kozarek RA, et al. Biliary tract complications of laparoscopic cholecystectomy are detected more frequently with routine intraoperative cholangiography. Surg Endosc1995; 9:1076–1080.
Vezakis A, Davides D, Ammori BJ, Martin IG, Larvin M, McMahon MJ. Intraoperative cholangiography during laparoscopic cholecystectomy. Surg Endosc2000; 14:1118–1122.
Ladocsi LT, Benitez LD, Filippone DR, Nance FC. Intraoperative cholangiography in laparoscopic cholecystectomy: a review of 734 consecutive cases. Am Surg1997; 63:150–156.
Clair DG, Brooks DC. Laparoscopic cholangiography. The case for a selective approach. Surg Clin North Am1994; 74:961–966.
Collins C, Maguire D, Ireland A, Fitzgerald E, O’Sullivan GC. A prospective study of common bile duct calculi in patients undergoing laparoscopic cholecystectomy: natural history of choledocholithiasis revisited. Ann Surg2004; 239:28–33.
Ponsky LE, Geisinger MA, Ponsky JL, Streem SB. Contemporary ‘urologic’ intervention in the pancreaticobiliary tree. Urology2001; 57:21–25.
Benninger J, Rabenstein T, Farnbacher M, Keppler J, Hahn EG, Schneider HT. Extracorporeal shockwave lithotripsy of gallstones in cystic duct remnants and Mirizzi syndrome. Gastrointest Endosc2004; 60:454–459.
Demetriades H, Pramateftakis MG, Kanellos I, Angelopoulos S, Mantzoros I, Betsis D. Retained gallbladder remnant after laparoscopic cholecystectomy. J Laparoendosc Adv Surg Tech A2008; 18:276–279.
Shaw C, O’Hanlon DM, Fenlon HM, McEntee GP. Cystic duct remnant and the ‘post-cholecystectomy syndrome. ’ Hepatogastroenterology2004; 51:36–38.
Rozsos I, Magyaródi Z, Orbán P. Cystic duct syndrome and minimally invasive surgery. [Hungarian] Orv Hetil1997; 138:2397–2401.
Chowbey PK, Bandyopadhyay SK, Sharma A, Khullar R, Soni V, Baijal M. Laparoscopic reintervention for residual gallstone disease. Surg Laparosc Endosc Percutan Tech2003; 13:31–35.
Clemente G, Giuliante F, Cadeddu F, Nuzzo G. Laparoscopic removal of gallbladder remnant and long cystic stump. Endoscopy2001; 33:814–815.
Rogy MA, Függer R, Herbst F, Schulz F. Reoperation after cholecystectomy. The role of the cystic duct stump. HPB Surg1991; 4:129–134.
Figure 1.A 40-year-old woman presents to the clinic with multiple excoriated lesions over her chest, arms, abdomen, and upper back (Figure 1, Figure 2). The lesions have been present for many years; a few of them show signs of recent bleeding.
She denies any history of itching, insect bites, exposure to new medications or jewelry, allergies, recent change in medications, travel, or intravenous drug abuse.
Figure 2.A review of systems finds no liver, kidney, or heart disease. On examination, we find multiple scattered, weeping, crusted ulcerations, hyperpigmented macules and papules, and atrophic scars in different stages of healing on the upper chest, arms, abdomen, and upper back.
Q: Which is the most likely diagnosis?
Allergic contact dermatitis
Xerosis
Dermatotillomania
Folliculitis
Infestation (scabies)
A: Dermatotillomania, ie, pathologic skin picking, is the correct diagnosis. On further questioning, the patient reveals that the wounds have been self-inflicted over many years, starting in her adolescence. The wounds are located only in areas she can reach. She admits that social and emotional stressors had made the condition significantly worse and that lately she had lost control of her skin-picking. She denies nail-biting, trichotillomania, or obsessive-compulsive behavior.
As for the other possible diagnoses:
Allergic contact dermatitis occurs when contact with a particular substance elicits a hypersensitivity reaction. This reaction is of the delayed type (type IV). The affected individual can develop skin erythema and swelling with vesicles that are intensely pruritic at the contact site. The erythema may become evident hours after exposure, or not until weeks later, which can make the diagnosis difficult at times.
Our patient’s lesions were not pruritic, and she denied recent exposure to allergens.
Xerosis. Xerotic (dry) skin is usually rough, with fine scales and fissures. Xerosis can affect people of all ages and is often more intense during the winter. It affects mainly the arms, legs, and hands. Patients note pruritus, which can be treated with liberal use of emollients and tepid water baths.
Our patient’s lesions were scarred, hyperpigmented, and nonpruritic.
Folliculitis is a superficial infection of the hair follicle that presents as an erythematous pustule on the extremities, buttocks, or scalp. The pustule can be tender to palpation and can progress to an abscess that requires incision and drainage and intravenous antibiotics. A moist environment and poor hygiene are predisposing factors. Staphylococcus aureus is the culprit in most cases.
Our patient’s lesions were on the chest and upper back, where hair follicles were sparse or absent, and there was no erythema or tenderness.
Scabies is a skin infestation with Sarcoptes scabiei mites, which burrow in the skin and cause intense pruritus, especially at night. Scabies usually affects the sides and webs of the fingers and skin folds. Sexual contact is a common way of transmission; however, transmission can also occur by sharing beds and towels.
Patients with dermatotillomania lack intense pruritus, and skin-picking occurs during the day, while the patient is awake.
SELF-INFLICTED WOUNDS
Pathologic skin-picking, neurotic excoriation, excoriated acne, or dermatotillomania results from scratching, picking, gouging, or squeezing of one’s skin via teeth, fingernails, tweezers, or other objects.1–3 Lesions are usually found on skin that the patient can easily reach, such as the face, chest, upper and lower extremities, and upper back.4
The prevalence of pathologic skin-picking is estimated at 2% in dermatology patients.5 The overall prevalence of psychiatric disorders in all dermatology outpatients is estimated at 30% to 40%. Women outnumber men with this disorder.6
Dermatotillomania is thought to be on the spectrum of obsessive-compulsive disorder, in which patients exhibit impulses and compulsions.5 It starts in childhood or early adulthood, with an average lifetime duration of 21 years.7 It is usually associated with anxiety, depression, obsessive-compulsive traits, eating disorders, body dysmorphic disorders, or hypochondriasis. Psychosocial stress is the main trigger. Patients report feelings of tension and stress before picking and relief while picking; there is no suicidal ideation.8
Treatments are both pharmacologic and behavioral.9 Cognitive behavioral therapy and habit reversal therapy have each been successful when used alone.8 In addition, several case reports10 and double-blind studies11,12 have shown that treatment with a selective serotonin-reuptake inhibitor (SSRI) can reduce pathologic skin-picking.13,14 However, SSRIs have also been reported to induce or aggravate this behavior in patients with underlying mild skin-picking and a family history of skin-picking.15 Thus, it is pertinent to extract a detailed history from the patient before prescribing an SSRI.
We referred our patient for behavioral therapy and prescribed fluoxetine (Prozac) 20 mg daily. She showed improvement in symptoms in 4 weeks and has since stopped skin-picking completely.
References
Arnold LM. Phenomenology and therapeutic options for dermatotillomania. Expert Rev Neurother2002; 2:725–730.
Bohne A, Keuthen N, Wilhelm S. Pathologic hairpulling, skin picking, and nail biting. Ann Clin Psychiatry2005; 17:227–232.
Gattu S, Rashid RM, Khachemoune A. Self-induced skin lesions: a review of dermatitis artefacta. Cutis2009; 84:247–251.
Keuthen NJ, Deckersbach T, Wilhelm S, et al. Repetitive skin-picking in a student population and comparison with a sample of self-injurious skin-pickers. Psychosomatics2000; 41:210–215.
Arnold LM, Auchenbach MB, McElroy SL. Psychogenic excoriation. Clinical features, proposed diagnostic criteria, epidemiology and approaches to treatment. CNS Drugs2001; 15:351–359.
Wilhelm S, Keuthen NJ, Deckersbach T, et al. Self-injurious skin picking: clinical characteristics and comorbidity. J Clin Psychiatry1999; 60:454–459.
Gupta MA, Gupta AK, Haberman HF. Neurotic excoriations: a review and some new perspectives. Compr Psychiatry1986; 27:381–386.
Rosenbaum MS, Ayllon T. The behavioral treatment of neurodermatitis through habit-reversal. Behav Res Ther1981; 19:313–318.
Deckersbach T, Wilhelm S, Keuthen N. Self-injurious skin picking: clinical characteristics, assessment methods, and treatment modalities. Brief Treatment and Crisis Intervention2003; 3:249–260.
Sharma H. Psychogenic excoriation responding to fluoxetine: a case report. J Indian Med Assoc2008; 106:245,262.
Bloch MR, Elliott M, Thompson H, Koran LM. Fluoxetine in pathologic skin-picking: open-label and double-blind results. Psychosomatics2001; 42:314–319.
Simeon D, Stein DJ, Gross S, Islam N, Schmeidler J, Hollander E. A double-blind trial of fluoxetine in pathologic skin picking. J Clin Psychiatry1997; 58:341–347.
Gupta MA, Gupta AK. The use of antidepressant drugs in dermatology. J Eur Acad Dermatol Venereol2001; 15:512–518.
Keuthen NJ, Jameson M, Loh R, Deckersbach T, Wilhelm S, Dougherty DD. Open-label escitalopram treatment for pathological skin picking. Int Clin Psychopharmacol2007; 22:268–274.
Denys D, van Megen HJ, Westenberg HG. Emerging skin-picking behaviour after serotonin reuptake inhibitor-treatment in patients with obsessive-compulsive disorder: possible mechanisms and implications for clinical care. J Psychopharmacol2003; 17:127–129.
Figure 1.A 40-year-old woman presents to the clinic with multiple excoriated lesions over her chest, arms, abdomen, and upper back (Figure 1, Figure 2). The lesions have been present for many years; a few of them show signs of recent bleeding.
She denies any history of itching, insect bites, exposure to new medications or jewelry, allergies, recent change in medications, travel, or intravenous drug abuse.
Figure 2.A review of systems finds no liver, kidney, or heart disease. On examination, we find multiple scattered, weeping, crusted ulcerations, hyperpigmented macules and papules, and atrophic scars in different stages of healing on the upper chest, arms, abdomen, and upper back.
Q: Which is the most likely diagnosis?
Allergic contact dermatitis
Xerosis
Dermatotillomania
Folliculitis
Infestation (scabies)
A: Dermatotillomania, ie, pathologic skin picking, is the correct diagnosis. On further questioning, the patient reveals that the wounds have been self-inflicted over many years, starting in her adolescence. The wounds are located only in areas she can reach. She admits that social and emotional stressors had made the condition significantly worse and that lately she had lost control of her skin-picking. She denies nail-biting, trichotillomania, or obsessive-compulsive behavior.
As for the other possible diagnoses:
Allergic contact dermatitis occurs when contact with a particular substance elicits a hypersensitivity reaction. This reaction is of the delayed type (type IV). The affected individual can develop skin erythema and swelling with vesicles that are intensely pruritic at the contact site. The erythema may become evident hours after exposure, or not until weeks later, which can make the diagnosis difficult at times.
Our patient’s lesions were not pruritic, and she denied recent exposure to allergens.
Xerosis. Xerotic (dry) skin is usually rough, with fine scales and fissures. Xerosis can affect people of all ages and is often more intense during the winter. It affects mainly the arms, legs, and hands. Patients note pruritus, which can be treated with liberal use of emollients and tepid water baths.
Our patient’s lesions were scarred, hyperpigmented, and nonpruritic.
Folliculitis is a superficial infection of the hair follicle that presents as an erythematous pustule on the extremities, buttocks, or scalp. The pustule can be tender to palpation and can progress to an abscess that requires incision and drainage and intravenous antibiotics. A moist environment and poor hygiene are predisposing factors. Staphylococcus aureus is the culprit in most cases.
Our patient’s lesions were on the chest and upper back, where hair follicles were sparse or absent, and there was no erythema or tenderness.
Scabies is a skin infestation with Sarcoptes scabiei mites, which burrow in the skin and cause intense pruritus, especially at night. Scabies usually affects the sides and webs of the fingers and skin folds. Sexual contact is a common way of transmission; however, transmission can also occur by sharing beds and towels.
Patients with dermatotillomania lack intense pruritus, and skin-picking occurs during the day, while the patient is awake.
SELF-INFLICTED WOUNDS
Pathologic skin-picking, neurotic excoriation, excoriated acne, or dermatotillomania results from scratching, picking, gouging, or squeezing of one’s skin via teeth, fingernails, tweezers, or other objects.1–3 Lesions are usually found on skin that the patient can easily reach, such as the face, chest, upper and lower extremities, and upper back.4
The prevalence of pathologic skin-picking is estimated at 2% in dermatology patients.5 The overall prevalence of psychiatric disorders in all dermatology outpatients is estimated at 30% to 40%. Women outnumber men with this disorder.6
Dermatotillomania is thought to be on the spectrum of obsessive-compulsive disorder, in which patients exhibit impulses and compulsions.5 It starts in childhood or early adulthood, with an average lifetime duration of 21 years.7 It is usually associated with anxiety, depression, obsessive-compulsive traits, eating disorders, body dysmorphic disorders, or hypochondriasis. Psychosocial stress is the main trigger. Patients report feelings of tension and stress before picking and relief while picking; there is no suicidal ideation.8
Treatments are both pharmacologic and behavioral.9 Cognitive behavioral therapy and habit reversal therapy have each been successful when used alone.8 In addition, several case reports10 and double-blind studies11,12 have shown that treatment with a selective serotonin-reuptake inhibitor (SSRI) can reduce pathologic skin-picking.13,14 However, SSRIs have also been reported to induce or aggravate this behavior in patients with underlying mild skin-picking and a family history of skin-picking.15 Thus, it is pertinent to extract a detailed history from the patient before prescribing an SSRI.
We referred our patient for behavioral therapy and prescribed fluoxetine (Prozac) 20 mg daily. She showed improvement in symptoms in 4 weeks and has since stopped skin-picking completely.
Figure 1.A 40-year-old woman presents to the clinic with multiple excoriated lesions over her chest, arms, abdomen, and upper back (Figure 1, Figure 2). The lesions have been present for many years; a few of them show signs of recent bleeding.
She denies any history of itching, insect bites, exposure to new medications or jewelry, allergies, recent change in medications, travel, or intravenous drug abuse.
Figure 2.A review of systems finds no liver, kidney, or heart disease. On examination, we find multiple scattered, weeping, crusted ulcerations, hyperpigmented macules and papules, and atrophic scars in different stages of healing on the upper chest, arms, abdomen, and upper back.
Q: Which is the most likely diagnosis?
Allergic contact dermatitis
Xerosis
Dermatotillomania
Folliculitis
Infestation (scabies)
A: Dermatotillomania, ie, pathologic skin picking, is the correct diagnosis. On further questioning, the patient reveals that the wounds have been self-inflicted over many years, starting in her adolescence. The wounds are located only in areas she can reach. She admits that social and emotional stressors had made the condition significantly worse and that lately she had lost control of her skin-picking. She denies nail-biting, trichotillomania, or obsessive-compulsive behavior.
As for the other possible diagnoses:
Allergic contact dermatitis occurs when contact with a particular substance elicits a hypersensitivity reaction. This reaction is of the delayed type (type IV). The affected individual can develop skin erythema and swelling with vesicles that are intensely pruritic at the contact site. The erythema may become evident hours after exposure, or not until weeks later, which can make the diagnosis difficult at times.
Our patient’s lesions were not pruritic, and she denied recent exposure to allergens.
Xerosis. Xerotic (dry) skin is usually rough, with fine scales and fissures. Xerosis can affect people of all ages and is often more intense during the winter. It affects mainly the arms, legs, and hands. Patients note pruritus, which can be treated with liberal use of emollients and tepid water baths.
Our patient’s lesions were scarred, hyperpigmented, and nonpruritic.
Folliculitis is a superficial infection of the hair follicle that presents as an erythematous pustule on the extremities, buttocks, or scalp. The pustule can be tender to palpation and can progress to an abscess that requires incision and drainage and intravenous antibiotics. A moist environment and poor hygiene are predisposing factors. Staphylococcus aureus is the culprit in most cases.
Our patient’s lesions were on the chest and upper back, where hair follicles were sparse or absent, and there was no erythema or tenderness.
Scabies is a skin infestation with Sarcoptes scabiei mites, which burrow in the skin and cause intense pruritus, especially at night. Scabies usually affects the sides and webs of the fingers and skin folds. Sexual contact is a common way of transmission; however, transmission can also occur by sharing beds and towels.
Patients with dermatotillomania lack intense pruritus, and skin-picking occurs during the day, while the patient is awake.
SELF-INFLICTED WOUNDS
Pathologic skin-picking, neurotic excoriation, excoriated acne, or dermatotillomania results from scratching, picking, gouging, or squeezing of one’s skin via teeth, fingernails, tweezers, or other objects.1–3 Lesions are usually found on skin that the patient can easily reach, such as the face, chest, upper and lower extremities, and upper back.4
The prevalence of pathologic skin-picking is estimated at 2% in dermatology patients.5 The overall prevalence of psychiatric disorders in all dermatology outpatients is estimated at 30% to 40%. Women outnumber men with this disorder.6
Dermatotillomania is thought to be on the spectrum of obsessive-compulsive disorder, in which patients exhibit impulses and compulsions.5 It starts in childhood or early adulthood, with an average lifetime duration of 21 years.7 It is usually associated with anxiety, depression, obsessive-compulsive traits, eating disorders, body dysmorphic disorders, or hypochondriasis. Psychosocial stress is the main trigger. Patients report feelings of tension and stress before picking and relief while picking; there is no suicidal ideation.8
Treatments are both pharmacologic and behavioral.9 Cognitive behavioral therapy and habit reversal therapy have each been successful when used alone.8 In addition, several case reports10 and double-blind studies11,12 have shown that treatment with a selective serotonin-reuptake inhibitor (SSRI) can reduce pathologic skin-picking.13,14 However, SSRIs have also been reported to induce or aggravate this behavior in patients with underlying mild skin-picking and a family history of skin-picking.15 Thus, it is pertinent to extract a detailed history from the patient before prescribing an SSRI.
We referred our patient for behavioral therapy and prescribed fluoxetine (Prozac) 20 mg daily. She showed improvement in symptoms in 4 weeks and has since stopped skin-picking completely.
References
Arnold LM. Phenomenology and therapeutic options for dermatotillomania. Expert Rev Neurother2002; 2:725–730.
Bohne A, Keuthen N, Wilhelm S. Pathologic hairpulling, skin picking, and nail biting. Ann Clin Psychiatry2005; 17:227–232.
Gattu S, Rashid RM, Khachemoune A. Self-induced skin lesions: a review of dermatitis artefacta. Cutis2009; 84:247–251.
Keuthen NJ, Deckersbach T, Wilhelm S, et al. Repetitive skin-picking in a student population and comparison with a sample of self-injurious skin-pickers. Psychosomatics2000; 41:210–215.
Arnold LM, Auchenbach MB, McElroy SL. Psychogenic excoriation. Clinical features, proposed diagnostic criteria, epidemiology and approaches to treatment. CNS Drugs2001; 15:351–359.
Wilhelm S, Keuthen NJ, Deckersbach T, et al. Self-injurious skin picking: clinical characteristics and comorbidity. J Clin Psychiatry1999; 60:454–459.
Gupta MA, Gupta AK, Haberman HF. Neurotic excoriations: a review and some new perspectives. Compr Psychiatry1986; 27:381–386.
Rosenbaum MS, Ayllon T. The behavioral treatment of neurodermatitis through habit-reversal. Behav Res Ther1981; 19:313–318.
Deckersbach T, Wilhelm S, Keuthen N. Self-injurious skin picking: clinical characteristics, assessment methods, and treatment modalities. Brief Treatment and Crisis Intervention2003; 3:249–260.
Sharma H. Psychogenic excoriation responding to fluoxetine: a case report. J Indian Med Assoc2008; 106:245,262.
Bloch MR, Elliott M, Thompson H, Koran LM. Fluoxetine in pathologic skin-picking: open-label and double-blind results. Psychosomatics2001; 42:314–319.
Simeon D, Stein DJ, Gross S, Islam N, Schmeidler J, Hollander E. A double-blind trial of fluoxetine in pathologic skin picking. J Clin Psychiatry1997; 58:341–347.
Gupta MA, Gupta AK. The use of antidepressant drugs in dermatology. J Eur Acad Dermatol Venereol2001; 15:512–518.
Keuthen NJ, Jameson M, Loh R, Deckersbach T, Wilhelm S, Dougherty DD. Open-label escitalopram treatment for pathological skin picking. Int Clin Psychopharmacol2007; 22:268–274.
Denys D, van Megen HJ, Westenberg HG. Emerging skin-picking behaviour after serotonin reuptake inhibitor-treatment in patients with obsessive-compulsive disorder: possible mechanisms and implications for clinical care. J Psychopharmacol2003; 17:127–129.
References
Arnold LM. Phenomenology and therapeutic options for dermatotillomania. Expert Rev Neurother2002; 2:725–730.
Bohne A, Keuthen N, Wilhelm S. Pathologic hairpulling, skin picking, and nail biting. Ann Clin Psychiatry2005; 17:227–232.
Gattu S, Rashid RM, Khachemoune A. Self-induced skin lesions: a review of dermatitis artefacta. Cutis2009; 84:247–251.
Keuthen NJ, Deckersbach T, Wilhelm S, et al. Repetitive skin-picking in a student population and comparison with a sample of self-injurious skin-pickers. Psychosomatics2000; 41:210–215.
Arnold LM, Auchenbach MB, McElroy SL. Psychogenic excoriation. Clinical features, proposed diagnostic criteria, epidemiology and approaches to treatment. CNS Drugs2001; 15:351–359.
Wilhelm S, Keuthen NJ, Deckersbach T, et al. Self-injurious skin picking: clinical characteristics and comorbidity. J Clin Psychiatry1999; 60:454–459.
Gupta MA, Gupta AK, Haberman HF. Neurotic excoriations: a review and some new perspectives. Compr Psychiatry1986; 27:381–386.
Rosenbaum MS, Ayllon T. The behavioral treatment of neurodermatitis through habit-reversal. Behav Res Ther1981; 19:313–318.
Deckersbach T, Wilhelm S, Keuthen N. Self-injurious skin picking: clinical characteristics, assessment methods, and treatment modalities. Brief Treatment and Crisis Intervention2003; 3:249–260.
Sharma H. Psychogenic excoriation responding to fluoxetine: a case report. J Indian Med Assoc2008; 106:245,262.
Bloch MR, Elliott M, Thompson H, Koran LM. Fluoxetine in pathologic skin-picking: open-label and double-blind results. Psychosomatics2001; 42:314–319.
Simeon D, Stein DJ, Gross S, Islam N, Schmeidler J, Hollander E. A double-blind trial of fluoxetine in pathologic skin picking. J Clin Psychiatry1997; 58:341–347.
Gupta MA, Gupta AK. The use of antidepressant drugs in dermatology. J Eur Acad Dermatol Venereol2001; 15:512–518.
Keuthen NJ, Jameson M, Loh R, Deckersbach T, Wilhelm S, Dougherty DD. Open-label escitalopram treatment for pathological skin picking. Int Clin Psychopharmacol2007; 22:268–274.
Denys D, van Megen HJ, Westenberg HG. Emerging skin-picking behaviour after serotonin reuptake inhibitor-treatment in patients with obsessive-compulsive disorder: possible mechanisms and implications for clinical care. J Psychopharmacol2003; 17:127–129.
A 73-year-old man presented to the emergency department with chest tightness and shortness of breath associated with diaphoresis. His symptoms began as he was taking the bowel preparation solution HalfLytely, 8 oz every 10 minutes as directed, in preparation for colonoscopy the next morning for asymptomatic hematochezia. He had consumed 1 L of the solution when he noticed the symptoms. He tried to continue, but stopped halfway through the second liter because of worsening shortness of breath.
He reported no nausea, vomiting, palpitations, or dizziness. The discomfort (chest tightness) was somewhat relieved with rest and by leaning forward.
His medical history included chronic congestive heart failure with an ejection fraction of 25%; coronary artery disease with multiple myocardial infarctions, coronary angioplasty of the left anterior descending artery, and placement of a cardiac defibrillator; gout; gastritis; glaucoma; hypertension; and hyperlipidemia. His was taking potassium chloride (microK), furosemide (Lasix), clopidogrel (Plavix), enalapril (Vasotec), carvedilol (Coreg), simvastatin (Zocor), folic acid, pantoprazole (Protonix), allopurinol (Zyloprim), colchicine (Colcrys), dorzolamidetimolol eye drops (Cosopt), atropine eyedrops (Atropine Care), and ofloxacin eyedrops (Ocuflox). He said he takes all medications as directed without missing any doses. He smoked 1 pack of cigarettes per day for 50 years (he quit several years ago), and he drinks two to three glasses of brandy weekly. He denied using illicit drugs.
On examination, he appeared alert and comfortable. His temperature was 97.3°F (36.3°C), blood pressure 154/71 mm Hg, heart rate 81 beats per minute, respiratory rate 24 breaths per minute, and oxygen saturation 93% while breathing 4 L of oxygen by nasal canula. His jugular venous pressure was approximately 10 cm. The cardiac examination revealed normal first and second heart sounds and regular rate and rhythm. A third heart sound was present. On lung examination, breath sounds were decreased in both bases. Both lower extremities had trace edema. The remainder of the examination was normal.
His laboratory values were as follows:
White blood cell count 5.9 × 109/L (reference range 4.5–11.0)
Electrocardiography showed an old anteroseptal myocardial infarction with ST-segment elevations of less than 1 mm in leads V1, V2, and V3. Chest radiography showed vascular congestion, blunting of the right costophrenic angle likely due to small effusions, no focal consolidation, and no pneumothorax.
WHAT EXACERBATED THIS PATIENT’S HEART FAILURE?
In this patient, causes of shortness of breath such as aspiration pneumonia and myocardial infarction should be considered. But the timing of the symptoms, the laboratory test results, and the chest radiographic findings point to an exacerbation of heart failure as a result of the bowel preparation solution. The physiology of what may have produced this exacerbation will be discussed later.
The ideal preparation for diagnostic and surgical procedures would do the following:
Reliably remove stool from the colon without affecting its appearance
Require only a short amount of time to administer
Cause little or no discomfort for the patient
Produce no significant shifts of fluids or electrolytes.1–4
POLYETHYLENE GLYCOL SOLUTIONS ARE THE MOST POPULAR PREPARATIONS
Polyethylene glycol solutions are by far the most commonly used method of bowel preparation. Polyethylene glycol is a nonabsorbable solution that, in theory, passes through the bowel without net absorption or secretion, thereby avoiding significant fluid and electrolyte shifts. Large volumes (4 L) are required to achieve a cathartic effect, and since many patients find this difficult to tolerate, instructing patients to divide the doses—eg, 3 L the night before and 1 L the morning of the procedure—increases compliance.
Low-volume polyethylene glycol preparations such as HalfLytely and MiraLax were developed to improve patient tolerance by reducing the amount of solution required, while still maintaining efficacy by adding bisacodyl or magnesium citrate. Studies have shown 2-L solutions to be as effective as 4-L solutions in terms of colon cleansing, and to be better tolerated.5–8
Is some water from polyethylene glycol solutions absorbed?
Several bowel preparation methods expand the intravascular volume, which can in turn exacerbate heart failure, since patients with left ventricular systolic dysfunction are unable to adjust to changes in volume.
High-volume gut lavage uses high volumes of saline solution or balanced electrolye solution, leading to significant fluid and electrolyte shifts. Because of these significant side effects, high-volume gut lavage is no longer recommended.9
Goldman and Reichelderfer10 evaluated 43 patients in whom polyethylene glycol was used for bowel preparation; they measured body weight, hematocrit, and serum electrolyte levels before and after the preparation to detect fluid and electrolyte shifts, but they observed no shifts.
Several authors proposed that this method was insensitive for detecting changes in plasma volume. Turnage et al11 used an isotope dilution technique involving iodine-125-labeled human serum albumin to measure plasma volume before and after whole-gut irrigation with a polyethylene glycol solution. Although there was no significant change in body weight or electrolytes, the mean plasma volume increased by 5.88 ± 2.4%, with an increase in plasma volume of 12.9 ± 2.6% in patients over age 70 years and 17.45 ± 5% in patients taking more than 20 mg of furosemide daily. Thus, patients physiologically unable to compensate for an acute change in intravascular fluid, such as those with left ventricular dysfunction, are the ones most likely to have an increase in plasma volume after bowel preparation.11,12 However, this phenomenon appears to be rare, as a literature search on Medline found only one other case report of a heart failure exacerbation with a polyethylene glycol preparation.12
SODIUM PHOSPHATE PREPARATIONS CAN CAUSE FLUID DEPLETION
Aqueous sodium phosphate preparations such as Fleet are low-volume hyperosmotic solutions containing monobasic and dibasic sodium phosphate. They clean the colon by osmotically drawing plasma water into the bowel lumen.
However, this osmotic shift can cause serious volume contraction, electrolyte abnormalities such as hyperphosphatemia, and even death.1 Serious problems are more likely to occur in elderly patients, but also in patients with bowel obstruction, poor gut motility, renal failure, liver failure, or congestive heart failure.
Studies have shown hemodynamic changes indicating hypovolemia during sodium phosphate ingestion.13,14 One reported that 28% of patients had decreases in systolic blood pressure of more than 10 mm Hg, and 12% had decreases in systolic blood pressure of more than 20 mm Hg.13,14
In a study by Barclay et al,15 the concomitant use of a carbohydrate-electrolyte rehydration solution with the sodium phosphate limited the amount of fluid contraction.15 A similar study by Tjandra and Tagkalidis16 using a carbohydrate-electrolyte replacement solution called E-lyte showed a reduction in the degree of intravascular volume contraction, hypokalemia, and need for intravenous rehydration, although there was no effect on hyperphosphatemia resulting from ingestion of sodium phosphate.16
Phosphate nephropathy has also been reported after use of these solutions in patients with renal insufficiency, dehydration, or hypercalcemia, as well as in patients taking angiotensin-converting enzyme inhibitors or angiotensin receptor blockers.9
Current recommendations of the American Society of Colon and Rectal Surgeons, the American Society of Gastrointestinal Endoscopy, and the Society of American Gastrointestinal and Endoscopic Surgeons state that aqueous sodium phosphate is the preferable form of sodium phosphate and is an equal alternative to polyethylene glycol solutions except in pediatric and elderly patients, patients with bowel obstruction and other structural intestinal disorders, and patients with poor gut motility, renal failure, liver failure, or congestive heart failure.9 In fact, because of the significant volume contraction and electrolyte abnormalities that can occur, these groups list sodium phosphate preparations as contraindicated in patients with congestive heart failure.9
POLYETHYLENE GLYCOL IS STILL SAFER FOR THOSE WITH HEART FAILURE
Polyethylene glycol preparations have been shown to cause an increase in plasma volume in patients with disease states that predispose them to fluid retention.11 It has been postulated that these adverse effects may occur less often with preparations that use a lower volume, such as the 2-L polyethylene glycol regimen (HalfLytely) combined with bisacodyl or the 2-L polyethylene glycol 3350 solution (MiraLax). However, our patient experienced a heart failure exacerbation while drinking a reduced-volume preparation.
Another concern with polyethylene glycol solutions, especially HalfLytely, is hyperkalemia. Although no clinical reports have shown this finding, the small amount of potassium in this solution could be worrisome in patients with heart failure who are taking potassium-sparing diuretics or angiotensin-converting enzyme inhibitors.9
Nonetheless, when one considers the risks of fluid shifts with sodium phosphate preparations (which are in any event contraindicated for patients with congestive heart failure), the safest preparation for patients with congestive heart failure would be either a low-volume polyethylene glycol preparation or a split-dose standard-volume polyethylene glycol preparation with careful monitoring during and after use. Clinicians should emphasize to these patients the importance of continuing their cardiac medications during bowel preparation. The patients should also be instructed to notify their doctor about a change in body weight, difficulty breathing, chest tightness, and lower-extremity edema as they are taking the bowel preparation solution.12
Patients with severe left ventricular dysfunction may require short-term intensive care monitoring to manage plasma volume during and after bowel preparation.
References
Nelson DB, Barkun AN, Block KP, et al. Technology status evaluation report. Colonoscopy preparations. May 2001. Gastrointest Endosc2001; 54:829–832.
Berry MA, DiPalma JA. Review article: orthograde gut lavage for colonoscopy. Aliment Pharmacol Ther1994; 8:391–395.
DiPalma JA, Brady CE. Colon cleansing for diagnostic and surgical procedures: polyethylene glycol-electrolyte lavage solution. Am J Gastroenterol1989; 84:1008–1016.
DiPalma JA, Wolff BG, Meagher A, Cleveland M. Comparison of reduced volume versus four liters sulfate-free electrolyte lavage solutions for colonoscopy colon cleansing. Am J Gastroenterol2003; 98:2187–2191.
Adams WJ, Meagher AP, Lubowski DZ, King DW. Bisacodyl reduces the volume of polyethylene glycol solution required for bowel preparation. Dis Colon Rectum1994; 37:229–233.
Sharma VK, Steinberg EN, Vasudeva R, Howden CW. Randomized, controlled study of pretreatment with magnesium citrate on the quality of colonoscopy preparation with polyethylene glycol electrolyte lavage solution. Gastrointest Endosc1997; 46:541–543.
Stratton S, Shelton P, Carleton V, Griglione GR. Feasibility of polyethylene glycol (PEG) 3350 (Miralax) for colon preparation prior to lower endoscopic examination in healthy adults; experience in a community clinic setting. Am J Gastroenterol2008; 103:2163–2164.
Wexner SD, Beck DE, Baron TH, et al; American Society of Colon and Rectal Surgeons. A consensus document on bowel preparation before colonoscopy: prepared by a task force from the American Society of Colon and Rectal Surgeons (ASCRS), the American Society for Gastrointestinal Endoscopy (ASGE), and the Society of American Gastrointestinal and Endoscopic Surgeons (SAGES). Gastrointest Endosc2006; 63:894–909.
Goldman J, Reichelderfer M. Evaluation of rapid colonoscopy preparation using a new gut lavage solution. Gastrointest Endosc1982; 28:9–11.
Turnage RH, Guice KS, Gannon P, Gross M. The effect of polyethylene glycol gavage on plasma volume. J Surg Res1994; 57:284–288.
Granberry MC, White LM, Gardner SF. Exacerbation of congestive heart failure after administration of polyethylene glycol-electrolyte lavage solution. Ann Pharmacother1995; 29:1232–1235.
Afridi SA, Barthel JS, King PD, Pineda JJ, Marshall JB. Prospective, randomized trial comparing a new sodium phosphate-bisacodyl regimen with conventional PEG-ES lavage for outpatient colonoscopy preparation. Gastrointest Endosc1995; 41:485–489.
Curran MP, Plosker GL. Oral sodium phosphate solution: a review of its use as a colorectal cleanser. Drugs2004; 64:1697–1714.
Barclay RL, Depew WT, Vanner SJ. Carbohydrateelectrolyte rehydration protects against intravascular volume contraction during colonic cleansing with orally administered sodium phosphate. Gastrointest Endosc2002; 56:633–638.
Tjandra JJ, Tagkalidis P. Carbohydrate-electrolyte (E-Lyte) solution enhances bowel preparation with oral fleet phospho-soda. Dis Colon Rectum2004; 47:1181–1186.
Kalindi Parikh, MD Department of Internal Medicine, Thomas Jefferson University, Philadelphia, PA
Howard Weitz, MD Director, Division of Cardiology, and Bernard L. Segal Professor of Clinical Cardiology, The Jefferson Heart Institute, Philadelphia, PA
Address: Kalindi Parikh, MD, Department of Internal Medicine, Thomas Jefferson University, 833 Chestnut Street, Suite 220, Philadelphia, PA 19107; e-mail kalindi.parikh@jeffersonhospital.org
Kalindi Parikh, MD Department of Internal Medicine, Thomas Jefferson University, Philadelphia, PA
Howard Weitz, MD Director, Division of Cardiology, and Bernard L. Segal Professor of Clinical Cardiology, The Jefferson Heart Institute, Philadelphia, PA
Address: Kalindi Parikh, MD, Department of Internal Medicine, Thomas Jefferson University, 833 Chestnut Street, Suite 220, Philadelphia, PA 19107; e-mail kalindi.parikh@jeffersonhospital.org
Author and Disclosure Information
Kalindi Parikh, MD Department of Internal Medicine, Thomas Jefferson University, Philadelphia, PA
Howard Weitz, MD Director, Division of Cardiology, and Bernard L. Segal Professor of Clinical Cardiology, The Jefferson Heart Institute, Philadelphia, PA
Address: Kalindi Parikh, MD, Department of Internal Medicine, Thomas Jefferson University, 833 Chestnut Street, Suite 220, Philadelphia, PA 19107; e-mail kalindi.parikh@jeffersonhospital.org
A 73-year-old man presented to the emergency department with chest tightness and shortness of breath associated with diaphoresis. His symptoms began as he was taking the bowel preparation solution HalfLytely, 8 oz every 10 minutes as directed, in preparation for colonoscopy the next morning for asymptomatic hematochezia. He had consumed 1 L of the solution when he noticed the symptoms. He tried to continue, but stopped halfway through the second liter because of worsening shortness of breath.
He reported no nausea, vomiting, palpitations, or dizziness. The discomfort (chest tightness) was somewhat relieved with rest and by leaning forward.
His medical history included chronic congestive heart failure with an ejection fraction of 25%; coronary artery disease with multiple myocardial infarctions, coronary angioplasty of the left anterior descending artery, and placement of a cardiac defibrillator; gout; gastritis; glaucoma; hypertension; and hyperlipidemia. His was taking potassium chloride (microK), furosemide (Lasix), clopidogrel (Plavix), enalapril (Vasotec), carvedilol (Coreg), simvastatin (Zocor), folic acid, pantoprazole (Protonix), allopurinol (Zyloprim), colchicine (Colcrys), dorzolamidetimolol eye drops (Cosopt), atropine eyedrops (Atropine Care), and ofloxacin eyedrops (Ocuflox). He said he takes all medications as directed without missing any doses. He smoked 1 pack of cigarettes per day for 50 years (he quit several years ago), and he drinks two to three glasses of brandy weekly. He denied using illicit drugs.
On examination, he appeared alert and comfortable. His temperature was 97.3°F (36.3°C), blood pressure 154/71 mm Hg, heart rate 81 beats per minute, respiratory rate 24 breaths per minute, and oxygen saturation 93% while breathing 4 L of oxygen by nasal canula. His jugular venous pressure was approximately 10 cm. The cardiac examination revealed normal first and second heart sounds and regular rate and rhythm. A third heart sound was present. On lung examination, breath sounds were decreased in both bases. Both lower extremities had trace edema. The remainder of the examination was normal.
His laboratory values were as follows:
White blood cell count 5.9 × 109/L (reference range 4.5–11.0)
Electrocardiography showed an old anteroseptal myocardial infarction with ST-segment elevations of less than 1 mm in leads V1, V2, and V3. Chest radiography showed vascular congestion, blunting of the right costophrenic angle likely due to small effusions, no focal consolidation, and no pneumothorax.
WHAT EXACERBATED THIS PATIENT’S HEART FAILURE?
In this patient, causes of shortness of breath such as aspiration pneumonia and myocardial infarction should be considered. But the timing of the symptoms, the laboratory test results, and the chest radiographic findings point to an exacerbation of heart failure as a result of the bowel preparation solution. The physiology of what may have produced this exacerbation will be discussed later.
The ideal preparation for diagnostic and surgical procedures would do the following:
Reliably remove stool from the colon without affecting its appearance
Require only a short amount of time to administer
Cause little or no discomfort for the patient
Produce no significant shifts of fluids or electrolytes.1–4
POLYETHYLENE GLYCOL SOLUTIONS ARE THE MOST POPULAR PREPARATIONS
Polyethylene glycol solutions are by far the most commonly used method of bowel preparation. Polyethylene glycol is a nonabsorbable solution that, in theory, passes through the bowel without net absorption or secretion, thereby avoiding significant fluid and electrolyte shifts. Large volumes (4 L) are required to achieve a cathartic effect, and since many patients find this difficult to tolerate, instructing patients to divide the doses—eg, 3 L the night before and 1 L the morning of the procedure—increases compliance.
Low-volume polyethylene glycol preparations such as HalfLytely and MiraLax were developed to improve patient tolerance by reducing the amount of solution required, while still maintaining efficacy by adding bisacodyl or magnesium citrate. Studies have shown 2-L solutions to be as effective as 4-L solutions in terms of colon cleansing, and to be better tolerated.5–8
Is some water from polyethylene glycol solutions absorbed?
Several bowel preparation methods expand the intravascular volume, which can in turn exacerbate heart failure, since patients with left ventricular systolic dysfunction are unable to adjust to changes in volume.
High-volume gut lavage uses high volumes of saline solution or balanced electrolye solution, leading to significant fluid and electrolyte shifts. Because of these significant side effects, high-volume gut lavage is no longer recommended.9
Goldman and Reichelderfer10 evaluated 43 patients in whom polyethylene glycol was used for bowel preparation; they measured body weight, hematocrit, and serum electrolyte levels before and after the preparation to detect fluid and electrolyte shifts, but they observed no shifts.
Several authors proposed that this method was insensitive for detecting changes in plasma volume. Turnage et al11 used an isotope dilution technique involving iodine-125-labeled human serum albumin to measure plasma volume before and after whole-gut irrigation with a polyethylene glycol solution. Although there was no significant change in body weight or electrolytes, the mean plasma volume increased by 5.88 ± 2.4%, with an increase in plasma volume of 12.9 ± 2.6% in patients over age 70 years and 17.45 ± 5% in patients taking more than 20 mg of furosemide daily. Thus, patients physiologically unable to compensate for an acute change in intravascular fluid, such as those with left ventricular dysfunction, are the ones most likely to have an increase in plasma volume after bowel preparation.11,12 However, this phenomenon appears to be rare, as a literature search on Medline found only one other case report of a heart failure exacerbation with a polyethylene glycol preparation.12
SODIUM PHOSPHATE PREPARATIONS CAN CAUSE FLUID DEPLETION
Aqueous sodium phosphate preparations such as Fleet are low-volume hyperosmotic solutions containing monobasic and dibasic sodium phosphate. They clean the colon by osmotically drawing plasma water into the bowel lumen.
However, this osmotic shift can cause serious volume contraction, electrolyte abnormalities such as hyperphosphatemia, and even death.1 Serious problems are more likely to occur in elderly patients, but also in patients with bowel obstruction, poor gut motility, renal failure, liver failure, or congestive heart failure.
Studies have shown hemodynamic changes indicating hypovolemia during sodium phosphate ingestion.13,14 One reported that 28% of patients had decreases in systolic blood pressure of more than 10 mm Hg, and 12% had decreases in systolic blood pressure of more than 20 mm Hg.13,14
In a study by Barclay et al,15 the concomitant use of a carbohydrate-electrolyte rehydration solution with the sodium phosphate limited the amount of fluid contraction.15 A similar study by Tjandra and Tagkalidis16 using a carbohydrate-electrolyte replacement solution called E-lyte showed a reduction in the degree of intravascular volume contraction, hypokalemia, and need for intravenous rehydration, although there was no effect on hyperphosphatemia resulting from ingestion of sodium phosphate.16
Phosphate nephropathy has also been reported after use of these solutions in patients with renal insufficiency, dehydration, or hypercalcemia, as well as in patients taking angiotensin-converting enzyme inhibitors or angiotensin receptor blockers.9
Current recommendations of the American Society of Colon and Rectal Surgeons, the American Society of Gastrointestinal Endoscopy, and the Society of American Gastrointestinal and Endoscopic Surgeons state that aqueous sodium phosphate is the preferable form of sodium phosphate and is an equal alternative to polyethylene glycol solutions except in pediatric and elderly patients, patients with bowel obstruction and other structural intestinal disorders, and patients with poor gut motility, renal failure, liver failure, or congestive heart failure.9 In fact, because of the significant volume contraction and electrolyte abnormalities that can occur, these groups list sodium phosphate preparations as contraindicated in patients with congestive heart failure.9
POLYETHYLENE GLYCOL IS STILL SAFER FOR THOSE WITH HEART FAILURE
Polyethylene glycol preparations have been shown to cause an increase in plasma volume in patients with disease states that predispose them to fluid retention.11 It has been postulated that these adverse effects may occur less often with preparations that use a lower volume, such as the 2-L polyethylene glycol regimen (HalfLytely) combined with bisacodyl or the 2-L polyethylene glycol 3350 solution (MiraLax). However, our patient experienced a heart failure exacerbation while drinking a reduced-volume preparation.
Another concern with polyethylene glycol solutions, especially HalfLytely, is hyperkalemia. Although no clinical reports have shown this finding, the small amount of potassium in this solution could be worrisome in patients with heart failure who are taking potassium-sparing diuretics or angiotensin-converting enzyme inhibitors.9
Nonetheless, when one considers the risks of fluid shifts with sodium phosphate preparations (which are in any event contraindicated for patients with congestive heart failure), the safest preparation for patients with congestive heart failure would be either a low-volume polyethylene glycol preparation or a split-dose standard-volume polyethylene glycol preparation with careful monitoring during and after use. Clinicians should emphasize to these patients the importance of continuing their cardiac medications during bowel preparation. The patients should also be instructed to notify their doctor about a change in body weight, difficulty breathing, chest tightness, and lower-extremity edema as they are taking the bowel preparation solution.12
Patients with severe left ventricular dysfunction may require short-term intensive care monitoring to manage plasma volume during and after bowel preparation.
A 73-year-old man presented to the emergency department with chest tightness and shortness of breath associated with diaphoresis. His symptoms began as he was taking the bowel preparation solution HalfLytely, 8 oz every 10 minutes as directed, in preparation for colonoscopy the next morning for asymptomatic hematochezia. He had consumed 1 L of the solution when he noticed the symptoms. He tried to continue, but stopped halfway through the second liter because of worsening shortness of breath.
He reported no nausea, vomiting, palpitations, or dizziness. The discomfort (chest tightness) was somewhat relieved with rest and by leaning forward.
His medical history included chronic congestive heart failure with an ejection fraction of 25%; coronary artery disease with multiple myocardial infarctions, coronary angioplasty of the left anterior descending artery, and placement of a cardiac defibrillator; gout; gastritis; glaucoma; hypertension; and hyperlipidemia. His was taking potassium chloride (microK), furosemide (Lasix), clopidogrel (Plavix), enalapril (Vasotec), carvedilol (Coreg), simvastatin (Zocor), folic acid, pantoprazole (Protonix), allopurinol (Zyloprim), colchicine (Colcrys), dorzolamidetimolol eye drops (Cosopt), atropine eyedrops (Atropine Care), and ofloxacin eyedrops (Ocuflox). He said he takes all medications as directed without missing any doses. He smoked 1 pack of cigarettes per day for 50 years (he quit several years ago), and he drinks two to three glasses of brandy weekly. He denied using illicit drugs.
On examination, he appeared alert and comfortable. His temperature was 97.3°F (36.3°C), blood pressure 154/71 mm Hg, heart rate 81 beats per minute, respiratory rate 24 breaths per minute, and oxygen saturation 93% while breathing 4 L of oxygen by nasal canula. His jugular venous pressure was approximately 10 cm. The cardiac examination revealed normal first and second heart sounds and regular rate and rhythm. A third heart sound was present. On lung examination, breath sounds were decreased in both bases. Both lower extremities had trace edema. The remainder of the examination was normal.
His laboratory values were as follows:
White blood cell count 5.9 × 109/L (reference range 4.5–11.0)
Electrocardiography showed an old anteroseptal myocardial infarction with ST-segment elevations of less than 1 mm in leads V1, V2, and V3. Chest radiography showed vascular congestion, blunting of the right costophrenic angle likely due to small effusions, no focal consolidation, and no pneumothorax.
WHAT EXACERBATED THIS PATIENT’S HEART FAILURE?
In this patient, causes of shortness of breath such as aspiration pneumonia and myocardial infarction should be considered. But the timing of the symptoms, the laboratory test results, and the chest radiographic findings point to an exacerbation of heart failure as a result of the bowel preparation solution. The physiology of what may have produced this exacerbation will be discussed later.
The ideal preparation for diagnostic and surgical procedures would do the following:
Reliably remove stool from the colon without affecting its appearance
Require only a short amount of time to administer
Cause little or no discomfort for the patient
Produce no significant shifts of fluids or electrolytes.1–4
POLYETHYLENE GLYCOL SOLUTIONS ARE THE MOST POPULAR PREPARATIONS
Polyethylene glycol solutions are by far the most commonly used method of bowel preparation. Polyethylene glycol is a nonabsorbable solution that, in theory, passes through the bowel without net absorption or secretion, thereby avoiding significant fluid and electrolyte shifts. Large volumes (4 L) are required to achieve a cathartic effect, and since many patients find this difficult to tolerate, instructing patients to divide the doses—eg, 3 L the night before and 1 L the morning of the procedure—increases compliance.
Low-volume polyethylene glycol preparations such as HalfLytely and MiraLax were developed to improve patient tolerance by reducing the amount of solution required, while still maintaining efficacy by adding bisacodyl or magnesium citrate. Studies have shown 2-L solutions to be as effective as 4-L solutions in terms of colon cleansing, and to be better tolerated.5–8
Is some water from polyethylene glycol solutions absorbed?
Several bowel preparation methods expand the intravascular volume, which can in turn exacerbate heart failure, since patients with left ventricular systolic dysfunction are unable to adjust to changes in volume.
High-volume gut lavage uses high volumes of saline solution or balanced electrolye solution, leading to significant fluid and electrolyte shifts. Because of these significant side effects, high-volume gut lavage is no longer recommended.9
Goldman and Reichelderfer10 evaluated 43 patients in whom polyethylene glycol was used for bowel preparation; they measured body weight, hematocrit, and serum electrolyte levels before and after the preparation to detect fluid and electrolyte shifts, but they observed no shifts.
Several authors proposed that this method was insensitive for detecting changes in plasma volume. Turnage et al11 used an isotope dilution technique involving iodine-125-labeled human serum albumin to measure plasma volume before and after whole-gut irrigation with a polyethylene glycol solution. Although there was no significant change in body weight or electrolytes, the mean plasma volume increased by 5.88 ± 2.4%, with an increase in plasma volume of 12.9 ± 2.6% in patients over age 70 years and 17.45 ± 5% in patients taking more than 20 mg of furosemide daily. Thus, patients physiologically unable to compensate for an acute change in intravascular fluid, such as those with left ventricular dysfunction, are the ones most likely to have an increase in plasma volume after bowel preparation.11,12 However, this phenomenon appears to be rare, as a literature search on Medline found only one other case report of a heart failure exacerbation with a polyethylene glycol preparation.12
SODIUM PHOSPHATE PREPARATIONS CAN CAUSE FLUID DEPLETION
Aqueous sodium phosphate preparations such as Fleet are low-volume hyperosmotic solutions containing monobasic and dibasic sodium phosphate. They clean the colon by osmotically drawing plasma water into the bowel lumen.
However, this osmotic shift can cause serious volume contraction, electrolyte abnormalities such as hyperphosphatemia, and even death.1 Serious problems are more likely to occur in elderly patients, but also in patients with bowel obstruction, poor gut motility, renal failure, liver failure, or congestive heart failure.
Studies have shown hemodynamic changes indicating hypovolemia during sodium phosphate ingestion.13,14 One reported that 28% of patients had decreases in systolic blood pressure of more than 10 mm Hg, and 12% had decreases in systolic blood pressure of more than 20 mm Hg.13,14
In a study by Barclay et al,15 the concomitant use of a carbohydrate-electrolyte rehydration solution with the sodium phosphate limited the amount of fluid contraction.15 A similar study by Tjandra and Tagkalidis16 using a carbohydrate-electrolyte replacement solution called E-lyte showed a reduction in the degree of intravascular volume contraction, hypokalemia, and need for intravenous rehydration, although there was no effect on hyperphosphatemia resulting from ingestion of sodium phosphate.16
Phosphate nephropathy has also been reported after use of these solutions in patients with renal insufficiency, dehydration, or hypercalcemia, as well as in patients taking angiotensin-converting enzyme inhibitors or angiotensin receptor blockers.9
Current recommendations of the American Society of Colon and Rectal Surgeons, the American Society of Gastrointestinal Endoscopy, and the Society of American Gastrointestinal and Endoscopic Surgeons state that aqueous sodium phosphate is the preferable form of sodium phosphate and is an equal alternative to polyethylene glycol solutions except in pediatric and elderly patients, patients with bowel obstruction and other structural intestinal disorders, and patients with poor gut motility, renal failure, liver failure, or congestive heart failure.9 In fact, because of the significant volume contraction and electrolyte abnormalities that can occur, these groups list sodium phosphate preparations as contraindicated in patients with congestive heart failure.9
POLYETHYLENE GLYCOL IS STILL SAFER FOR THOSE WITH HEART FAILURE
Polyethylene glycol preparations have been shown to cause an increase in plasma volume in patients with disease states that predispose them to fluid retention.11 It has been postulated that these adverse effects may occur less often with preparations that use a lower volume, such as the 2-L polyethylene glycol regimen (HalfLytely) combined with bisacodyl or the 2-L polyethylene glycol 3350 solution (MiraLax). However, our patient experienced a heart failure exacerbation while drinking a reduced-volume preparation.
Another concern with polyethylene glycol solutions, especially HalfLytely, is hyperkalemia. Although no clinical reports have shown this finding, the small amount of potassium in this solution could be worrisome in patients with heart failure who are taking potassium-sparing diuretics or angiotensin-converting enzyme inhibitors.9
Nonetheless, when one considers the risks of fluid shifts with sodium phosphate preparations (which are in any event contraindicated for patients with congestive heart failure), the safest preparation for patients with congestive heart failure would be either a low-volume polyethylene glycol preparation or a split-dose standard-volume polyethylene glycol preparation with careful monitoring during and after use. Clinicians should emphasize to these patients the importance of continuing their cardiac medications during bowel preparation. The patients should also be instructed to notify their doctor about a change in body weight, difficulty breathing, chest tightness, and lower-extremity edema as they are taking the bowel preparation solution.12
Patients with severe left ventricular dysfunction may require short-term intensive care monitoring to manage plasma volume during and after bowel preparation.
References
Nelson DB, Barkun AN, Block KP, et al. Technology status evaluation report. Colonoscopy preparations. May 2001. Gastrointest Endosc2001; 54:829–832.
Berry MA, DiPalma JA. Review article: orthograde gut lavage for colonoscopy. Aliment Pharmacol Ther1994; 8:391–395.
DiPalma JA, Brady CE. Colon cleansing for diagnostic and surgical procedures: polyethylene glycol-electrolyte lavage solution. Am J Gastroenterol1989; 84:1008–1016.
DiPalma JA, Wolff BG, Meagher A, Cleveland M. Comparison of reduced volume versus four liters sulfate-free electrolyte lavage solutions for colonoscopy colon cleansing. Am J Gastroenterol2003; 98:2187–2191.
Adams WJ, Meagher AP, Lubowski DZ, King DW. Bisacodyl reduces the volume of polyethylene glycol solution required for bowel preparation. Dis Colon Rectum1994; 37:229–233.
Sharma VK, Steinberg EN, Vasudeva R, Howden CW. Randomized, controlled study of pretreatment with magnesium citrate on the quality of colonoscopy preparation with polyethylene glycol electrolyte lavage solution. Gastrointest Endosc1997; 46:541–543.
Stratton S, Shelton P, Carleton V, Griglione GR. Feasibility of polyethylene glycol (PEG) 3350 (Miralax) for colon preparation prior to lower endoscopic examination in healthy adults; experience in a community clinic setting. Am J Gastroenterol2008; 103:2163–2164.
Wexner SD, Beck DE, Baron TH, et al; American Society of Colon and Rectal Surgeons. A consensus document on bowel preparation before colonoscopy: prepared by a task force from the American Society of Colon and Rectal Surgeons (ASCRS), the American Society for Gastrointestinal Endoscopy (ASGE), and the Society of American Gastrointestinal and Endoscopic Surgeons (SAGES). Gastrointest Endosc2006; 63:894–909.
Goldman J, Reichelderfer M. Evaluation of rapid colonoscopy preparation using a new gut lavage solution. Gastrointest Endosc1982; 28:9–11.
Turnage RH, Guice KS, Gannon P, Gross M. The effect of polyethylene glycol gavage on plasma volume. J Surg Res1994; 57:284–288.
Granberry MC, White LM, Gardner SF. Exacerbation of congestive heart failure after administration of polyethylene glycol-electrolyte lavage solution. Ann Pharmacother1995; 29:1232–1235.
Afridi SA, Barthel JS, King PD, Pineda JJ, Marshall JB. Prospective, randomized trial comparing a new sodium phosphate-bisacodyl regimen with conventional PEG-ES lavage for outpatient colonoscopy preparation. Gastrointest Endosc1995; 41:485–489.
Curran MP, Plosker GL. Oral sodium phosphate solution: a review of its use as a colorectal cleanser. Drugs2004; 64:1697–1714.
Barclay RL, Depew WT, Vanner SJ. Carbohydrateelectrolyte rehydration protects against intravascular volume contraction during colonic cleansing with orally administered sodium phosphate. Gastrointest Endosc2002; 56:633–638.
Tjandra JJ, Tagkalidis P. Carbohydrate-electrolyte (E-Lyte) solution enhances bowel preparation with oral fleet phospho-soda. Dis Colon Rectum2004; 47:1181–1186.
References
Nelson DB, Barkun AN, Block KP, et al. Technology status evaluation report. Colonoscopy preparations. May 2001. Gastrointest Endosc2001; 54:829–832.
Berry MA, DiPalma JA. Review article: orthograde gut lavage for colonoscopy. Aliment Pharmacol Ther1994; 8:391–395.
DiPalma JA, Brady CE. Colon cleansing for diagnostic and surgical procedures: polyethylene glycol-electrolyte lavage solution. Am J Gastroenterol1989; 84:1008–1016.
DiPalma JA, Wolff BG, Meagher A, Cleveland M. Comparison of reduced volume versus four liters sulfate-free electrolyte lavage solutions for colonoscopy colon cleansing. Am J Gastroenterol2003; 98:2187–2191.
Adams WJ, Meagher AP, Lubowski DZ, King DW. Bisacodyl reduces the volume of polyethylene glycol solution required for bowel preparation. Dis Colon Rectum1994; 37:229–233.
Sharma VK, Steinberg EN, Vasudeva R, Howden CW. Randomized, controlled study of pretreatment with magnesium citrate on the quality of colonoscopy preparation with polyethylene glycol electrolyte lavage solution. Gastrointest Endosc1997; 46:541–543.
Stratton S, Shelton P, Carleton V, Griglione GR. Feasibility of polyethylene glycol (PEG) 3350 (Miralax) for colon preparation prior to lower endoscopic examination in healthy adults; experience in a community clinic setting. Am J Gastroenterol2008; 103:2163–2164.
Wexner SD, Beck DE, Baron TH, et al; American Society of Colon and Rectal Surgeons. A consensus document on bowel preparation before colonoscopy: prepared by a task force from the American Society of Colon and Rectal Surgeons (ASCRS), the American Society for Gastrointestinal Endoscopy (ASGE), and the Society of American Gastrointestinal and Endoscopic Surgeons (SAGES). Gastrointest Endosc2006; 63:894–909.
Goldman J, Reichelderfer M. Evaluation of rapid colonoscopy preparation using a new gut lavage solution. Gastrointest Endosc1982; 28:9–11.
Turnage RH, Guice KS, Gannon P, Gross M. The effect of polyethylene glycol gavage on plasma volume. J Surg Res1994; 57:284–288.
Granberry MC, White LM, Gardner SF. Exacerbation of congestive heart failure after administration of polyethylene glycol-electrolyte lavage solution. Ann Pharmacother1995; 29:1232–1235.
Afridi SA, Barthel JS, King PD, Pineda JJ, Marshall JB. Prospective, randomized trial comparing a new sodium phosphate-bisacodyl regimen with conventional PEG-ES lavage for outpatient colonoscopy preparation. Gastrointest Endosc1995; 41:485–489.
Curran MP, Plosker GL. Oral sodium phosphate solution: a review of its use as a colorectal cleanser. Drugs2004; 64:1697–1714.
Barclay RL, Depew WT, Vanner SJ. Carbohydrateelectrolyte rehydration protects against intravascular volume contraction during colonic cleansing with orally administered sodium phosphate. Gastrointest Endosc2002; 56:633–638.
Tjandra JJ, Tagkalidis P. Carbohydrate-electrolyte (E-Lyte) solution enhances bowel preparation with oral fleet phospho-soda. Dis Colon Rectum2004; 47:1181–1186.
Hyperosmotic sodium phosphate preparations can cause significant volume contraction in vulnerable patients, along with a risk of hyperphosphatemia.
Low-volume (2-L) polyethylene glycol preparations are more tolerable than 4-L solutions.
Although increases in plasma volume can be detected with polyethylene glycol solutions, these solutions are still the safest choice in patients with heart failure.